0% found this document useful (0 votes)
2 views

G9-MATH

The document is a mathematics module for Grade Nine that focuses on quadratic equations, their applications, and related concepts such as rational expressions and variations. It outlines learning outcomes, chapter contents, and specific lessons aimed at helping students understand and solve quadratic equations and inequalities. The module emphasizes practical applications of quadratic functions in real-life situations and includes quizzes for assessment.

Uploaded by

manikilsadi
Copyright
© © All Rights Reserved
We take content rights seriously. If you suspect this is your content, claim it here.
Available Formats
Download as PDF, TXT or read online on Scribd
0% found this document useful (0 votes)
2 views

G9-MATH

The document is a mathematics module for Grade Nine that focuses on quadratic equations, their applications, and related concepts such as rational expressions and variations. It outlines learning outcomes, chapter contents, and specific lessons aimed at helping students understand and solve quadratic equations and inequalities. The module emphasizes practical applications of quadratic functions in real-life situations and includes quizzes for assessment.

Uploaded by

manikilsadi
Copyright
© © All Rights Reserved
We take content rights seriously. If you suspect this is your content, claim it here.
Available Formats
Download as PDF, TXT or read online on Scribd
You are on page 1/ 330

Philippine Copyright 2022

Published by Victory Elijah Christian College, Inc.

L10B2 (#43) Rose Street, Lourdes Subdivision, Mambugan, Antipolo

Rizal, Philippines, 1870

All rights Reserved

No part of this book may be reproduced, stored in retrieval system, or transmitted in any form
or by any means without prior permission of Victory Elijah Christian College, Inc.

[email protected]

2022
Mathematics
Grade Nine (9)
Module 1

Learning Introduction and


Overview
In different fields of studies like science, technology, and
specially engineering, problems that require nonlinear
solutions are usually encountered. In such case, a quadratic
equation plays a very important role, even in solving real -life
problems. This module will discuss the importance of a
quadratic equation in your everyday life.

The important part of the study of Algebra is learning to


model and solve problems. More often than not, solving
problems include quadratic function -a second degree
polynomial. This module also introduces the concept of
variation or the relationship between two variables. Variation
has a wide application in the field of science. Hooke’s law
applies direct variation while Boyle’s law applies inverse
variation.

3|P age
Mathematics
Grade Nine (9)
Furthermore, exponents are useful in expressing very small
or very large numbers. This module familiarizes how exponents,
whether it is integral or rational exponents are used in many
ways. Explore the world of exponents with this chapter.

Learning Outcomes
Upon successful completion of the module the learners will be
able to:

1. Define and illustrate quadratic equations.


2. Write quadratic equation in the form of ax 2 + bx + c = 0.
3. Solves quadratic equations by: (a) zero product rule; (b)
extracting square roots; (c) factoring; (d) completing
the square; (e) using the quadratic formula.
4. Characterizes the roots of a quadratic equation using
the discriminant.
5. Describe the relationship between the coefficients and
the roots of a quadratic equation.
6. Solves equations transformable to quadratic equations
(including rational algebraic equations.
7. Solves problem involving quadratic equations and
rational algebraic equations.

4|P age
Mathematics
Grade Nine (9)
8. Illustrates quadratic inequalities.
9. Solves quadratic inequalities.
10. Solves problems involving quadratic inequalities.
11. Models real-life situations using quadratic functions.
12. Represents a quadratic functions using: (a) table of
values; (b) graph; (c) equation transforms the quadratic
function defined by y = ax 2 + bx + c into the form of y =
a (x – h) 2 + k.
13. Graphs of quadratic functions: (a) domain; (b)
range; (c) intercepts; (d) axis of symmetry; (e) vertex, (f)
direction of the opening of the parabola.
14. Analyzes the effects of changing the values of a, h
and k in the equation y = a (x – h) 2 + k of a quadratic
functions on its graph.
15. Determine the equation of a quadratic function
given: (a) a table of values; (b) graph; (zeros).
16. Solves problems involving quadratic functions.
17. Define and find the domain of rational expre ssion.
18. Perform operations on rational expression.
19. Solves rational equations.
20. Illustrates situations that involve the following
variations (a)direct; (b)inverse; (c)joint; (d)combined.

5|P age
Mathematics
Grade Nine (9)
21. Translates into variation statement a relationship
between two quantities given by: (a) a table of values;
(b) a mathematical equation; a graph and vice versa.
22. Solves problems involving variation.
23. Writes number in scientific notation and vice versa.
24. Applies the laws involving positive integral exponent to
zero and negative integral exponent.
25. Illustrates expression with rational exponents.
26. Simplifies expression with rational exponent.
27. Writes expressions with rational exponents and radicals
and vice versa.
28. Find the square root and cube root.
29. Derives the laws of radicals using the laws of radicals.

6|P age
Mathematics
Grade Nine (9)
Learning Contents:
CHAPTER 1: SOLVING QUADRATIC EQUATIONS
Lesson 1: Definitions of Quadratic Equations
Lesson 2: Solutions of Quadratic Equations Using the Zero
Product Principle
Lesson 3: Quadratic Equations of the form ax 2 = c
Lesson 4: Solving Quadratic Equations by Factoring
Lesson 5: Solving Quadratic Equations by Completing the
Square
Lesson 6: Solving Quadratic Equations by Using Quadratic
Formula
Lesson 7: The Discriminant
Lesson 8: The Sum and Product of Roots
Lesson 9: Solving Rational Algebraic Equations
Transformable to Quadratic Equations
Lesson 10: Quadratic Inequalities

CHAPTER 2: GRAPHING QUADRATIC FUNCTIONS


Lesson 11: Quadratic Functions
Lesson 12: Parabolas
Lesson 13: Quadratic Functions of the Form
f(x) = a (x – h) 2 + k
7|P age
Mathematics
Grade Nine (9)
Lesson 14: Effects of Changes in the Values of a, h, and k
on the Graphs of Quadratic Function
Lesson 15: Writing a Quadratic Given its Vertex and a Point

Lesson 16: Writing Quadratic Functions Given Three


Intercepts or Three Points
Lesson 17: Applications of Quadratic Functions

CHAPTER 3: SOLVING EQUATIONS INVOLVING RATIONAL


EXPRESSIONS

Lesson 18: Recalling Rational expression and Their Domains


Lesson 19: Recalling Addition and Subtraction of Rational
Expression
Lesson 20: Equations Involving Rational Expressions

CHAPTER 4: VARIATIONS
Lesson 21: Direct Variation
Lesson 22: Direct Square Variation
Lesson 23: Inverse Variation
Lesson 24: Joint Variation and Combined Variation

CHAPTER 5: RATIONAL EXPONENTS AND RADICALS


Lesson 25: Scientific Notation
Lesson 26: Integral Exponents

8|P age
Mathematics
Grade Nine (9)
Lesson 27: Expressions with Rational Exponent
Lesson 28: Square and Cube Root
Lesson 29: Simplifying Radicals
Lesson 30: Rationalizing the Denominator
Lesson 31: Rational Exponents and Simplifying Radicals

9|P age
Mathematics
Grade Nine (9)
LESSON 1

DEFINITIONS OF QUADRATIC EQUATIONS


A quadratic equations is a function that can be described by an equation of
the form y = ax 2 + bx + c, where a, b and c are constants a ≠ 0. An equation such
as y= x 2 – 2x + 4 is an example of a quadrant function. A quadratic equation is also
called a second degree equation in one variable.

Quadratic function is also called quadratic equation and second degree


equation.

Look at following example of quadratic equations in one variable.

x2 – x + 7 = 0

3x 2 – x = 0

-2x 2 + 3 = 0

5x 2 + x – 3 = 0

The given examples are written in descending powers of the variable, and
the right side of each equation is zero. Such quadratic equations are said to be in
standard form

Examples of Quadratic Equations

Quadratic Equation Standard Form Values of a, b and c


x 2 - 2x + 6 = 0 x 2 - 2x + 6 = 0 a= 1 b = -2 c = 6
3x 2 + x = 8 3x 2 + x – 7 = 0 a = 3 b = 1 c = -7
3x 2 = 9 3x 2 – 9 = 0 a = 3 b = 0 c = -9

10| P a g e
Mathematics
Grade Nine (9)
QUIZ

A. Determine whether each equation is quadratic equation or not.

1. (x + 4) + 5 = 0 6. x 2 – 3x + 6 = 12 11. 4x 3 + 3x 3 – 4x = 2x 2 + 3x – 7

2. x 2 + 10 = 4x 7. (x – 4) 2 + 6x = 4x – 6 12. x(x + 3) 2 + 3x - 6= -3x + 12

3. 2(x + 3) 2 = 0 1 13. 4(x – 4) 2 – 3x + 12 = 12x – 9


8. 𝑥 2 = 3x – 4
2

5. (x – 5) 2 + 8 = 9 14. x(x – 5) 2 = 3x + 6 – 12
9. [x (x – 2) 2 - 3] = 7
15. (x – 4) 2 = 3(x – 5) 2 + 4
10. 2x 2 – 3x + 14 = x 3

B. Write each quadratic equation in standard form.

1. 3 – 6(x 2 + 2) = 0 6. x 2 = -3x + 8 11. y = -2x2 - 12x-5

2. (2x – 3) 2 = 5 7. 2x = (3x – 2) 2 12. x = -5x2 - 50x-100

3. 2x 2 + 3x = 4 8. 4 = (2x – 5) 2 13. (x+9) (x+4)=0

4. (x – 5) 2 + 8 = 9 9. 2 (x 2 – 4) = 3x + 8 14. 4(1+ 4x) = x(1-7)

5. x 2 = -3x + 8 10. 5(x 2 + 3x + 8) = 0 15. x(-18 +9x) - 14=-1

C. Identify the value of a, b and c of the following quadratic equations.


4
1. 2x 2 + 3x – 8 = 0 5. 5(x – 1) 2 = 0 9. 2x = 3𝑥 −8

2. x 2 – 4x = 4 6. 2x 2 – (3x – 6) = 0 10. 3x = x(x – 6)

3. 3x 2 = - 2x + 8 7. (x + 4)(x – 5) = 9
11. 3x 2 -4 = 2x - 9
𝑥+3 3𝑥
4. (x + 3) 2 = 9 8. =
4 𝑥
12. 2(2x-5) = -4x(x-1)

11 | P a g e
Mathematics
Grade Nine (9)
LESSON 2

SOLUTIONS OF QUADRATIC EQUATIONS


USING THE ZERO PRODUCT PRINCIPLE
If the product of two numbers is zero, then at least one of the n umbers has
to be zero. This is known as the Zero Product Rule.

Zero Product Rule

If a and b are real numbers and if a•b = 0, then either a = 0 or b = 0.

When we are solving equations, we are trying to find all possible numbers
that make the equation true. If we have a product of factors equal to zero, we will
use the Zero Product Rule to set each factor equal to zero. Then, we will solve each
resulting equation. This will give us a list of all possible real solutions to the original
equation.

Solving Equations Using the Zero Product Rule

It is important to follow these three steps when solving equations by factoring:

1) Be sure that you have 0 on one side of the equation.

2) Be sure that you have a product on the other side of the equation. If not, use
the techniques established in this chapter to factor the other side of the equation
into a product.

3) Once steps #1 and #2 are satis fied, use the zero product rule to set each factor
equal to zero, and solve

12 | P a g e
Mathematics
Grade Nine (9)
Solve the following

1. (x – 3)(x – 4)

Solution: Set each factor equal to zero and solve

(x – 3)(x – 4) = 0 Zero Product Rule

x–3=0 or x–4=0

x=3 x=4

So, our answer is (3, 4)

2. 3x(2x – 5)(x + 4) = 0

Solution: Set each factor equal to zero and solve:

3x(2x – 5)(x + 4) = 0 (Zero Product Rule)

3x = 0 or (2x – 5) = 0 or (x + 4) = 0

3x = 0 or 2x – 5 = 0 or x+4=0

3𝑥 0 2𝑥 5
= = x = -4
3 3 2 2

x=0 x = 5/2 x = -4

So, our solution of x can be – 4, 0, or 5/2. We will write this solution as the
set {– 4, 0, 2.5}.

We always like to list the numbers in the set from the smallest to the largest.
If the problem happens to have no solution, we will write the empty set { }. Some
books will also write ∅ for the empty set, but we stick with “no solution” or { }.

13 | P a g e
Mathematics
Grade Nine (9)
QUIZ

Apply the Zero Product Principle to solve each equation.

1. (x – 5)(x – 6) = 0

2. (2x – 3)(x – 5) = 0

3. (4x – 2)(x – 5) = 0

4. (4x – 16)(x – 4) = 0

5. (3x)(3x – 15)(2x – 12) = 0

6. (x – 5)(x – 3)(3x – 9) = 0

7. 2x(x – 5) = 0

8. 3x(2x – 46) = 0

9. (4x )(3x – 120)(2x – 13) = 0

10. (4x)(12x + 80)(13x – 65) = 0

11. (4x)(x + 4)(12x – 60) = 0

12. (3x)(12x – 200)(13x – 89) = 0

13. (2x)(3x – 45)(4x – 24) = 0

14. (12x – 56)(12x + 56)(2x) = 0

15. (2x)(3x – 100)(12x + 124) = 0

14 | P a g e
Mathematics
Grade Nine (9)
LESSON 3

QUADRATIC EQUATIONS OF THE FORM AX 2 = C

The definition of a square root can be used to solve equations in the form ax 2 = c

Consider quadratic equations of the form ax 2 = c, where c is an integer.

Examples

1. x 2 = 144 2. x2 – 4 = 0

√𝑥 2 = √144 x2 = 4

x = ±12 √𝑥 2 = √4

x=±2

3. x 2 – 10 = 0 4. 4x 2 = 121

4𝑥 2 121
x 2 = 10 =
4 4

121
√𝑥 2 = √10 √𝑥 2 = √ 4

x = ± √10 x = 11/4

15 | P a g e
Mathematics
Grade Nine (9)
QUIZ

A. Solve the following.

1. x 2 – 25 = 0 9. x 2 – 121 = 0

2. x 2 – 144 = 0 10. 25x 2 = 100

3. 4x 2 – 25 = 0 11. 24x 2 = 384

4. x 2 = 169 12. 7x 2 – 252 = 0

5. x 2 = 1024 13. x 2 – 361 = 0

6. 36x 2 – 72 = 0 14. 36x 2 – 289 = 0

7. 4x 2 = 64 15. 25x 2 = 36

8. 3x 2 = 675

B. Solve each quadratic equation.

1. x 2 = 36 6. 100 = 4x 2

2. 2c 2 – 32 = 0 7. 3x 2 = 135

4
3. y 2 = 8. (b – 3) 2 – 6 = 0
100

36 1 2
4. d 2 = 49
9. 1 = 16
x

5. (x – 2)(x + 2) = 12 10. (2x – 8)(2x + 8) = 55

C. Answer the following.

1. Why do you write the square root of 100 as ±10?

2. Explain why x 2 + 100 = 0 has no solution.

16 | P a g e
Mathematics
Grade Nine (9)
LESSON 4

SOLVING QUADRATIC EQUATIONS BY


FACTORING
One way of getting the values of x, which are solutions to the given
equation, is by factoring.

Example: 1. Solve x 2 – 4 = 0.

This equation is in "(quadratic) equals (zero)" form, so it's ready to solve.


The quadratic itself is a difference of squares, so you'll apply the difference-of-
squares formula:

x2 – 4 = 0
(x – 2)(x + 2) = 0
x – 2 = 0 or x + 2 = 0
x = 2 or x = –2

Then the solution is x = –2, 2

Note: This solution may also be formatted as " x = ± 2".

Example 2. Solve x 2 – 5 x = 0.

This two-term quadratic is easier to factor than were the


previous quadratics: You can factor an x out of both terms, taking the x out front.
(Warning: Do not "divide the x off", do not make it magically "disappear", or you'll
lose one of your solutions!)

x(x – 5) = 0
x = 0 or x – 5 = 0
x = 0 or x = 5

Then the solution to x 2 – 5 x = 0 is x = 0, 5

17 | P a g e
Mathematics
Grade Nine (9)
3. Solve x 2 + 5 x + 6 = 0.

This equation is already in the form "(quadratic) equals (zero)" but, unlike
the previous example, this isn't yet factored. The quadratic must first be
factored, because it is only when you MULTIPLY and get zero that you can say
anything about the factors and solutions. You can't conclude anything about
the individual terms of the unfactored quadratic (like the 5 x or the 6), because
you can add lots of stuff that totals zero.

So the first thing I have to do is fa ctor: x 2 + 5 x + 6 = ( x + 2)( x + 3)


Set this equal to zero: ( x + 2)( x + 3) = 0
Solve each factor:
x + 2 = 0 or x + 3 = 0
x = –2 or x = – 3
The solution to x 2 + 5 x + 6 = 0 is x = –3, –2
Checking x = –3 and x = –2 in x 2 + 5 x + 6 = 0:
[–3] 2 + 5[–3] + 6 ?=? 0
9 – 15 + 6 ?=? 0
9 + 6 – 15 ?=? 0
15 – 15 ?=? 0
0 = 0
[–2] 2 + 5[–2] + 6 ?=? 0
4 – 10 + 6 ?=? 0
4 + 6 – 10 ?=? 0
10 – 10 ?=? 0
0 = 0

4. Solve x 2 – 3 = 2 x .

This equation is not in "(quadratic) equals (zero)" form, so you can't try to
solve it yet. The first thing you need to do is get all the terms over on one
side, with zero on the other side. Then can you factor and solve:

18 | P a g e
Mathematics
Grade Nine (9)
x2 – 3 = 2x

x2 – 2x – 3 = 0

( x – 3)( x + 1) = 0

x – 3 = 0 or x + 1 = 0

x = 3 or x = –1

Then the solution to x 2 – 3 = 2 x is x = –1, 3

𝟑
5. Find the quadratic equation in x whose roots are 2 and - 𝟒.
Solution
3
Since x=2 or x = -4
x – 2 = 0 or 4x + 3 = 0
( x – 2)(4x + 3) = 0
The equation is 4x 2 – 5x – 6 = 0

6. Find the quadratic equation in y whose roots are -5 and 8.

Solution

Since y = -5 or y=8
y+5=0 or y –8=0
(y + 5)(y – 8) = 0
The equation is y 2 – 3y – 40 = 0

19 | P a g e
Mathematics
Grade Nine (9)
QUIZ

A. Solve by factoring.

1. x2 = 9 11. x 2 – 3x = 88

2. x 2 – 36 = 0 12. x 2 + 19x + 78 = 0

3. x 2 = 81 13. x 2 = -3x + 108

4. n2 – n = 6 14. x 2 – 4x – 96 = 0

5. 36x 2 + 7x = 15 15. x 2 + 4x – 117 = 0

6. b 2 – 6b = 7

7. x 2 – 7x + 12 = 0

8. x 2 – 4x – 12 = 9

9. 2x 2 – 7x – 8 = 0

10. 3x 2 +x = -4 + 8x

B. Form a quadratic equation in x with the given roots for each of the
following.

4
1. -3, 9 6. - 5 , 6

2
2. 5, 8 7. 3, 3

3
3. 5, 5 8. , 2.5
4

7
4. -4, 6 9. , -6
8

3 7
5. -4, 12 10. , -8
4

20 | P a g e
Mathematics
Grade Nine (9)
LESSON 5

SOLVING QUADRATIC EQUATIONS BY


COMPLETING THE SQUARE
Quadratic equations of the form ax 2 + bx + c = 0 can be solved by factoring.
However, therefore are some equations that cannot be solved by factoring.

Examples:

Problem
Find the roots of the quadratic
equation
The roots are the x intercepts,
where the graph crosses
the x axis. The y value for any
point on the x axis is 0,
so substitute 0 for y .
Rewrite the equation with the
left side in the form x 2 + bx, to
prepare to complete the
square.

Add to the left side to


complete the square, and also
x 2 – 4x + 4 = -1 + 4 to the right side so the equation
is still true.
x2 – 4x + 4 = 3

21 | P a g e
Mathematics
Grade Nine (9)

b = -4, so =
Rewrite the left side as a
binomial squared.

Take the square root of both


sides. We need both the positive
and negative square root, or
or
we’ll miss one of the solutions.

Solve for x . These are the x-


coordinates of the roots.
or

Answer
or

2. Use Complete the Square Method to solve 𝟐𝒙𝟐 + 𝟔𝒙 − 𝟑 = 𝟎

Solution.

First note that the previous ideas were developed for quadratic functions with no
coefficient in front of . Therefore, let divide the equation by 2, to get

3
𝑥 2 + 3𝑥 − =0
2
which equivalent to
3
𝑥 2 + 3𝑥 =
2
3 2
In order to generate a perfect square we add ( ) to both sides of the equation
2
2 2
2
3 3 3
𝑥 + 3𝑥 + ( ) = + ( )
2 2 2

22 | P a g e
Mathematics
Grade Nine (9)
Easy algebraic calculations give

3 2 3 9 15
(𝑥 + ) = + =
2 2 4 4

Taking the square-roots lead to

3 15 3 15
𝑥+ =√ 𝑜𝑟 𝑥 + = −√
2 4 2 4

which give the solutions to the equation

3 15 3 15
𝑥 = − +√ 𝑜𝑟 𝑥 = − − √
2 4 2 4

3. Use completing the square to solve x 2 – 4 x – 8 = 0.

As noted above, this quadratic does not factor, so I can't solve the equation by
factoring. And they haven't given me the quadratic in a form that is ready to
square-root. But there is a way for me to manipulate the quadratic to put it into
that form, and then solve. It works like this:

First, you put the loose number on the other side of the equation:

x2 – 4x – 8 = 0

x 2 – 4 x = 8 Copyright © Elizabeth Rights Res erved

Then you look at the coefficient of the x -term, which is –4 in this case. You take
half of this number (including the sign), giving me –2. Then you square this value
to get +4, and add this squared value to both sides of the equation:

x2 – 4x + 4 = 8 + 4

x 2 – 4 x + 4 = 12

23 | P a g e
Mathematics
Grade Nine (9)
This process creates a quadratic that is a perfect square, and factoring gives me:
( x – 2) 2 = 12

(You know it's a "minus two" inside the parentheses because half of –4 is –2. If you
note the sign when you're finding one -half of the coefficient, then you won't mess
up the sign when you're converting to squared -binomial form.)
Now you can square-root both sides of the equation, simplify , and solve:
( x – 2) 2 = 12

Then the solution is 𝒙 = 𝟐 ± 𝟐√𝟑

4. What must be added to each of the following expressions to obtain a


perfect square.
a. x 2 + 5x b. a 2 + 2ak
Solution
a. x 2 + 5x
5
The coefficient of x is 5. Half of this is
2
5 5
x 2 + 5x +( 2) 2 = (x + 2) 2
25
Therefore must be added.
4

b. a 2 + 2ka
This is a quadratic expression in a 2 . The coefficient of a is 2k. half of this is k.
a 2 + 2ak + k 2 = (a + k) 2
Therefore, k 2 must be added.

24 | P a g e
Mathematics
Grade Nine (9)
QUIZ

A. Solve the Equations by completing the squares.

1. x 2 + 4x = 6 = 0

2. a 2 + 6a = -8

3. 2m 2 + 3m + 1 = 0

4. 3n 2 + 4n + 1 = 0

5. 4x 2 + 8x + 3 = 0

6. p 2 – 8p = 20

7. y 2 + 10y – 5 = 0

8. -4 = a 2 – 6a

9. b 2 – 6b = 2

10. t 2 + 18t = 45

B. What must be added to each of the following expressions to obtain a


perfect square.

1. x 2 + 7x 6. x 2 – 3x

7
2. x 2 + 2x 7. x 2 – 1.8x

14
3. a 2 + 2.4a 8. c 2 + c
3

4 7
4. y 2 - 5y 9. v 2 - 2v

5. b 2 – 10bk 10. g 2 – 5gk

25 | P a g e
Mathematics
Grade Nine (9)
LESSON 6

SOLVING QUADRATIC EQUATIONS BY USING


QUADRATIC FORMULA
Somebody (possibly in seventh-century India) was solving a lot of quadratic
equations by completing the square. At some point, you noticed that you were
always doing the exact same steps in the exact same order for every equation.
Taking advantage of the one of the great powers and benefits of algebra (namely,
the ability to deal with abstractions, rather than having to muck about with the
numbers every single time), he made a formula out of what he'd been doing:

The Quadratic Formula:


For ax 2 + bx + c = 0, the value of x is given by:

The nice thing about the Quadratic Formula is that the Quadratic Formula
always works. There are some quadratics (most of them, actually) that you can't
solve by factoring. But the Quadratic Formula will always spit out an answer,
whether the quadratic was factorable or not.

You have a lesson on the Quadratic Formula, which gives examples and
shows the connection between the discriminant (the stuff inside the square root),
the number and type of solutions of the quadratic equation, and the gra ph of the
related parabola. So you'll just do one example here. If you need further
instruction, study the lesson at the above hyperlink.

Let's try that last problem from the previous section again, but this time
we'll use the Quadratic Formula:

26 | P a g e
Mathematics
Grade Nine (9)
Examples:
1. Solve x 2 + 3 x – 4 = 0
This quadratic happens to factor:
x 2 + 3 x – 4 = ( x + 4)( x – 1) = 0

...so you already know that the solutions are x = –4 and x = 1. How about by
using Quadratic Formula? Using a = 1, b = 3, and c = –4, you’ll plug them into
the Formula, and simplify:

Then, as expected, the solution is x = –4, x = 1.

2. Use the Quadratic Formula to solve x 2 – 4 x – 8 = 0.

Looking at the coefficients, you see that a = 1, b = –4, and c = –8. You’ll plug
them into the Formula, and simplify. You should get the same answer as
before:

Then the solution is 𝒙 = 𝟐 ± 𝟐√𝟑

27 | P a g e
Mathematics
Grade Nine (9)

3. Solve 2 x 2 – 4 x – 3 = 0. Round your answer to two decimal places,


if necessary.

There are no factors of (2)(–3) = –6 that add up to –4, so you know that this
quadratic cannot be factored. you will apply the Quadratic Formula. In
this case, a = 2, b = –4, and c = –3:

28 | P a g e
Mathematics
Grade Nine (9)
QUIZ

A. Solve each of these equations using the quadratic formula.

1. x 2 – 8x + 16 = 0

2. 3p 2 – 3p + 9 = 0

3. y 2 – 3y – 28 = 0

4. m 2 + 6m + 9 = 0

5. 5x 2 +16x + 3 = 0

6. 3x 2 – 108 = 0

7. x2 – x + 1 = 0

8. 2s 2 + 5s = 9

9. 7x 2 – 3x – 2 = 0

10. x 2 – 7x + 12 = 0

B. Identify the value of a, b and c and then use the quadratic formula
to solve for the real solutions of each equation.

1. x 2 + 3x – 12 = 0

2. x 2 + 3x = -4x -12

3. 4x – 8 = x 2 + 2x – 6

4. x 2 + 3x – 8 = 19

5. 2x 2 + 3x = 2x + 20

29 | P a g e
Mathematics
Grade Nine (9)
6. 4x – 6 = x 2 + 12x

7. 2x 2 + 3x = 4x – 24

8. 3x – 4 = 4x 2 + 2x – 6

9. 2x 2 + 4x – 2x = 3x – 16

10. 2x + 3 = 2x 2 + 3x

11. x 2 + 3x = -4x + 6

12. x 2 + 5x – 6 = 2x + 5

13. 2x 2 + 3x = 4x – 6

14. 4x 2 = 3x 2 + 4x – 4

15. 6x 2 + 4x = -3x + 6

C. The rectangle and the square below have th e same area. Write a
quadratic equation to indicate that th e two areas are equal, and then
solve the equation. What is the area of each figure?

30 | P a g e
Mathematics
Grade Nine (9)
LESSON 7

THE DISCRIMINANT
Most of the quadratic equations have one or two solutions, but sometimes,
a given equation has no real number solution.

We can tell whether a quadratic equation has two or no roots by evaluating


the radicand in the quadratic formula. The quantity b 2 – 4ac is called the
discriminant D. The value of the discriminant gives information on the number of
distinct roots of quadratic equation. Consider any quadratic equation in the form
ax 2 + bx + cx = 0.

Discriminant Kinds of Solution for

b 2 – 4ac Ax 2 + bx + c = 0
1. b 2 – 4ac > 0 If the value of the discriminant is
positive, then the equation has two real
roots, and this is called two distinct
real solutions.
2. b 2 – 4ac = 0 If the value of the discriminant is zero
(0), then the equation has one real root
and this called one real solution.
3. b 2 – 4ac < 0 If the value of the discriminant is
negative, then the equation has no
distinct real roots. These non-real roots
are called imaginary roots. The solution
is called two distinct imaginary
solution.

31 | P a g e
Mathematics
Grade Nine (9)

Examples:

Consider the following equations and solve. Use the discriminant b 2 – 4ac in
the equation.

1. x 2 + 2x + 1 = 0

D = b 2 – 4ac, where a = 1, b = 2, c = 1

= 2 2 – 4(1)(1) = 4 – 4 D = 0 (one real root)

2. x 2 + 3x + 2 = 0

D = b 2 – 4ac, where a = 1, b = 3, c = 2

= (3) 2 - 4(1)(2) = 9 – 8 = 1 (two distinct real roots)

32 | P a g e
Mathematics
Grade Nine (9)
QUIZ

A. State the number of real roots in the following quadratic equations.

1. 3x 2 + 2x + 4 = 0

2. 2x 2 + 4x – 8 = 0

3. 4x 2 + 4x + 1 = 0

4. 5x 2 – x + 1 = 0

5. 7x 2 – x – 1 = 0

6. 2m 2 – 5m = 2

7. 3y 2 + 5y + 1 = 0

8. 2y 2 = -5y + 5

9. 5p 2 – 6(p -1) = 0

10. x 2 + 4(x + 3) = 0

B. Compute the value of discriminant for each equation, and then


describe the nature of the roots. If the roots are real, find them.

1. p 2 + 5p – 3 = 0

2. x 2 – 5x – 4 = 0

3. m 2 = 6m + 9

4. 6x 2 + 6 = -10x

5. –x 2 + x + 6 = 0

6. 2x 2 – 4x + 1 = 0

33 | P a g e
Mathematics
Grade Nine (9)
7. 2x 2 – 5x + 1 = 0

8. 2n 2 – n = -3

9. x 2 – x = 1

10. x 2 + 1 = x

11. y 2 + 3y = -1

12. 4x 2 – 2x = 12

2 2
13. y – 3y = -1
3

14. 0.5c 2 – 0.2c = 1.4

1 1
15. − 2 𝑠 2 + 𝑠+1=0
3

C. A square piece of cardboard is to be formed into a box by cutting out


from each corner equal squares measuring 4 cm on a side and then folding
the sides. If the volume of the box is to be 576 cm 3 , what is the original
size of the cardboard?

34 | P a g e
Mathematics
Grade Nine (9)
LESSON 8

THE SUM AND PRODUCT OF ROOTS


Most often, scientists must find an equation to describe a certain situation

Given a quadratic equation in the form ax 2 + bx + cx = 0 , where the


quadratic formula is

If we let r be the first root and s the second root, and then, if we solve the r + s we
will get the sum of the roots is the additive inverse the quotient of b and a

−𝑏
r+s= 𝑎

Examples: Find the sum of the roots of the following quadratic equations:

1. 2x 2 – 4x + 5 = 0,

where a = 2, b = -4, c = 5

−𝑏 −(−4) 4
Sum = = = =2
𝑎 2 2

2. 3x 2 + 4x - 9 = 0,

where a = 3, b = 4, c = -9

−𝑏 −4
Sum = =
𝑎 3

3. 4x 2 + 3x = -5

4x 2 + 3x + 5 = 0

where a = 4, b = 3, c = 5

35 | P a g e
Mathematics
Grade Nine (9)
−𝑏 −3
Sum = =
𝑎 4

Given a quadratic equation in the form ax 2 + bx + cx = 0 , where the quadratic


formula is

If we let r be the first root and s the second root, and then, if we solve the rs
we will get the product of the roots is the quotient of c and a
𝑐
rs = 𝑎

Find the product of the roots of the following quadratic equations:


1. 2x 2 – 4x + 5 = 0
where a = 2, b = -4, c = 5
𝑐 5
Product = =2
𝑎

2. 3x 2 + 4x - 9 = 0
where a = 3, b = 4, c = -9
𝑐 −9
Product = 𝑎
= 3
= -3
3. 4x 2 + 3x + 8 = 0
where a = 4, b = 3, c = 8
𝑐 8
Product = = =2
𝑎 4

When both sides of the standard form ax 2 + bx + c = 0 is divided by a ≠ 0,


𝑏 𝑐
then it becomes x 2 + 𝑥+ = 0. The quadratic equation can then be written as x 2
𝑎 𝑎
– (sum of the roots(x) + (product of the roots) = 0

This relationship is useful in writing the quadratic equation whose roots are
given.

36 | P a g e
Mathematics
Grade Nine (9)
1. What is the quadratic equation whose roots are 2 and -5?

Solution

s 1 + s 2 = 2 + (-5) s 1 s 2 = 2(-5)

= -3 = -10

Therefore, the equation is x 2 – (sum of the roots)x + (product of roots) = 0

x 2 – (-3)x + (-10) = 0

x 2 + 3x – 10 = 0

2. What is the quadratic equation whose roots are 5 + √𝟑 and 5 - √𝟑?


Solution
s 1 + s 2 = (5 + √3 )+ (5 - √3) = 10
s 1 s 2 = (5 + √3 ) (5 - √3) = 25 – 3 = 22
Therefore, the equation is
x 2 – 10x + 22 = 0

3. Find a quadratic equation whose roots are the reciprocals of the roots
of the quadratic equation x 2 + x – 6 = 0

Solution:

Find the sum and product of the roots of the given quadratic equation.

x2 + x – 6 = 0 a=1 b=1 c = -6

−𝑏 −1
s1 + s2 = = = −1
𝑎 1

𝑐 −6
s1s2 = = = −6
𝑎 1

37 | P a g e
Mathematics
Grade Nine (9)
1 1
The reciprocals of the roots are and , which are the roots of the quadratic
𝑠1 𝑠2
equation we want to find.

The sum and product of these roots are obtained as follows.

Substituting the values above, we have

1 1 𝑠2 + 𝑠1 −1
𝑠1
+ 𝑠2
= 𝑠1 𝑠2
= −6
=6 Since s 1 + s 2 = -1

1 1 1 1 −1
∙ = = = Since s 1 s 2 = -6
𝑠1 𝑠2 𝑠1 𝑠2 −6 6

1
Thus, the quadratic equation is x 2 – 6x - =0
6

Other equivalent equations may also be obtained by multiplying each term of the
equation by any nonzero constant.

38 | P a g e
Mathematics
Grade Nine (9)

QUIZ

A. Find the sum of the roots in the following equations.

1. 3x 2 – 4x = 9 = 0

2. 4x 2 – x – 10 = 0

3. 3x 2 + 9x – 11 = 0

4. 4x 2 = -3x + 5

5. m 2 + 8m = 4

6. 3y 2 = 5y + 8

7. 6x 2 – 3 = 4x

8. p(p + 3) = 4p + 5

9. m(2m + 7) = 3

10. x 2 + 3x – 28 = 0

B. Find the product of the roots in the following equations.

1. x 2 – 5x + 6 = 0

2. 3m 2 + 6m – 3 = 0

3. 4x 2 + 4x = 35

4. y 2 – 8y – 20 = 0

5. 4x 2 +20x – 16= 0

39 | P a g e
Mathematics
Grade Nine (9)
6. 6a 2 – 13a = 15

7. b 2 = 12b – 28

8. 4y 2 + 8y + 3 = 0

9. 6b 2 – 5b = 21

10. x 2 – 2x – 21 = 0

C. Write the quadratic equation whose sum and product of roots are
given.

Sum Product Equation

1. 2 6

2. -3 5

3. 4 -8

4. 5 6

5. 6 5

6. -3 2
3

7. 4 6

7

8. 5 1
2

9. 3 3
7 8

10. 4 4
5 5

40 | P a g e
Mathematics
Grade Nine (9)

D. Write a quadratic equation for the given of the roots.

1. 2 and 3 6. 4 and 7

3
2. 4 and 8 7. and – 4
2

1
3. 4 and 5 8. 8
and 6

4. 5 and -5 9. 4 + √3 and 4 - √3

5. 6 and -6 10. 2 + √5 and 2 - √5

41 | P a g e
Mathematics
Grade Nine (9)
LESSON 9
SOLVING RATIONAL ALGEBRAIC EQUATIONS
TRANSFORMABLE TO QUADRATIC
EQUATIONS
Solving rational equations may result in quadratic equations as in the
following example.
Example 1
2 1
+ =1
𝑦+1 𝑦−1
To solve this equation, we recall that to solve a ratio nal equation we
must multiply both sides by the least common denominator then solve the
remaining equation. In this case the LCD is (y + 1) (y – 1). So we solve as follows

2 1
+ =1
𝑦+1 𝑦−1
2 1
(y + 1)(y – 1) + = 1(y + 1)(y – 1)
𝑦+1 𝑦−1

2(y – 1) + (y + 1) = (y + 1) (y – 1)
2y – 2 + y + 1 = y 2 – 1
y 2 – 3y = 0
y (y – 3) = 0
y = 0, 3

Since we started with a rational equation (which has a limited domain) we


must make sure that we have no extraneous solutions. To do this we simply need
to check that neither solution makes the original equation undefined. That is, does
either solution give the original equation a zero in the denominator.

Since it is clear that the only values that make the denominator zero are –1
and +1, our solutions are not extr aneous.

Thus our solution set is . {0, 3}

42 | P a g e
Mathematics
Grade Nine (9)
Notice that the quadratic equations in our examples all factored nicely. If
the equation does not factor, simply use the quadratic formula to solve. If the
equation required a substitution to solve, be sure to re -substitute and finish solving
after using the quadratic formula. To check the answers simply use a decimal
approximation.

Example 2

𝒙 𝟏 𝟐
Solve the following equation:
𝒙−𝟐
+ 𝒙−𝟒 = 𝒙𝟐−𝟔𝒙+𝟖

First we need to factor that quadratic, so we can tell what factors we will
have in our common denominator.
x 2 – 6x + 8 = (x – 4)(x – 2)

That worked out nicely: the factors of the quadratic are duplicates of the
other denominators. (This often happens for these problems.) So the common
denominator will be (x – 4)(x – 2)., and we will need to remember (at the end)
that x cannot be 2 or 4.

We can convert everything to the common denominator and then solve the
numerators:

(x 2 – 4x) + (x – 2) = 2
x 2 – 4x + x – 2 = 2
x 2 – 3x – 4 = 0
(x – 4)(x + 1) = 0
x = 4 or x = –1

43 | P a g e
Mathematics
Grade Nine (9)
...or we can multiply through on both sides by the common denominator and solve
the resulting equation:

x(x – 4) + 1(x – 2) = 2
x 2 – 4x + x – 2 = 2
x 2 – 3x – 2 = 2
x 2 – 3x – 4 = 0
(x – 4)(x + 1) = 0
x = 4 or x = –1

Either way, we get the same result: x = 4 and x = –1. Checking these solutions
against the denominators of the original equation, we see that "x = 4" would cause
division by zero, so we throw that solution out. Then the answer is:
x = –1

44 | P a g e
Mathematics
Grade Nine (9)
QUIZ

A. Solve the following equations.

𝑥+8
1. = 𝑥2
5

𝑥+9 1
2. =
𝑥2 4

𝑥2 𝑥−5
3. 2
= 3

𝑥+3
4. = 𝑥−2
𝑥−5

𝑥2 𝑥−4
5. =
4 2

𝑥+3 𝑥−1
6. = 2𝑥−1
𝑥−5

𝑥+4 𝑥−2
7. =
𝑥 5

𝑥+7 2𝑥−1
8. =
5+𝑥 𝑥+1

𝑥+2 1
9. 4
= 𝑥−1

𝑥+1 4 𝑥
10. + =
3 𝑥−2 4

B. Solve each of the following equations by firstly rewrit ing each


equation in quadratic form.

8 3
1. - =5
𝑥 𝑥+1

6 3
2. + = 6
𝑥 𝑥−1

45 | P a g e
Mathematics
Grade Nine (9)
8 3
3. + =5
𝑥−1 𝑥−2

11 5
4. + =3
𝑥+3 3𝑥−7

1 1 1
5. − =
𝑥+3 2𝑥+3 18

3 5 1
6. + =
𝑥−2 𝑥−6 𝑥

1 3 1
7. 2𝑥−1
− 𝑥+5
= 𝑥+1

𝑥+1 2𝑥−1
8. =
𝑥+4 𝑥+6

𝑥 6 4 𝑥
9. + = +
6 𝑥 𝑥 4

2 1 3
10. + +
2𝑥−1 𝑥−4 𝑥−2

2 1 1
11. + =
𝑥−3 2𝑥−6 2𝑥

1 3 2
12. − =
2𝑥−4 2𝑥+10 𝑥+5

𝑥+2 2𝑥−2
13. =
𝑥+4 2𝑥+6

𝑥 2 3 𝑥
14. + = +
8 𝑥 𝑥 5

1 1 2
15. + +
𝑥−4 2𝑥−2 2𝑥−6

46 | P a g e
Mathematics
Grade Nine (9)
LESSON 10

QUADRATIC INEQUALITIES
Quadratic inequalities are inequalities with the highest power of the
unknown being two.

Examples. 3x 2 - 2x + 1 > 0 and 4x 2 + 5x < 7 are quadratic inequalities.

The critical values of an inequality are the solutions of the quadratic


equation formed by replacing the inequality symbol by “=”.

Example: Find the critical values of the inequality


x 2 – 3x ≤ 10
x 2 – 3x = 10 (-10)
x 2 – 3x – 10 = 0
(x + 2)(x – 5) = 0
Either x = -2 or x = 5.
-2 and 5 are the critical values.

The critical values split the number line into three possible solution sets.

Example: Find the possible solution sets of x 2 – 3x ≤ 10

The critical values of the inequality are –2 and 5.

47 | P a g e
Mathematics
Grade Nine (9)
The possible solution sets are:

x≤ −2, −2 ≤ 𝑥 ≤ 5, 5 ≤ 𝑥

Notice that we include the “or equal to” part of the inequality sign in the
possible solution sets as the “or equal to” part was allowed in the original
inequality. Otherwise we wouldn’t have included it.

To solve a quadratic inequality we should:

• Rearrange to obtain 0 as one side of the i nequality, as with an equation.


• Find the critical values.
• Describe the possible solution sets.
• Identify the correct solution set by testing a point in each possible set to see
whether it satisfies the inequality.
Example 1: Find the solution set of the inequality x 2 ≤ 16
First rearrange
x 2 – 16 = 0
x 2 – 16 ≤ o
The find the critical values
x 2 – 16 = 0
(x – 4)(x + 4) = 0
x = 4 or x = -4
Identify the possible solution sets.

x ≤ - 4, -4 ≤ x ≤ 4, 4 ≤ x

48 | P a g e
Mathematics
Grade Nine (9)
Possible set x≤-4 -4 ≤ x ≤ 4 4≤x

Point in set - 5 0 5

Does it satisfy the no yes no


inequality

we see that – 5 does not satisfy the inequality by substituting it into the
original inequality.

4 × (-5) 2 ≤ 64

Clearly this is false therefore the solution set is - 4 ≤ x ≤ 4.

Example 2: Solve the inequality x 2 – 7x > 30

x 2 – 7x > 30

x 2 – 7x – 30 > 0

x 2 – 7x – 30 = 0

(x – 10)(x + 3) = 0

x = 10, x = -3

Set X < -3 -3 < x < 10 10 < x


Point -10 1 20
Satisfy? yes No Yes

The solution sets are x < -3 and 10 < x

49 | P a g e
Mathematics
Grade Nine (9)
QUIZ

A. Find the critical values of the following inequalities.

1. x2 – 4 > 0
2
2. x – 6x < 16
3. x 2 – 6x ≤ 0
4. 2x 2 + 5x ≤ -2
2
5. +x≤3
𝑥

6. x 2 + 3x < -2
7. x 2 – 16 > 0
8. x 2 – 8x + 16 < 0
9. x 2 + 10x ≤ -25
10. x 2 + 6x + 9 ≤ 0

B. Fins the solution sets for the following inequalities.

1. x2 – 4 > 0

2. x 2 – 6x < 16

3. x 2 – 6x ≤ 0

4. 2x 2 + 5x ≤ -2
2
5. +x≤3
𝑥

6. 5x 2 – 6x < -1

7. 6x 2 > 14 – 17x

8. x 2 + 8x + 12 > 0

9. x 2 + 2x + 1 < 0

10. x 2 + 3x + 2 ≥ 0

50 | P a g e
Mathematics
Grade Nine (9)
LESSON 11

QUADRATIC FUNCTIONS
In algebra, a quadratic function, a quadratic polynomial,
a polynomial of degree 2, or simply a quadratic, is a polynomial function in
one or more variables in which the highest -degree term is of the second degree.
For example, a quadratic function in three variables x , y, and z contains
exclusively terms x 2 , y 2 , z 2 , xy , xz , yz , x , y , z , and a constant:

The graphs of quadratic function have a common characteristic. They all have
a general shaped called parabola. The vertical line containing the minimum point
is the axis of symmetry. When the graph is in the minimum point, it opens upward,
and when the graph is in the maximum point, it opens downward. The point (0,0)
is called the vertex of the parabola.

If the graph of a linear equation in two variables is a straight line, the graph
of a quadratic equation is a curve.

Example 1: Graph the parabola .

Compare the equation with to find the values of a , b , and c .

Here, a = 1, b = –7, and c = 2.

Use the values of the coefficients to write the equation of axis of symmetry.

The graph of a quadratic equation in the form y = ax 2 + bx + c has as its axis of

symmetry the line . So, the equation of the axis of symmetry of the

given parabola is or .

Substitute in the equation to find the y -coordinate of the vertex.

51 | P a g e
Mathematics
Grade Nine (9)

Therefore, the coordinates of the vertex are

Now, substitute a few more x -values in the equation to get the corresponding y -
values.

Plot the points and join them to get the parabola.

52 | P a g e
Mathematics
Grade Nine (9)

Example 2:

Graph the parabola .

Compare the equation with to find the values of a , b , and c .

Here, a = –2, b = 5, and c = –1.

Use the values of the coefficients to write the equation of axis of symmetry.

The graph of a quadratic equation in the form y = ax 2 + bx + c has as its axis of


𝑏
symmetry the line 𝑥 = − 2𝑎 . So, the equation of the axis of symmetry of the giv en
5 5
parabola is 𝑥 = − 𝑜𝑟 𝑥 = .
2(−2) 4

5
Substitute 𝑥 = 4 in the equation to find the y -coordinate of the vertex.

5 17
Therefore, the coordinates of the vertex are , .
4 8
Now, substitute a few more x -values in the equation to get the corresponding y -
values.

53 | P a g e
Mathematics
Grade Nine (9)
Plot the points and join them to get the parabola.

54 | P a g e
Mathematics
Grade Nine (9)
QUIZ

A. Graph the following equations.


1. y = -3x 2
2. y = x 2 + 5x + 6
3. y = 5x 2
4. y = -x 2 + 2x + 3
5. y = x2 – x – 4
6. y = x2 + x + 3
7. y = x 2 + 6x + 8
8. y = 8x + 3 +x 2
9. y = -x 2 + 4x + 5
10. y = 3x 2 + 6x + 16
B. Determine whether each relation describe a quadratic function
1. y = 4x 2
2. y = 4(x + 3)
3. y = 3 – 6(x + 2)
4. 2xy = 3
5. y = (2x – 3) 2
6. y = 3x 2 = 2x – 1
7. y = 2(x -1)
8. 3(x – 1) = y – (x + 1)(x – 1)
9. 3x = 8y
10. 2(x – 2) 3 = y

55 | P a g e
Mathematics
Grade Nine (9)
LESSON 12
PARABOLAS

The general form of a quadratic function is f ( x ) = ax 2 + bx + c . The graph of


a quadratic function is a parabola, a type of 2-dimensional curve.
The "basic" parabola, y = x 2 , looks like this:

The function of the coefficient a in the general equation is to make the parabola
"wider" or "skinnier", or to turn it upside down (if negative):

If the coefficient of x 2 is positive, the parabola opens up; otherwise it opens down.

The Vertex

The vertex of a parabola is the point at the bottom of the "U" shape (or the top,
if the parabola opens downward).
The equation for a parabola can also be written in "vertex form":

y = a(x – h)2 + k
In this equation, the vertex of the parabola is the point ( h , k ).

56 | P a g e
Mathematics
Grade Nine (9)

You can see how this relates to the standard equation by multiplying it out:

y = a ( x – h )( x – h ) + k
y = ax 2 – 2 ahx + ah 2 + k

The coefficient of x here is –2 ah . This means that in the standard


𝑏
form, y = ax 2 + bx + c , the expression − 2𝑎 gives the x -coordinate of the vertex .

Example:

Find the vertex domain and range of the parabola.


y = 3 x 2 + 12 x – 12

Here, a = 3 and b = 12. So, the x -coordinate of the vertex is:


12
− = −2
2(3)
Substituting in the original equation to get the y -coordinate, we get:
y = 3(–2) 2 + 12(–2) – 12
= –24

So, the vertex of the parabola is at ( –2, –24).

Domain: x ϵ R Range: y ≥ -24

57 | P a g e
Mathematics
Grade Nine (9)
The Axis of Symmetry

The axis of symmetry of a parabola is the vertical line through the vertex. For a
parabola in standard form, y =ax 2 + bx + c , the axis of symmetry has the equation

Note that – b /2 a is also the x -coordinate of the vertex of the parabola.

Example:

Find the axis of symmetry , domain and range.

y = 2x2 + x – 1
Here, a = 2 and b = 1. So, the axis of symmetry is the vertical line

Range:

Other solution to get the range


4𝑎𝑐− 𝑏2
y≥ 4𝑎

4(2)(−1)− (1)2
y≥ 4(2)

(−8)− (1)
y≥
4(2)

(−8)− (1)
y≥ 4(2)

−9
y ≥8

58 | P a g e
Mathematics
Grade Nine (9)
9
Domain : x ϵ R Range: y ≥ -
8

Remember: for Range


𝟒𝒂𝒄− 𝒃𝟐
y = ax 2 + bx + c, a > 0 {𝒚|𝒚 𝝐 𝑹, 𝒚 ≥ }
𝟒𝒂

𝟒𝒂𝒄− 𝒃𝟐
y = ax 2 + bx + c, a < 0 {𝒚|𝒚 𝝐 𝑹, 𝒚 ≤ }
𝟒𝒂

Intercepts

You can find the y -intercept of a parabola simply by entering 0 for x . If the
equation is in standard form, then you can just take c as the y -intercept. For
instance, in the above example:
y = 2(0) 2 + (0) – 1 = –1
So the y -intercept is –1.
The x -intercepts are a bit trickier. You can use factoring , or completing the square,
or the quadratic formula to find these (if they exist!).

59 | P a g e
Mathematics
Grade Nine (9)

Domain and Range

As with any function, the domain of a quadratic function f ( x ) is the set of x -values
for which the function is defined, and the range is the set of all the output values
(values of f ).
Quadratic functions generally have the whole real line as their domain: any x is a
legitimate input. The range is restricted to those points greater than or equal to
the y -coordinate of the vertex (or less than or equal to, depending on whether the
parabola opens up or down).

60 | P a g e
Mathematics
Grade Nine (9)
QUIZ
A. Determine the domain and range of each quadratic function.

1. f(x) = 2x 2 + 3x + 5
2. f(x) = x 2 + 3x + 5
3. f(x) = x 2 + 4x – 5
4. f(x) = -3x 2 + 5x + 6
5. f(x) = -4x 2 + 4x + 8
6. f(x) = -x 2 + 12x + 8
7. f(x) = x 2 + 4x + 20
8. f(x) = 5x 2 – 6x + 6
9. fx) = -6x 2 + 3x + 5
10. f(x) = 4x 2 + 3x + 6
B. Determine the vertex and tell whether it opens upward or downward.
1. f(x) = 4x 2 + 2x + 4
2. f(x) = -x 2 + 7x + 12
3. f(x) = x 2 +6x – 9
4. f(x) = -4x 2 + 2x + 12
5. f(x) = -4x 2 + 5x + 12
6. f(x) = -x 2 + 6x + 12
7. f(x) = -x 2 + 16x + 28
8. f(x) = 5x 2 – 3x + 12
9. fx) = -12x 2 + 16x + 12
10. f(x) = 4x 2 + 12x + 24

61 | P a g e
Mathematics
Grade Nine (9)
C. Determine the vertex, axis of symmetry, domain, range and y -
intercept of the graph of each function. Tell whether it opens upward or
downward. Then graph the function.

1. y = 4x 2

2. y = 2x 2 + 1

3. y = x 2 + 4x + 4

4. y = x2 + 1

5. y = x 2 + 5x + 6

6. y = 4x 2 – 2x + 8

7. x 2 + 5x – 24 = 0

8. 4x 2 - 9x = 0

9. x 2 + 4x = 9

10. x 2 + 6x + 9 = 0

11. -x 2 + 3x + 12 = 0

12. f(x) = -4x 2 + 8x + 12

13. f(x) = 6x 2 + 6x – 8

14. f(x) = -2x 2 + 8x – 19

15. f(x) = -4x 2 + 8x + 16

62 | P a g e
Mathematics
Grade Nine (9)

LESSON 13
QUADRATIC FUNCTIONS OF THE FORM
F(X) = A (X – H) 2 + K
The vertex form of a quadratic function is given by
f ( x ) = a ( x - h ) + k , where ( h, k ) is the vertex of the parabola.
2

FYI: Different textbooks have different interpretations of the reference


"standard form" of a quadratic function. Some say f ( x ) = ax 2 + bx + c is
"standard form", while others say that f ( x ) = a ( x - h ) 2 + k is "standard
form". To avoid confusion, this site will not refer to either as "standard
form", but will reference f ( x ) = a ( x - h ) 2 + k as "vertex form" and will
reference f(x ) = ax 2 + bx + c by its full statement.
When written in " vertex form ":
• (h, k) is the vertex of the parabola, and x = h is the axis of symmetry.

• the h represents a horizontal shift (how far left, or right, the graph has shifted
from x = 0).
• the k represents a vertical shift (how far up, or down, the graph has shifted
from y = 0).
• notice that the h value is subtracted in this form, and that the k value is added.
If the equation is y = 2( x - 1) 2 + 5, the value of h is 1, and k is 5.
If the equation is y = 3( x + 4) 2 - 6, the value of h is -4, and k is -6.

To Convert from f ( x ) = ax 2 + bx + c Form to Vertex Form:

Method 1: Completing the Square

To convert a quadratic from y = ax 2 + bx + c form to vertex form, y = a ( x - h ) 2 + k ,


you use the process of completing the square. Let's see an example.

63 | P a g e
Mathematics
Grade Nine (9)
Example: Convert y = 2 x 2 - 4 x + 5 into vertex form, and state the vertex.

Equation in y = ax 2 + bx + c form. y = 2x2 - 4x + 5

Since we will be "completing the


square" we will isolate
y - 5 = 2x2 - 4x
the x 2 and x terms ... so move the + 5
to the other side of the equal sign.

We need a leading coefficient of 1 for


y - 5 = 2( x 2 - 2 x )
completing the square ... so factor out
the current leading coefficient of 2.

Get ready to create a perfect square


trinomial. BUT be careful!! In previous
completing the square problems with
a leading coefficient not 1, our
equations were set equal to 0. Now,
we have to deal with an additional
variable, "y" ... so we cannot "get rid of
" the factored 2. When we add a box
to both sides, the box will be
multiplied by 2 on both sides of the
equal sign.

Find the perfect square


trinomial. Take half of the coefficient
of the x -term inside the parentheses,
square it, and place it in the box.

Simplify and convert the right side to


y - 3 = 2( x - 1) 2
a squared expression.

y = 2( x - 1) 2 + 3

64 | P a g e
Mathematics
Grade Nine (9)
Isolate the y -term ... so move the -3 to
the other side of the equal sign.

In some cases, you may need to


y= 2( x - 1) 2 + 3
transform the equation into the
Vertex form of the equation.
"exact" vertex form of
Vertex = ( h, k) = (1, 3)
y = a(x - h) + k,
2 showing a
(The vertex of this graph will be moved
"subtraction" sign in the parentheses
one unit to the right and three units up
before the h term, and the "addition"
from (0,0), the vertex of its parent y =
of the k term. (This was not needed in
x 2 .)
this problem.)

𝑏
When working with the vertex form of a quadratic function, ℎ = − and 𝑘 = 𝑓(ℎ)
2𝑎

The " a " and " b " referenced here refer to f ( x ) = ax 2 + bx + c.

Method 2

y = ax 2 + bx + c form of the equation. y = 2x2 - 4x + 5

Find the vertex, ( h, k ). a = 2 and b = -4


𝑏
ℎ=− and 𝑘 = 𝑓(ℎ) −(−4) 4
2𝑎
ℎ=− 2(2)
=4=1
[ f ( h ) means to plug your answer
for h into the original equation for x .] 𝑘 = 2(1)2 − 4(1) + 5 = 3

Vertex: (1,3

Write the vertex form. y = 2( x - 1) 2 + 3


y = a(x - h)2 + k

65 | P a g e
Mathematics
Grade Nine (9)
To Convert from Vertex Form to y = ax 2 + bx + c Form:
Simply multiply out and combine like terms:
y = 2( x - 1) 2 + 3
y = 2( x 2 - 2 x + 1) + 3
y = 2x2 - 4x + 2 + 3
y = 2x2 - 4x + 5
Minimum and Maximum Values of f(x) = ax 2 + bx + c, x𝝐 𝑹

1. Transform the equation into the form y = a(x – h) 2 + k

2. If a > 0, then the function has a minimum value of k, and it occurs when x = h.
The function has no maximum value. If a < 0, then the function has a maximum
value of k, and it occurs when x = h. The function has no minimum value.

Examples

Find the minimum or maximum value of each quadratic function, and the
value of x that attains this value.

1. y + (x – 1) 2 = -1

2. y = (2x – 6) 2 + 5

3. y = x 2 + 2x – 4

Solutions

We transform each equation into the form y = a(x – h) 2 + k.

1. y + (x – 1) 2 + -1

y = -(x – 1) 2 – 1

Here, a = -1 < 0. The function has the maximum value of -1, and this value is
attained when x = 1.

66 | P a g e
Mathematics
Grade Nine (9)
2. y = (2x – 6) 2 + 5 y= [2(x – 3)] 2 + 5

y = 4(x – 3) 2 + 5

Here, a = 4 > 0. The function has the minimum value of 5, and this value is
attained when x = 3.

3. y = x 2 + 2x – 4 y = (x 2 + 2x + 1) – 4 – 1

y = (x + 1) 2 – 5

Here, it is understood that a = 1> 0. The function has the minimum val ue of -5,
and this value is attained when x = -1

67 | P a g e
Mathematics
Grade Nine (9)
QUIZ

A. Change the following to vertex form and state the vertex.


1. y = x 2 + 3x + 2
2. y = x 2 – 2x + 1
3. y = x 2 – 5x – 45
4. y = 4x 2 – 3x + 8
5. y = x 2 + 10x + 24
6. y = x 2 + 6x + 7 = 0
7. y = 2x 2 – 7x + 3
8. y = 6x 2 – 8x + 3
9. y = x 2 – 4x + 4
10. y = x 2 + 5x + 6
B. Change the following to y = ax 2 + bx + c
1. y = 3(x – 1) 2 + 5
2. y = (x – 2) 2 – 6
3. y = 3(x – 9) 2 + 6
4. y = 4(x – 2) 2 + 3
5. y = 2(x + 3) 2 + 4
6. y = (4x – 5) 2 + 6
7. y = 3(x – 3) 2 – 7
8. y = 2(2x – 1) 2 – 12
9. y = (x – 3) 2 + 21
10. y = 6(x – 5) 2 + 32

68 | P a g e
Mathematics
Grade Nine (9)
C. Find the minimum or maximum value of the quadratic function
defined by the given equation, and also give the value of x that attains
this minimum or maximum value.

1. y = (x + 3) 2

2. y = (2x – 3) 2 – 5

3. y = x 2 – 4x

4. y = (2x + 1)(x – 2)

5. y = 4x – (x + 2) 2

6. y + 3x 2 = 1 + 2x

7. y = -2x 2 + 20x – 53

8. y = 2(x – 2) 2 – 1

9. y = (x – 4) 2 – 5

10. y = 8x 2 – 144x + 653

11. y = (2x – 3) 2

12. y = (3 – 2x) 2 + 4

13. y + 2(x + 2) 2 = 2x

14. y + 2x = (x + 2) 2

15. y = 4(x – 5) 2 + 6

69 | P a g e
Mathematics
Grade Nine (9)
LESSON 14
EFFECTS OF CHANGES IN THE VALUES OF A,
H, AND K ON THE GRAPHS OF QUADRATIC
FUNCTION
Change in the Value of a

The sign of a in the quadratic function f(x) = a(x – h) 2 + k determines


whether the parabola opens upward or downward. Recall that the parabola opens
of a > 0. it opens downward if a < 0.

Consider the graphs of f(x) = x 2 and and g(x) = 2x 2 . Notice that the graph of
g(x) has a narrower opening compared with that of (x).

The graphs are those of the quadratic functions f(x) = x 2 and g(x) = 2x 2

In general, for functions of the form f(x) = a (x – h) 2 + k, as IaI becomes larger


in the value, the opening of the parabola becomes narrower.

70 | P a g e
Mathematics
Grade Nine (9)
Change in the value of h

Consider the graph of g(x) = (x – 2) 2 . it is a parabola that opens upward and


whose vertex is the point (2,0). Comparing the graph of g(x) = (x – 2) 2 to that of
f(x) = x 2 you may notice that the graph of g(x) is two units to the right of the graph
of f(x). Thus, if h > 0, subtracting h from x shifts the graph to the right by h units.

The graphs are those of the quadratic functions f(x) = x 2 and g(x) = (x – 2) 2

Now, the graph of g(x) = (x + 3) 2 is also a parabola that opens upward and
whose vertex is the point (-3, 0). The figure below shows the graphs of f(x) and
g(x). Comparing the graph of g(x) = (x + 3) 2 to that of f(x) = x 2 , note that the graph
of g(x) in three units to the left of the graph of f(x). thus, if h < 0, subtracting h
units from x shifts the graph to left by I h I units.

71 | P a g e
Mathematics
Grade Nine (9)
In general, if h > 0, subtracting h from x shifts the graph to the right by h
units. On the other hand, if h < 0, subtracting from x shifts the graph to the left by
I h I units. These are called horizontal shifts of the graph.

Change in the Value of k

Refer to the graph of f(x) = x 2 above. It is a parabola that opens upward


and whose vertex is at the point (0, 0).Now, consider the graph of g(x) = x 2 = 2. It
is parabola that opens upward and whose vertex is the point (0, 2). The figure
below shows the graphs of f(x) and g(x). Note that the graph of g(x) is two units
above the graph of f(x). Thus, adding 2 to the quadratic function f(x) shifts the
graph upward by 2 units.

The graphs are those of the quadratic functions f(x) = x 2 and g(x) = x 2 + 2

In general, if k > 0. adding k to f(x) shifts the graph upward b k units, while
subtracting k from f(x) shifts the graph downward by k units. These are called
vertical shifts of the graph of f(x).

72 | P a g e
Mathematics
Grade Nine (9)
QUIZ

1. Explain how you can change the quadratic function f(x) = x 2 so that its graph
will shift upward.

2. Explain how you can change a quadratic function f(x) = x 2 so that its graph will
shift to the left.

73 | P a g e
Mathematics
Grade Nine (9)
LESSON 15

WRITING A QUADRATIC GIVEN ITS VERTEX


AND A POINT
To determine a quadratic function, we need at least to know its vertex
and another point of the function.

To write a quadratic function given its vertex (h,k) and another point
(x 1 ,y 1 )

1. Determine whether the parabola opens upward or downward.

2. Substitute the values of the vertex (h,k) in the equation y =a(x – h) 2 + k.

3. Solve for a by substituting the values of (x 1 ,y 1 ) in the equation obtained in step


2.

4. Simplify the equation.

Example:

1. Find the equation of the parab ola whose vertex is (2,3) that passes through the
point (0,0)

Solution

a. The parabola opens downward since the given point (0,0) is below the vertex
(2, 3)

b. Substitute 2 for h and 3 for k in the equation y =a(x – h) 2 + k

y =a(x – h) 2 + k

y =a(x – 2) 2 + 3

74 | P a g e
Mathematics
Grade Nine (9)

c. Solve for a by substituting 0 for x and 0 for y in the equation derived instep.

0 =a(0 – 2) 2 + 3

0 = 4a + 3

a = -3/4

d. Simplify the equation by substituting -3/4 for a in the equation

y =a(x – h) 2 + k

y =-3/4(x – 2) 2 + 3

3
y = - 4 x 2 + 3x – 3 + 3

𝟑
y = - 𝟒 x 2 + 3x

75 | P a g e
Mathematics
Grade Nine (9)

QUIZ

Write the quadratic function that passes through the given vertex and
point.

1. V(0,-1) ; (1, 0)

2. V(2,-2) ; (4, 0)

3. V(-2,1) ; (1, 0)

4. V(0,3) ; (1, 0)

5. V(2,-1) ; (1, 1)

6. V(3,-1) ; (5, 0)

7. V(1,-2) ; (2, 1)

8. V(-1,-4) ; (-2, -3)

9. V(2,-4) ; (0, 0)

3 1
10. V( 2,− 2) ; (3, -5)

11. V(2,-1) ; (2, 0)

12 V(2,-2) ; (4, 1)

13. V(-2,-5) ; (-4, -4)

14. V(6,-4) ; (2, 1)

2 1
15. V( ,− ) ; (2, -1)
3 2

76 | P a g e
Mathematics
Grade Nine (9)
LESSON 16

WRITING QUADRATIC FUNCTIONS GIVEN


THREE INTERCEPTS OR THREE POINTS
To write a quadratic function given three intercepts:

1. Write three equations by substituting the data points in the equation y = ax 2


+bx + c.

2. Simplify the three equations.

3. Substitute the value determined for c into the other two equations.

4. Solve the resulting system of two equations and two unknowns.

5. Substitute the values of a, b, and c in the equation y = ax 2 +bx + c.

Example 1:

Write the equation function whose graph passes thr ough (0,3),(3,0) and (-1,0)

a. Determine the equation

Eq. y = ax 2 +bx + c.

Eq. 1 3 = a(0) 2 +b(0) + c Substitute (0, 3)

Eq. 2 0 = a(3) 2 +b(3) + c Substitute (3, 0)

Eq. 3 0 = a(-1) 2 +b(-1) + c Substitute (-1, 0)

77 | P a g e
Mathematics
Grade Nine (9)
b. Simplify each equation.

Eq. 1 3= c

Eq. 2 0 = 9a +3b + c

Eq. 3 0 = -a 2 –b + c

c. Substitute 3 for c in the simplified equations 2and 3.

Eq. 2 0 = 9a +3b + c

Eq. 3 0 = -a 2 –b + c

d. Solve the remaining system of linear equations. Use the elimination


method.

Eq. 2 9a +3b = -3

Eq. 3 3a – 3b = -9 Multiply (Eq. 3) by 3

Add the equations 2 and 3: 12a = -12

a = -1

Eq. 2 9(-1) + 3b = -3

b=2

Therefore a = 1, b = 2 and c = 3. Substitute th ese values in the quadratic


function y = ax 2 +bx + c .

y = (-1) 2 + 2x + 3

y = -x 2 + 2x + 3

78 | P a g e
Mathematics
Grade Nine (9)
To write a quadratic function given the three data points.
1. Write the three equations by substituting the data points in the equation
y = ax 2 + bx + c.
2. Simplify the three equations.
3. Determine two system containing two equations each
4. Eliminate the same variable in each system.
5. Solve the new system.
6. Substitute the values of a, b, and c in the equation y = ax 2 + bx + c.
Example Write the quadratic function whose graph passes through (2, -15), (-2,
-7), and (-4, 9)
Solution
a. Determine the three equations.
Eq. y = ax 2 + bx + c
(Eq. 1) -15 = a(2) 2 +b(2) +c Substitute (2, -15)
(Eq. 2) -7 = a(-2) 2 +b(-2) + c Substitute (-2, -7)
(Eq.3) 9 = a(-4) 2 +b(-4) + c Substitute (-4, 9)
b. Simplify each Equation
(Eq. 1) -15 = 4a +2b +c
(Eq. 2) -7 = 4a – 2b + c
(Eq.3) 9 = 16a – 4b + c
c. Determine two system containing two equation each.
First System
(Eq. 1) -15 = 4a + 2b + c
(Eq. 2) -7 = 4a – 2b + c
Add the equations 1 and 2:
(Eq. 4) -22 + 8a + 2c
Second System
(Eq. 1) -15 = 4a + 2b + c

79 | P a g e
Mathematics
Grade Nine (9)
(Eq.3) 9 = 16a – 4b + c
Multiply (1) by 2 and then add.
(Eq.1) -30 = 8a + 4b + 2c
(Eq. 3) 9 = 16a – 4b + c
Eq. 5 -21 = 24a + 3c
d. New System
Eq. 4 -22 +8a + 2c
Eq. 5 -21 = 24a + 3c
e. Solve the new system. Multiply (Eq.4) by -3
Eq.4 66 = -24a – 6c
Eq. 5 -21 = 24 a + 3c
45 = -3c
C = -15
Eq. 4 -22 = 8a + 2(-15)
-22= 8a – 30
a=1
Eq. 1 -15 = 4(1) + 2b – 15
-15 = 4 + 2b – 15
b = -2

Therefore, a = 1, b = -2 and c = -15. Substitute these values in the quadratic


function y = ax 2 + bx + c

y = (1) 2 + (-2)X +-15

y = x 2 – 2x – 15

80 | P a g e
Mathematics
Grade Nine (9)
QUIZ

A. Write the quadratic function that passes through the given x -


intercepts and y-intercepts.
1. (5, 0), (-1, 0) and (0, -1)
2. (1.5, 0), (-2, 0) and (0, 2)
3. (2/3, 0), (-1, 0) and (0, -2)
4. (3, 0), (-2, 0) and (0, -2)
5. (2, 0), (-4, 0) and (0, -4)
6. (3, 0), (-3, 0) and (0, 3)
7. (2, 0), (-4, 0) and (0, -8)
8. (1/2, 0), (9/2, 0) and (0, -2)
9. (-2/3, 0), (1/3, 0) and (0, 1)
10. (2, 0), (-2, 0) and (0, 2/3)

B. Write the quadratic function that passes through the given points.
1. (2, 16), (3, 25) and (1, 9)
2. (2, 6), (-3, -21) and (1, -5)
3. (-2, 5), (2, -3) and (1, 2)
4. (-1, -3), (-2, -5) and (0, -11)
5. (2, -3), (-2, -15) and (6, -7)
6. (1, 2), (3, 4), and (4, 6)
7. (1, 4), (2, 9), and (3, 16)
8. (-1, 4), (-2, 4), and (-3, 6)
9. (2, 12), (3, 28), and (-2, -12)
10. (-1, -2), (-2, 4) and (1, -8)

81 | P a g e
Mathematics
Grade Nine (9)

LESSON 17
APPLICATIONS OF QUADRATIC FUNCTIONS
In this section we’re going to go back and revisit some of the applications
that we saw in the Linear Applications section and see some examples that will
require us to solve a quadratic equation to get the answer.

Note that the solutions in these cases will almost always require the
quadratic formula so expect to use it and don’t get excited about it. Also, we are
going to assume that you can do the quadratic formula work and so we won’t be
showing that work. We will give the results of the quadratic formula; we just won’t
be showing the work.

Also, as we will see, we will need to get decimal answer to these and so as a
general rule here we will round all answers to 4 decimal places.

Example 1: We are going to fence in a rectangular field and we know that for
some reason we want the field to have an enclosed area of 75 ft 2 . We also know
that we want the width of the field to be 3 feet longer than the length of the
field. What are the dimensions of the field?

Solution

So, we’ll let x be the length of the field and so we know that will be the
width of the field. Now, we also know that area of a rectangle is length times
width and so we know that,

82 | P a g e
Mathematics
Grade Nine (9)
Now, this is a quadratic equation so let’s first write it in standard form.

Using the quadratic formula gives,

Now, at this point, we’ve got to deal with the fact that there are two solutions
here and we only want a single answer. So, let’s convert to decimals and see
what the solutions actually are.

So, we have one positive and one negative. From the stand point of needing
the dimensions of a field the negative solution doesn’t make any sense so we will
ignore it.

Therefore, the length of the field is 7.2892 feet. The width is 3 feet longer
than this and so is 10.2892 feet.

Notice that the width is almost the second solution to the quadratic
equation. The only difference is the minus sign. Do NOT expect this to always
happen. In this case this is more of a function of the problem. For a more
complicated set up this will NOT happen.

Now, from a physical standpoint we can see that we should expect to NOT get
complex solutions to these problems. Upon solving the quadratic equation, we
should get either two real distinct solutions or a double root. Also, as the previous

83 | P a g e
Mathematics
Grade Nine (9)
example has shown, when we get two real distinct solutions we will be able to
eliminate one of them for physical reasons.

Let’s work another example or two.

Example 2 Two cars start out at the same point. One car starts out driving
north at 25 mph. Two hours later the second car starts driving east at 20
mph. How long after the first car starts traveling does it take for the two cars
to be 300 miles apart?

Solution

We’ll start off by letting t be the amount of time that the first car, let’s call it
car A, travels. Since the second car, let’s call that car B, starts out two hours
later then we know that it will travel for hours.

Now, we know that the distance traveled by an object (or car since that’s what
we’re dealing with here) is its speed times time traveled. So we have the
following distances traveled for each car.

At this point a quick sketch of the situation is probably in order so we can see
just what is going on. In the sketch we will assume that the two cars have
traveled long enough so that they are 300 miles apart.

84 | P a g e
Mathematics
Grade Nine (9)
So, we have a right triangle here. That means that we can use the Pythagorean
Theorem to say,

This is a quadratic equation, but it is going to need some fairly heavy


simplification before we can solve it so let’s do that.

Now, the coefficients here are quite large, but that is just something that will
happen fairly often with these problems so don’t worry about that. Using the
quadratic formula (and simplifying that answer) gives,

Again, we have two solutions and we’re going to need to determine which
one is the correct one, so let’s convert them to decimals.

As with the previous example the negative answer just doesn’t make any
sense. So, it looks like the car A traveled for 10.09998 hours when they were
finally 300 miles apart.

85 | P a g e
Mathematics
Grade Nine (9)
Also, even though the problem didn’t ask for it, the second car will have
traveled for 8.09998 hours before they are 300 miles a part. Notice as well that
this is NOT the second solution without the negative this time, unlike the first
example.

Example 3: An office has two envelope stuffing machines. Working together


they can stuff a batch of envelopes in 2 hours. Working separately, it will take
the second machine 1 hour longer than the first machine to stuff a batch of
envelopes. How long would it take each machine to stuff a batch of envelopes
by themselves?

Solution

Let t be the amount of time it takes the first machine (Machine A) to stuff a
batch of envelopes by itself. That means that it will take the second machine
(Machine B) hours to stuff a batch of envelopes by itself.

The word equation for this problem is then,

We know the time spent working together (2 hours) so we need to work rates of
each machine. Here are those computations.

86 | P a g e
Mathematics
Grade Nine (9)
Note that it’s okay that the work rates contain t . In fact they will need to so we
can solve for it! Plugging into the word equation gives,

So, to solve we’ll first need to clear denominators and get the equation in
standard form.

Using the quadratic formula gives,

Converting to decimals gives,

Again, the negative doesn’t make any sense and so Machine A will work for
3.5616 hours to stuff a batch of envelopes by itself. Machine B will need 4.5616
hours to stuff a batch of envelopes by itself. Again, unlike the first example, note
that the time for Machine B was NOT the second solution from the quadratic
without the minus sign.

87 | P a g e
Mathematics
Grade Nine (9)
QUIZ

Solve the following word problems.

1. The length of a rectangle is 6 cm more than its width. Find its lengt h and width
when its area is 40 cm 2 .

2. Rizen is five years older than JB. Find their present age if the product of Rizen’s
age four years from now and JB’s five years ago is 51.

3. Train A is 5 kph faster than train B and covers t he distance of 300 km in 2hrs
less time than train B. Find the rate of the two trains.

4. The hypotenuse of a right triangle is 17 cm. H ow long are the legs of one is 7cm
longer than the other?

5. The product of two consecutive integers is 600. Find th e integers.

6. The product of two consecutive positive integers is 156. Find the numbers.

7. The product of two consecutive odd numbers is 255. Find the numbers.

8. The numerator of a fraction is 2 less than its denominator. When both numerator
3
and denominator are increased by three, the fraction is increased by 20
. Find the
original fraction.

9. The altitude of a triangle is 6 cm greater than its base. If its area is 108 cm 2 ,
find its base.

10. The length of a rectangle is 8 cm more than the width and its area is 172 cm 2 .
Find

a. the width of the rectangle

b. the length of the diagonal of the rectangle

88 | P a g e
Mathematics
Grade Nine (9)
LESSON 18
RECALLING RATIONAL EXPRESSION AND
THEIR DOMAINS

What is the difference between a real number and a rational number? Are
all real numbers rational numbers, or is it the other way around? Are -2, 12.5, √2 ,
1
, , and 𝜋 are rational numbers?
2

A real number is a value that represents a point along a number line. On


the other hand, a rational number is a real number that can be expressed as a
quotient of two integers. In its decimal representation, a rational number is
terminating or repeating. Real numbers are composed of the sets of rational and
irrational numbers. It means that all rational numbers are real and not the other
way around.

The following are examples of rational numbers .


−4 −10 −24
a. -2, since it can be expressed as , 𝑜𝑟 .
2 5 12

25 50 100
b. 12.5, since it can be expressed as 2
, 4
, 𝑜𝑟 8
.

1
c. , by definition
2

d. , since it can be expressed as 1/3

The following are examples of irrational numbers.

a. √𝑎 , since 2 is not a perfect square and it’s a val ue is 1.414213562…, which is a


nonrepeating and nonterminating decimal.

b. 𝜋, since its value is 3.141592654…, which is also a n onrepeating and


nonterminating decimal.

89 | P a g e
Mathematics
Grade Nine (9)
We extend the concept of rational numbers to involve variables, and we call it a
rational expression.

A rational expression is a ratio of two polynomial expressions.


𝑃
A rational expression can be written in the form , where P and Q are
𝑄
polynomial expressions and Q ≠ 0.

The following are examples of rational expressions


2
a.
3−4𝑥

(3−𝑎𝑏)2
b. (5+6𝑎𝑏)3

2𝑥+3𝑦−4𝑧
c.
4𝑥−7𝑦+12𝑧

1
+𝑛
d. 𝑚
1
𝑚−
𝑛

A rational expression is defined for all real numbers for which the
denominator is not equal to zero. The set of real numbers that make the expression
defined is the domain of the expression. Two rational expression are equivalent if
they have the same lowest term.

1. Determine whether the two given rational expression are equivalent.


4𝑥 5𝑥
a. and
16𝑥−4 20𝑥−5

𝑥+1 1
b. and
𝑥2 − 1 𝑥+1

Solution: To check whether the two rational expressions are equivalent or not, we
must first express each rational expr ession in lowest terms.
4𝑥 4𝑥 𝑥
a. = =
16𝑥−4 4(4𝑥−1) 4𝑥−1

𝑥
is in lowest term
4𝑥−1

5𝑥 5𝑥 𝑥
= 5(4𝑥−1) =
20𝑥−5 4𝑥−1

90 | P a g e
Mathematics
Grade Nine (9)
𝑥
is in lowest term.
4𝑥−1

Therefore, the two rational expressions are equivalent.


𝑥+1 𝑥+1 1
b. = = ,x≠0
𝑥2 − 1 (𝑥+1)(𝑥−1) 𝑥−1

1
The lowest term is 𝑥−1

1
It is already in the lowest term since the highest common factor between the
𝑥+1
numerator and denominator is 1.

Therefore, the two rational expressions are not equivalent.

2. Determine the domain of the rational expression.


4𝑥−5
a. 3𝑥−12

3
b. 𝑥2 + 4

5−3𝑥
c. 𝑥 2 − 5𝑥−24

Solutions
4𝑥−5
a. 3𝑥−12

3x – 12 = 0

3x = 12

x=4

The domain is {𝑥 ∈ 𝑅|𝑥 ≠ 4}


3
b. 𝑥2 + 4

x2 + 4 = 0

Since x 2 ≥ 0 for any value of x, x 2 + 4 ≥ 4

91 | P a g e
Mathematics
Grade Nine (9)
Therefore, the rational expression is defined for any value of x, and so the
domain 𝑅.
5−3𝑥
c. 𝑥 2 − 5𝑥−24

x 2 – 5x – 24 = 0

(x – 8)(x + 3) = 0 x –8=0 or x+3=0

x = 8 0r x = -3

Therefore, the rational expression is not define d when x = 8 and x = -3. Here the
domain is {𝑥 ∈ 𝑅|𝑥 ≠ 8 𝑎𝑛𝑑 𝑥 ≠ −3}.

92 | P a g e
Mathematics
Grade Nine (9)
QUIZ

A. Find the domain of the rational expression.

4−𝑚 𝑚3
1. 6.
𝑚 𝑚2 + 3𝑚+2

𝑚−2 7
2. 7.
2−𝑚 𝑚−2

𝑥 𝑎−𝑏
3. 𝑥+3
8. 𝑎2 − 2𝑎𝑏+ 𝑏 2

𝑥 𝑥+3
4. 9.
𝑥 2 − 𝑥−2 𝑥 2 − 8𝑥+16

2𝑎 2𝑥
5. 10.
𝑎−5 𝑥 2 − 16

B. For what value(s) of x will be the following rational expressions be


undefined.
𝑥 𝑥+3
1. 6.
𝑥−4 4𝑥 2 + 28𝑥+49

𝑥+5 𝑥+5
2. 7.
(𝑥−3)(𝑥−5) 25𝑥 2 + 40𝑥+16

3𝑥 𝑎−𝑏
3. 𝑥 2 + 6𝑥+9
8. 4𝑥 2 + 16𝑥+15

5 2𝑥−3
4. 9.
(2𝑥−2)(𝑥−3) 25𝑎2 − 4

𝑎+3 2𝑥−5
5. 10.
(𝑥 2 − 25) 𝑥 2 + 7𝑥+12

C. Which of the following rational expressions is not defined when x = 3?


2𝑥 𝑥+5
1. 6.
𝑥−4 𝑥 2 + 5𝑥+6

𝑥+7 2𝑥−6
2. 7.
(𝑥−3)(𝑥−3) 𝑥 2 + 9𝑥+20

93 | P a g e
Mathematics
Grade Nine (9)
𝑥+4 4𝑎
3. 8.
𝑥 2 + 6𝑥+9 4𝑥 2 + 16𝑥+12

3𝑥 3𝑥−5
4. (𝑥−2)(𝑥−3)
9. 2𝑥−6

𝑥+5 3𝑥−8
5. 10.
(𝑥 2 − 4) 𝑥 2 + 9𝑥+20

D. Determine whether the two given rational expressions are equivalent


or not.
𝑥 4𝑥
1. 𝑎𝑛𝑑
𝑥−4 4𝑥−16

𝑥+3 2𝑥+6
6. 𝑥 2 + 5𝑥+6
𝑎𝑛𝑑 2𝑥 2 + 10𝑥+12

2𝑥−5 1
2. 𝑎𝑛𝑑
2𝑥 2 −7𝑥+5 𝑥−1

2𝑥−7 1
7. 𝑎𝑛𝑑
4𝑥 2 − 49 2𝑥−7

𝑥+3 𝑥+3
3. 𝑎𝑛𝑑
𝑥 2 + 6𝑥+9 𝑥 2 + 4𝑥+9

𝑥+3 3𝑥+9
8. 𝑎𝑛𝑑
4𝑥+12 12𝑥+144

𝑥−5 3𝑥−2
4. 𝑎𝑛𝑑
(𝑥−2)(𝑥−5) 3𝑥 2 −17𝑥+10

2𝑥−7 2𝑥−7
9. 𝑎𝑛𝑑
3𝑥 2 − 5𝑥+12 6𝑥 2 − 10𝑥+24

𝑥−2 3𝑥−6
5. and
(𝑥 2 − 16) 4𝑥 2 − 10𝑥+24

2𝑥−5 3𝑥−6
10. 𝑎𝑛𝑑
2𝑥 2 + 9𝑥+20 6𝑥 2 + 10𝑥+12

94 | P a g e
Mathematics
Grade Nine (9)
LESSON 19
RECALLING ADDITION AND SUBTRACTION
OF RATIONAL EXPRESSION

In adding and subtracting fractions. It is very important to determine the


least common denominator (LCD), especially if two or more fractions are not
similar. Can you determine the LCD of the following set of fractions?
1 1 1
a. + +
2 3 6

9 3 7
b. − −
10 16 20

Two or more fractions are similar if they have th e same denominators. In this
case, to add or subtract similar fractions, simply add or subtract their numerators
and keep the common denominator, then reduce the answer to the lowest term, if
possible.

If two or more fractions are dissimilar, we have to de termine first their least
common denominator (LCD) and find their equivalent fraction before adding and
subtracting their numerators.
1 1 1
a. + + , the LCD is 6
2 3 6

To find the answer, rewrite each fraction in terms of the LCD.


1 3
=
2 6

1 2
=
3 6

1 1
=
6 6

1 1 1 3 2 1 6
+ + = + + = or 1
2 3 6 6 6 6 6

95 | P a g e
Mathematics
Grade Nine (9)
9 3 7
b. − − , the LCD is 80
10 16 20

To find the answer, rewrite each fraction in terms of the LCD.


9 72
=
10 80
3 15
=
16 80
7 28
=
20 80
9 3 7 72 15 28 29
10
− 16
− 20
= 80
− 80
- 80
= 80

In the same manner, we can add or subtract rational expression using the same
process.
To add or subtract rational expressions with the same denominators, add or
subtract the numerators and keep the common denominator for the result.
𝑃 𝑅 𝑃 ±𝑅
Hence, ± = 𝑤ℎ𝑒𝑟𝑒 𝑄 ≠ 0.
𝑄 𝑄 𝑄

To add or subtract rational expressions with different denominators, find the


least common denominator (LCD), rewrite each rational expression in terms of
the LCD, and add or subtract as in similar rational expressions

Perform the indicated operations and reduce the answers to lowest terms.
12𝑥−3 5𝑥 17
1. − +
𝑥+2 𝑥+2 𝑥+2

Solution: Since the fractions are all similar, we can already perform the
operations in the numerator
12𝑥−3 5𝑥 17 (12 𝑥−3)− 5𝑥+17
− + =
𝑥+2 𝑥+2 𝑥+2 𝑥+2
7𝑥+14
= 𝑥+2
7(𝑥+2)
=
𝑥+2
=7

96 | P a g e
Mathematics
Grade Nine (9)
4𝑥+2 7−10𝑥
2. +
5𝑥 5𝑥

Solution:
4𝑥+2 7−10𝑥 (4𝑥+2) + (7−10𝑥)
+ =
5𝑥 5𝑥 5𝑥
4𝑥+2+7−10𝑥
= 5𝑥
9−6𝑥
= 5𝑥
15 9 7
3. + +
2𝑥 4𝑥 𝑥2

Solution:
The LCD is 4x 2
Rewriting each expression in terms of the LCD, we have
15 30𝑥
=
2𝑥 4𝑥 2
9 9𝑥
=
4𝑥 4𝑥 2
7 28
=
𝑥2 4𝑥 2
15 9 7 30𝑥+9𝑥+28
+ + =
2𝑥 4𝑥 𝑥2 4𝑥 2
39𝑥+28
= 4𝑥 2
7 8𝑥
4. −
𝑥−3 𝑥2− 9

Solution
Since (x – 3) is a factor of (x 2 – 9), the LCD is (x 2 – 9) or (x – 3)(x + 3)
7 7(𝑥 + 3)
=
𝑥−3 (𝑥 − 3)(𝑥 + 3)
8𝑥 8𝑥
=
𝑥2 − 9 (𝑥 − 3)(𝑥 + 3)

97 | P a g e
Mathematics
Grade Nine (9)
7 8𝑥 7(𝑥+3)− 8𝑥
− =
𝑥−3 𝑥2 − 9 (𝑥−3)(𝑥+3)

7𝑥+21−8𝑥
= (𝑥−3)(𝑥+3)

21−𝑥
= (𝑥 −3)(𝑥+3)

−4 16
5. +
𝑥 2 + 𝑥+8 𝑥 2 + 𝑥−12

Solution

The factors of x 2 + 6x + 8 are (x + 2)(x + 4) and the factors of x 2 + x – 12 are

(x + 4)(x – 3). Hence, the LCD is (x + 2)(x + 4)(x – 3)

−4 −4 −4(𝑥 − 3)
= =
𝑥2 + 𝑥+8 (𝑥 + 2)(𝑥 + 4) (𝑥 + 2)(𝑥 + 4)(𝑥 − 3)

16 16 16(𝑥 + 2)
= =
𝑥 2 + 𝑥 − 12 (𝑥 + 4)(𝑥 − 3) (𝑥 + 2)(𝑥 + 4)(𝑥 − 3)

−4 16 −4(𝑥 + 3) + 16(𝑥 + 2)
+ =
𝑥 2 + 𝑥 + 8 𝑥 2 + 𝑥 − 12 (𝑥 + 2)(𝑥 + 4)(𝑥 − 3)

−4𝑥+12+16𝑥+32
= (𝑥+2)(𝑥+4)(𝑥−3)

12𝑥+44
= (𝑥+2)(𝑥+4)(𝑥−3)

98 | P a g e
Mathematics
Grade Nine (9)
QUIZ

A. Perform the indicated operations and reduce answers to lowest term.


1 2 1
1. + +
5 5 5

3 2 1
2. − +
9 9 9

4 3 6 1
3. + − +
12 12 12 12

4 2 6 9
4. + + −
5 3 8 10

6 2 6 1 1
5. − + + −
7 7 8 4 2

3 7 6 5
6. + + +
6 9 8 8

4 3 9 2 3
7. + + − +
5 7 10 5 8

1 2 6 3 9
8. + − − +
3 5 8 12 10

2 6 7 5
9. − + −
4 10 12 8

6 3 2 9
10. 10
+ 5
− 7
+ 10

B. Determine the LCD of the following se of rational expression.


𝑎 2 3
1. , ,
4 4𝑎 12𝑥

𝑥 2𝑥
2. ,
𝑥+3 𝑥 2 + 6𝑥+9

4 5 𝑥+4
3. , ,
𝑥 2 2𝑥+4 𝑥 2 + 4𝑥+4

5 3 3 2𝑥
4. , , ,
𝑥+3 2𝑥−3 2𝑥+6 2𝑥 2 + 3𝑥−9

𝑥 2 2
5. , ,
9𝑥 𝑥 2 − 9𝑥 𝑥+3

99 | P a g e
Mathematics
Grade Nine (9)
2𝑥 2𝑥 𝑥+5
6. , ,
𝑥 2 + 8𝑥+16 3𝑥+12 𝑥 2 + 9𝑥+20

3 𝑥+6 2+3𝑥
7. , ,
𝑥 𝑥 2 + 𝑥 𝑥 2 −2𝑥−3

𝑥 2𝑥 2𝑥+5 5
8. , , ,
𝑥+ 3 𝑥−5 𝑥 2 − 2𝑥−15 3𝑥−15

4−𝑚 𝑚 2
9. , ,
𝑚−4 ,𝑚2 − 4𝑚 3𝑚

𝑥 𝑥+4
10. ,
𝑥 2 + 3𝑥+2 3𝑥 2 + 17𝑥 +10

C. Express the following rational expressions into a single fraction in


lowest terms.
4 5𝑦𝑧 7𝑥𝑧 9𝑥𝑦
1. − + −
3𝑥𝑦𝑧 6𝑥 8𝑦 10𝑧

𝑎−4 7−3𝑎 14𝑎


2. − +
𝑎−2 𝑎−2 𝑎−2

𝑐−2 𝑐−3 𝑐−6


3. + −
𝑐−3 𝑐+3 𝑐2− 9

𝑑+10 𝑑−10
4. +
𝑑−10 𝑑2 − 100

𝑒 𝑒 𝑒
5. 𝑒−2
+ 𝑒2− 4
+ 𝑒3− 8

10 20 30 40
6. + + +
𝑐2𝑑 𝑐𝑑2 𝑐𝑑 𝑐 2 𝑑2

ℎ+2 ℎ+3 ℎ+4 ℎ+5


7. + + −
2ℎ−4 3ℎ−6 4ℎ−8 5ℎ−10

4𝑎 16𝑏 12𝑎𝑏
8. + -
𝑎−𝑏 𝑎+𝑏 𝑎2 − 𝑏2

𝑤+1 𝑤−1 𝑤2− 1


9. − −
𝑤−1 𝑤+1 𝑤 2 + 2𝑤 + 1

2 4 1
10. − +
𝑎−2 𝑎+2 𝑎2 − 4

100 | P a g e
Mathematics
Grade Nine (9)
LESSON 20
EQUATIONS INVOLVING RATIONAL
EXPRESSIONS
Formulating an equation is a useful way to solve mathematical problems. In
this case, the Additional Property of Equality (APE). Multiplication Property of
Equality (MPE), and Transitive Property are essential in finding the solution(s) of
a given problem. How will you use these properties already involve rational
equations?

An equation that involves one or more rational expressions is called rational


equation . The number obtained from solving a rational equation may not always
satisfy the original equation because of the restriction in the denominator. In such
cases, we call such number an extraneous solution . It is always important to check
whether the number is a true solution or extraneous.
𝑥 𝑥 𝑥 2 − 𝑥+6
Consider the equation + =
𝑥+2 𝑥−2 𝑥2− 4

Since x 2 – 4 = (x + 2)(x – 2), the LCD of the fractions is (x + 2)(x – 2). We note that
x must not be equal to -2 and 2. Now, solving for x, we have
𝑥 𝑥 𝑥 2 − 𝑥+6
(x + 2)(x – 2)[𝑥+2 + ]=[ ](x + 2)(x – 2) Multiplication Property of Equality
𝑥−2 𝑥2 − 4

x(x – 2) + x(x + 2) = x 2 – x + 6 Simplify


x 2 – 2x + x 2 + 2x = x 2 – x + 6 Distributive Property
2x 2 = x 2 – x + 6 Combine similar terms
2x 2 - x 2 + x – 6 = 0 Addition Property of Equality
x2 + x – 6 = 0 Combine similar terms
(x – 2)(x + 3) = 0 Factor
x–2=0 or x+3 =0 Zero-Factor Theorem
x=2 or x = -3

101 | P a g e
Mathematics
Grade Nine (9)
Check :

To verify whether -3 and 2 are the solutions of the original equation, we use
substitution method.

If x = 2

𝑥 𝑥 𝑥2 − 𝑥 + 6
+ =
𝑥+2 𝑥−2 𝑥2 − 4
2 2 22 − 2 + 6
+ =
2+2 2−2 22 − 4
2 2 8
+ ≠
4 0 0
Two fractions are not defined since their denominators are bot h zero.
Therefore, 2 is extraneous solution .

If x = -3

𝑥 𝑥 𝑥2 − 𝑥 + 6
+ =
𝑥+2 𝑥−2 𝑥2 − 4

−3 −3 (−3)2 − (−30 + 6
+ =
−3 + 2 −3 − 2 (−3)2 − 4

−3 −3 9+3+6
= =
−1 −5 9−4
3 18
3+ =
5 5
18 18
=
5 5
Therefore, -3 is a solution.

102 | P a g e
Mathematics
Grade Nine (9)
Solve the following problems.
1. Find the real solution(s) of the following rational equations.
2+𝑎 5𝑎
a. − =1
3 4
9 7
b. − =0
2𝑏−1 2𝑏+1

Solutions:
2+𝑎 5𝑎
a. − =1
3 4
2+𝑎 5𝑎
(12)[ − ] = 1(12) Multiplication Property of Equality
3 4

(4)(2 + a) – 3(5a) = 12 Simplify


8 + 4a – 15a = 12 Distributive Property
-11a = 12 – 8 Combine similar terms
-11a = 4
4
a =- 11
4
Check: If a = - 11
2 + 𝑎 5𝑎
− =1
3 4
4 4
2 + (− ) 5(− )
11 11
− =1
3 4
18 20

11 11
− =1
3 4
18 10
+ =1
33 22
36 + 30
=1
66
66
=1
66
1=1
4
Therefore, is a solution
11

103 | P a g e
Mathematics
Grade Nine (9)
9 7
b. − =0
2𝑏−1 2𝑏+1
9 7
(2b – 1)(2b + 1) [2𝑏−1 − ] = 0(2b – 1)(2b + 1)
2𝑏+1

9(2b + 1) – 7(2b – 1) = 0
18b + 9 – 14n + 7 = 0
4b = -16
b = -4
Check :
If b = -4, then
9 7
− =0
2𝑏 − 1 2𝑏 + 1
9 7
− =0
2(−4) − 1 2(−4) + 1
9 7
− =0
−8 − 1 −8 + 1
9 7
− =0
−9 −7
- 1 – (-1) = 0
0=0
Therefore, -4 is a solution.

2. Ben can paint the entire room in 6 hours. John can finis h the same task in 4
hours. If they work together, how long will it take them to finish the task?

Solution:

Let x be the number of hours Ben and Jo hn will take to finish the task
working together.

Since there is only one task, we can express the given as ratios:
1
ratio of the task to the number of hours that Ben will spend
6

104 | P a g e
Mathematics
Grade Nine (9)
1
ratio of the task to the number of hours that John will spend
4

1
ratio of the task to the number of hours that Ben an d John will spend if they
𝑥
work together.

The equation now is

1 1 1
+ =
6 4 𝑥
Solving for x, we have
1 1 1
(12x)[6 + ] = (12x) [ ]
4 𝑥

2x(1) + 3x(1) = 12

2x + 3x = 12

5x =12
12
x= or 2.4 hours or 2 hours and 24 minutes
5

Therefore, Ben and John can finish the painting task in 2 hours and 24 minutes
if they will work together.

If two persons are working on the same job, and the first would take a hours
to complete the job alone and the second would take b hours to complete the job
1 1 1
alone, then the equation + = can be used to find x, the time it will take to
𝑎 𝑏 𝑥
complete the job.

105 | P a g e
Mathematics
Grade Nine (9)
QUIZ

A. Solve each rational equations and determine whether the solution is


real or extraneous.
2𝑥 3𝑥 1 𝑥 2 + 8𝑥+15
1. + = 10. =9
4 6 12 𝑥+3

1 7 9 1 1 1
2. − = 11. + =
2𝑥 4𝑥 8𝑥 𝑥+1 2 𝑥

𝑥+1 𝑥−1 𝑥 2𝑥 𝑥
3. + = 12. + =1
𝑥−2 𝑥+2 𝑥2 − 4 𝑥2 − 4 𝑥+3

𝑥 𝑥 1 4𝑎 6𝑎 12𝑎 16𝑎
4. 𝑥+1
+ 2𝑥+2
− −𝑥−1
=1 13. 12
+ 18
= 24
− 32

𝑥 𝑥−4 9 1 4
5. = 14. + =
𝑥−4 𝑥 2 − 16 2( 𝑥+2) 2𝑥 𝑥+1

1 2 4 3𝑥 4
6. + − =8 15. − =0
𝑥 3𝑥 5𝑥 4 3𝑥

𝑥+1 𝑥 2 + 2𝑥+1
7. =
𝑥−9 𝑥 2 − 81

(𝑥−5)2 𝑥 2 − 25
8. + = 10
𝑥−5 𝑥+5

2𝑥 3𝑥 4𝑥 6𝑥
9. 5
+ 7
= 9
− 11

B. Solve the following problems.

1. Ann can finish a certain job in 12 days. Mary can fini sh the same job in 15 days.
If Ann and Mary will work together, how long will it take them to finish the job?
1
2. A motorist took 3 hours to drive 160 km. He drove p art of the ways at an
3
average speed of 50 km/h and the rest of the way at 45 km/h. What is the distance
he travelled at 50 km/h?

3. A mother is 21 years older than her new born daughter. How old will the
1
daughter be when her age is that of her mother.
4

106 | P a g e
Mathematics
Grade Nine (9)
LESSON 21

DIRECT VARIATION
When you start studying algebra, you will also study how two (or more)
variables can relate to each other specifically. The cases you’ll study are:

Direct Variation, where both variables either increase or decrease together

Inverse or Indirect Variation, where when one of the variables increases, the
other one decreases

Joint Variation, where more than two variables are related directly

Combined Variation, which involves a combination of direct or joint


variation, and indirect variation

These sound like a lot of fancy math words, but it’s really not too
bad. Here are some examples of direct and inverse variation:

Direct: The number of dollars I make varies directly (or you can say varies
proportionally) with how much I work.

Direct: The length of the side a square varies directly with the perimeter of the
square.

Inverse: The number of people I invite to my bowling party varies inversely with
the number of games they might get to play (or you can say is proportional to
the inverse of).

Inverse: The temperature in my house varies indirectly (same as inversely) with


the amount of time the air conditioning is running.

Inverse: My GPA may vary directly inversely with the number of hours I watch
TV.

107 | P a g e
Mathematics
Grade Nine (9)
Direct or Proportional Variation

When two variables are related directly, the ratio of their values is
always the same. So as one goes up, so does the other, and if one goes down, so
does the other. Think of linear direct variation as a “ y = m x ” line, where the ratio
of y to x is the slope (m). With direct variation, the y -intercept is always 0
(zero); this is how it’s defined.

Direct variation problems are typically written:

→ y = kx where k is the ratio of y to x (which is the same as


the slope or rate).

Some problems will ask for that k value (which is called the constant
of variation or constant of proportionality – it’s like a slope!); others will just
give you 3 out of the 4 values for x and y and you can simply set up a rat io to find
the other value. You’re thinking the k comes from the word “constant” in another
language.

Remember the example of making $10 an hour at the mall ( y = 10 x )? This


is an example of direct variation, since the ratio of how much you make to how
many hours you work is always constant.

We can also set up direct variation problems in a ratio, as long as we have


the same variable in either the top or bottom of the ratio, or on the same
side. This will look like the following. Don’t let this scare you; the subscripts just
refer to the either the first set of variables , or the second .

Direct Variation Word Problem:

So we might have a problem like this:

The value of y varies directly with x , and y = 20 when x =

108 | P a g e
Mathematics
Grade Nine (9)
2. Find y when x = 8. (Note that this may be also be written “ y is proportional
to x , and y = 20 when x = 2. Find y when x = 8.”)

Solution: We can solve this problem in one of two ways, as shown. We do these
methods when we are given any three of the four values for x and y .

Formula Method:

Proportion Method:

109 | P a g e
Mathematics
Grade Nine (9)
It’s really that easy. Can you see why the proportion method can be the
preferred method, unless you are asked to find the k constant in the formula?

Again, if the problem asks for the equation that models this situation, it would
be “y = 10x“.

Direct Variation Word Problem:


The amount of money raised at a school fundraiser is directly proportional
to the number of people who attend. Last year, the amount of money raised for
100 attendees was $2500. How much money will be raised if 1000 people
attend this year?
Solution: Let’s do this problem using both the Formula Method and
the Proportion Method:

110 | P a g e
Mathematics
Grade Nine (9)
Direct Variation Word Problem:

Brady bought an energy efficient washing machine for her new


apartment. If she saves about 10 gallons of water per load, how many gallons
of water will she save if she washes 20 loads of laundry?
Solution:
Let’s do this with the proportion mo del:

111 | P a g e
Mathematics
Grade Nine (9)
QUIZ

A. Find the constant k and the equation of variation if y varies directly


as x.
1. x = 16 and y = 12
2. x = 20 and y = 4
3. x = 5 and y = 6.5
1
4. x = 2
and y = 8

5. x = -6 and y = 24
3 2
6. x = and y =
4 3

7. x = 16 and y = 24
8. x = 12 and y = 6
9. x = 2.5 and y = 7.5
B. In each of the following, “y varies directly as x”.
1. If y = 2 when x = 4, find y when x = 5
2. If y = 9 when x = 18, find y when x = 16.
3. If y = 25 when x = 16, find x when y = 24
4. If y = 4.8 when x = 24, find y when x = 16
4
5. If y = -4 when x = 4, find x when y =
5

6. If y = 6 when x = 6, find y when x = 6


7. If y = 12 when x = 24, find y when x = 12.
1
8. If y = 12 when x = 2, find x when y = 12

9. If y = -9.6 when x = 20, find y when x = 10


2
10. If y = when x = 15, find x when y = 6
5

112 | P a g e
Mathematics
Grade Nine (9)
C. Solve the following problems.
1. If y varies directly as x and is equal to 11 when x = 7, find y when x = 5.

2. If y varies directly as x and y = 12 when x = 15, find x when y = 21.

3. The circumference of a circle varies directly with its diameter. If the


circumference is 8 𝜋 cm and the diameter of a circle is 8 cm, what is the
circumference of a circle whose diameter is 12 cm.

4. Given that y and x are directly proportional, and y = 2 when x = 5, find the
value when x = 5, find the value when x = 15.

5. If y varies directly as x, with y = 8 when x = 2, find y when x = 10.

6. The electric current I(in amperes) in an electric circuit varies directly with the
voltage V.When 24 volts are applied, the current is 9 amp. F ind the current when
15 volts are applied.

7. The weight W of an object on Mars is directly proportion al to its weight B on


Earth. A man who weighs 121.62 kg on Earth weighs 49.3 kg on Mars. Find the
weight of a man on Mars if he weighs 110 kg on Earth.

8. The electric current I in an electric circuit varies dire ctly as the voltage V. When
21 volts is applied the current is 7 amperes. Find the current when 12 vol ts is applied.

9. The amount of commission is directly proportional to the amount of sales. An


insurance agent received a commission of P21 000 on a sale of P14 000 worth of
insurance. How much would the commission be on P210 000 worth of insurance?

10. The distance an automobile travels varies directly as the time. If an automobile
travels 140 km in 2 hours, how far will it go in 9 hours?

113 | P a g e
Mathematics
Grade Nine (9)

LESSON 22
DIRECT SQUARE VARIATION

Direct Square Variation Word Problem


Again, a Direct Square Variation is when y is proportional to the
square of x , or . Let’s work a word problem with this type of variation:

If y varies directly with the square of x , and if y = 4 when x = 3, what


is y when x = 2?

Solution:
Let’s do this with the formula method and the proportion method:

114 | P a g e
Mathematics
Grade Nine (9)
Direct Square Variation Word Problem:

1. The price P of diamond varies directly as the square of its weight w. If the price
of 1.5 gram diamonds is P4 500, find the price of 3.5-gram diamond.

Solution:

Since P varies directly as the square of w, then

P = kw 2
𝑷
k= 𝒘𝟐

𝟒 𝟓𝟎𝟎
= 𝟏.𝟓𝟐

= 2 000

2. The power P in an electric circuit varies directly as the square of the current I.
If 30 volt power has a current of 2 amperes how many volts does a 6 ampere
current have?

Solution:

The relation between P and I is given by the equation

P = kI 2

Substitute P = 30 and I = 2. Then, solve for k

30 = k(2) 2

k = 7.5

Thus, P = 7.5I 2 . If = 6, then

P = 7.5(6) 2

P = 270 volts

115 | P a g e
Mathematics
Grade Nine (9)
QUIZ

A. Express each statement as an equation. Use k as the constant of


variation.

1. The distance (S) of a freely falling body varies directly as the square of the time
(t) during which the body falls.

2. The rate (R) of the flow of water through a pipe varies as the square of the
radius of the pipe.

3. The lateral area (A) of a cube varies directly as the square of an edge.

4. The area (A) of the surface of a sphere varies as the square of the radius.

5. The time (t) required by a pendulum to make one oscillation varies directly as
the square root its length.

6. The stopping distance (d) after breaks are applied varies directly as the square
of velocity.

7. The horsepower (P) required to push onward a ship varies directly as the cube
of its length(L).

8. The kinetic energy (E) of a moving object directly as the square of its velocity(v).

B. Answer the following problems.

1. If D varies directly as the square of R, and D = 10 when R = 5, find D when R =


10.

2. The area of a circle varies directly as the square of its radius r, a nd the constant
of proportionality is 𝝅. Find the area if the radius is 4.7 cm.

3. The wing surface area of an airplane varies directly as the square of the width
of the plane. An airplane 9 m wide has 823 sq. m of wing surface area. What is the
wing surface area of a similar type of airplane 15 m wide?

116 | P a g e
Mathematics
Grade Nine (9)
4. The prize (P) of pizza varies directly as the square of its diameter (d). If the
diameter of a pizza is 20 cm, it costs P180. What is the cost of a pizza whose
diameter is 30 cm?

5. The surface area (A) of a sphere varies directly as the square of the radius (r).
If the surface area is 113 cm 2 when the radius is 3 cm, what is the surface area when
the radius is 9 cm?

6. The area of a square varies directly as the square of its side (s). If the area is 36
sq. cm when the side is 6 cm, find the side when the area is 120 sq. cm

7. If y varies directly as the square of x, and y = 36 when x = 3, find the constant


of variation.

8. If y varies directly as the square of x, and y = 24 when x = 4, find the constant


of variation.

9. If y varies as the square of x and is equal to 2 when x = 6, what is the value of


y when x = 9?

10. The displacement D of a body falling from rest varies directly as the square of
the time T of fall. If it falls 44 m in 3 secs, complete the distance it has traveled
after 8 sec.

11. The area of a circle is directly proportional to the square of its diameter. If the
area of a circle whose diameter is 35 cm is 1136 sq cm, what is the area of a circle
whose diameter is 30 cm? 20 cm? 18 cm? 10 cm?

12. The period of a pendulum (the time of a back and forth swing) is directly
proportional to the square root of its length. If a pendulum 3 cm long has a period
of 2 seconds, what is the length of a pendulum with a period of 6 seconds.

117 | P a g e
Mathematics
Grade Nine (9)

LESSON 23
INVERSE VARIATION

Inverse or Indirect Variation

Inverse or Indirect Variation is referring to relationships of two


variables that go in the opposite direction. Let’s supposed you are comparing how
fast you are driving (average speed) to how fast you get to your school. You might
have measured the following speeds and times:

Average Speed of car Time to get to school x times y


(y) minutes

25 10 25 x 10 = 250

30 8.33 20 x 8.33 ≈250

35 7.14 35 x 7.14 ≈ 250

40 6.25 40 x 6.25 = 250

(Note that means “approximately equal to”).

Do you see how when the x variable goes up, the y goes down, and when
you multiply the x with the y , we always get the same number? (Note that this
is different than a negative slope, since with a negative slope, we can’t multiply
the x ’s and y ’x to get the same number).

So the formula for inverse or indirect variation is:

→ where k is always the same number.

118 | P a g e
Mathematics
Grade Nine (9)
(Note that you could also have an Indirect Square Variation or Inverse
Square Variation, like we saw above for a Direct Variation. This would be of
the form

Here is a sample graph for inverse or indirect variation. This is actually


a type of Rational Function (function with a variable in the denominator) that
we will talk about in the Rational Expressions and Functions section here.

Inverse Variation Word Problem:

So we might have a problem like this:

The value of y varies inversely with x , and y = 4 when x =

119 | P a g e
Mathematics
Grade Nine (9)
3. Find x when y = 6.

The problem may also be worded like this:

Let = 3, = 4, and = 6. Let y vary inversely as x . Find .

Solution:

We can solve this problem in one of two ways, as shown. We do these


methods when we are given any three of the four values for x and y .

Formula Method:

120 | P a g e
Mathematics
Grade Nine (9)
Product Rule Method:

Inverse Variation Word Problem:

For the Choir fundraiser, the number of tickets Allie can buy is inversely
proportional to the price of the tickets. She can afford 15 tickets that cost $5
each. How many tickets can Allie buy if each cost $3?

Solution: Let’s use the product method:

121 | P a g e
Mathematics
Grade Nine (9)
“Work” Inverse Proportion Word Problem:

Here’s a more advanced problem that uses inverse proportions in


a “work” word problem; we’ll see more “work problems” here in the Systems of
Linear Equations Section and here in the Rational Functions and
Equations Section.

If 16 women working 7 hours day can paint a mural in 48 days, how many days will
it take 14 women working 12 hours a day to paint the same mural?

Solution:

The three different values are inversely proportional; for example, the
more women you have, the less days it takes to paint the mural, and the more
hours in a day the women paint, the less days they need to c omplete the mural:

You might be asked to look at functions (equations or points that


compare x’s to unique y’s – we’ll discuss later in the Algebraic
Functions section) and determine if they are direct, inverse, or neither:

122 | P a g e
Mathematics
Grade Nine (9)

123 | P a g e
Mathematics
Grade Nine (9)
QUIZ
A. Translate the following into mathematical statement.

1. The number R of revolution per minute of a gear meshed to another varies


inversely as the number T of the teeth in the gear.

2. The number M of men required to do a specific job varies inversely as the hours
required to do the job.

3. The resistance (R) of wires varies inversel y as the square of its diameter (D).

4. The variable x varies inversely as the square root of y.

5. The variable a varies inversely as the cube root of y.

B. Graph the following.

1. X 6 11 16 21

Y 3 5 7 9

2. X 4 3 6 7

Y 11 16 7 6

3.
X 5 10 15 20

Y 44 22 14.7 11

X 2 5 7 10
4.
Y 21 18 10 8

5. X 1 4 5 6

Y 24 15 11 7

124 | P a g e
Mathematics
Grade Nine (9)
B. Solve the following problems.
3
1. If y varies inversely as x and has the value when x = 10, then what is the value
8
8
of y when x = 9?

2. The distance from the center d of a seesaw vari es inversely with weight of the
child w. Miguel, who weights 90 lb, sits 3.5 ft from the fulcrum. How far from the
fulcrum. must Jose sit to balance Miguel if he weight 85 lb?

3. If time t varies inversely as rate r, with r = 15 when t = -5, find r when t = -10.
𝟏
4. If y varies inversely as x, with y = 𝟒
, when x = 8 find y when x = -4

5. If x varies inversely as y, with y = 27 when x = 3, find x when y = 30.

6. If three men can do a certain job in two days, how many days will it take six
men to the job?

7. The weight of an object varies inversely as the square of its distance d from the
center of the Earth. Find the constant and equation variation if an object weighs
180 lb on the surface of the earth, which is estimated to be 4 000 mi from the
earth’s center.

8. Seven person can finish a job in 9 hours. How many persons are needed to finish
the job in three hours?

9. How long does it take a car to travel a certain distance at 40 kph if the dame
distance can be traveled in 8 hours at 25 kph.

10. If t varies inversely as the square root of p and t = 8 when p = 4, find p when x
= 25

125 | P a g e
Mathematics
Grade Nine (9)

LESSON 24
JOINT VARIATION AND COMBINED
VARIATION

Joint Variation and Combined Variation

Joint variation is just like direct variation, but involves more than one
other variable. All the variables are directly proportional, taken one at a
time. Let’s do a joint variation problem:

Supposed x varies jointly with y and the square root of z. When x = –18 and y =
2, then z = 9. Find y when x = 10 and z = 4.

Let’s set this up like we did with direct variation, find the k, and then solve for y:

126 | P a g e
Mathematics
Grade Nine (9)
Combined variation involves a combination of direct or joint
variation, and indirect variation. Since these equations are a little more
complicated, you probably want to plug in all the variables, solve for k, and then
solve back to get what’s missing. Here is the type of problem you may get:

(a) y varies jointly as x and w and inversely as the square of z . Find the
equation of variation when y = 100, x = 2, w = 4, and z = 20.

(b) Then solve for y when x = 1, w = 5, and z = 4.

Let’s solve:

127 | P a g e
Mathematics
Grade Nine (9)
Combined Variation Word Problem:

The volume of wood in a tree ( V ) varies directly as the height ( h )


and inversely as the square of the girth ( g ). If the volume of a tree is 144 cubic
meters when the height is 20 meters and the girth is 1.5 meters, what is the
height of a tree with a volume of 1000 and girth of 2 meters?

Solution:

128 | P a g e
Mathematics
Grade Nine (9)
Combined Variation Word Problem:

The average number of phone calls per day between two cities has found
to be jointly proportional to the populations of the cities, and inversely
proportional to the square of the distance between the two cities. The
population of Charlotte is about 1,500,000 and the population of Nashville is
about 1,200,000, and the distance between the two cities is about 400 miles. The
average number of calls between the cities is about 200,000.

(a) Find the k and write the equation of variation.

(b) The average number of daily phone calls between Charlotte and Indianapolis
(which has a population of about 1,700,000) is about 134,000. Find the distance
between the two cities.

Solution:

This one looks really tough, but it’s really not that bad if you take it one step at a
time:

129 | P a g e
Mathematics
Grade Nine (9)
Joint Variation Word Problem:

The area of a triangle is jointly related to the height and the base. If the
base is increased by 40% and the height is decreased by 10%, what will be the
percentage change of the area?

Solution:

We probably know the equation for the area of a triangle to be , (b =


base and h = height) so we can think of the area having a joint
variation with b and h , with . So let’s do the math for this problem; we can
just keep the variable k in the problem:

Combined Variation Word Problem:

y varies jointly with and z , and varies inversely with . What is the
effect on y when x is doubled and r is halved?

130 | P a g e
Mathematics
Grade Nine (9)
Solution:

Since we want x to double and r to be halved, we can just put in the new
“values” and see what happens to y . Make sure to put them in parentheses, and
“push the exponents through”:

One word of caution: You found a variation problem in an SAT book that
stated something like this: “If x varies inversely with y and varies directly with z,
and if y and z are both 12 when x = 3, what is the value of y + z when x = 5”. You
found that you had to solve it setting up two variation equations with two
different k‘s (otherwise you can’t really get an answer). So watch the wording
of the problems.

131 | P a g e
Mathematics
Grade Nine (9)
QUIZ

Solve the following word problems.

1. The quantity x varies jointly as x and y, and has the value of 15 when x = 6 and

y = 10.

a. Find the value of z when x = 8 and y = 2

b. Find the value of y when x = 12 and z = 3/2.

2. Find the equation where r varies jointly as s and t and r = 60 when s = 3 and t
= 4.

3. If x varies jointly as y and the square of z and x = 1 5 when y = 5 and z = 1, find x


when y = 4 and z = 6.
7
4. A varies directly as b and inversely as the square of c. A = when b = 14 and c =
2
6. Find the constant variation.

5. If j varies directly as o and inversely as y and j = 30 when o = 20 and y = 50, find


j when o = 40 and y = 25.

6. o varies jointly as u and t. if o = 1.5 when u = 55 and t = 4, find o when u = 75


and t = 3.

7. Find an equation of variation where a varies jointl y as b and c, where a = 30


when b = 6 and c = 8.

8. The area of a rectangle varies jointly as the base (b) and height (h). Fin d the
equation of joint variation if A = 200, b = 50 and h = 16.

9. If x varies jointly as y and z, and x = 5 when z = 8 and y = 480, find the constant
of variation,

10. If y varies jointly as x and z, find y if x = 3, k = 6 and z = 9.

132 | P a g e
Mathematics
Grade Nine (9)
LESSON 25
SCIENTIFIC NOTATION
Scientific notation (also referred to as "standard form" or "standard
index form") is a way of writing numbers that are too big or too small to be
conveniently written in decimal form. Scientific notation has a number of useful
properties and is commonly used in calculators and by scientists, mathematicians
and engineers.
In scientific notation all numbers are written in the form
a × 10 b
( a times ten raised to the power of b ), where the exponent b is an integer, and
the coefficient a is any real number.
Decimal floating point is a computer arithmetic system closely related to
scientific notation.

Decimal notation Scientific notation


2 2×10 0
300 3×10 2
4,321.768 4.321768×10 3
−53,000 −5.3×10 4
6,720,000,000 6.72×10 9
0.2 2×10 −1
0.000 000 007 51 7.51×10 −9

Writing a Number in Scientific Notation


Example 1: 700
Why is 700 written as 7 × 10 2 in Scientific Notation?
700 = 7 × 100 and 100 = 10 2 (see powers of 10 ) so 700 = 7 × 10 2
Both 700 and 7 × 10 2 have the same value, just shown in different ways.

133 | P a g e
Mathematics
Grade Nine (9)
Example 2: 4,900,000,000
1,000,000,000 = 10 9
so 4,900,000,000 = 4.9 × 10 9 in Scientific Notation
Example 3 : 0.0055 is written 5.5 × 10 - 3

Because 0.0055 = 5.5 × 0.001 = 5.5 × 10 -3


Writing a Number from Scientific Notation to Decimal Notation
• Convert 4.2 × 10 – 7 to decimal notation.
Since the exponent on 10 is negative, you are looking for a small number.
Since the exponent is a seven, you will be moving the decimal point seven
places. Since you need to move the point to get a small number, you’ll be
moving it to the left. The answer is 0.000 000 42
• Write in decimal notation: 3.6 × 10 12
Since the exponent on 10 is positive, yu know they are looking for a LARGE
number, so you'll need to move the decimal point to the right, in order to
make the number LARGER. Since the exponent on 10 is "12", you'll need to
move the decimal point twelve places over. First, you'll move the decimal
point twelve places over. You make little loops when you count off the
places, to keep track:

Then you fill in the loops with zeroes:

In other words, the number is 3,600,000,000,000, or 3.6 trillion

134 | P a g e
Mathematics
Grade Nine (9)
QUIZ

A. Express the following to scientific notation.

1. 0.000 000 34

2. 230 000 000

3. 2 000 000 000

4. 0.000 000 345

5. 345 000 000

6. 0. 000 000 34

7. 0. 000 000 000 45

8. 45 000 000 000

9. 90 000 000 000

10. 0. 000 000 000 000 000 6

B. Express the following in decimal notation.

1. 2 x 10 -5

2. 2.34 x 10 3

3. 2 x 10 6

4. 5.678 x 10 -4

5. 5.6 x 10 -7

6. 3.5 x 10 -9

7. 3 x 10 9

8. 2.45 x 10 7

9. 6.78 x 10 -9

10. 3.56 x 10 -9

135 | P a g e
Mathematics
Grade Nine (9)
LESSON 26
INTEGRAL EXPONENTS
Rules for Integral Exponents
1
1. a -n = Division of negative Exponent
𝑎𝑛

1
2. = an Negative Exponent Rules
𝑎−𝑛

3. a 0 = 1 Definition of zero exponent

4. a m a n = a m + n
Product Rule
𝑎𝑚
5. = 𝑎𝑚−𝑛 Quotient Rule
𝑎𝑛

6. (a m ) n = a m n Power of a Power Rule

7. (ab) n = a n b n Power of a Product Rule

𝑎 𝑛 𝑎𝑛
8. (𝑏 ) = Power of Quotient Rule
𝑏𝑛

Examples:

1. (3x 8 y 2 )(-2xy 4 ) = -6x 9 y 6


1 1
2. 2 -3 = =
23 8

2 2−3 33 27
3. (3)−3 = = =
3−3 23 8

4𝑥 9
4. = 4x 8
𝑥

12
5. (3x 2 y 3 )(-4x -2 y -5 ) = -12x 2+ ( - 2) y 3 + ( - 5) = -12x 0 y -2 = -
𝑦2

−6𝑎5𝑏 −1 3
6. = -3a 5 – 7 b -1 – (-4 ) = -3a - 2 b – 5 = - 𝑎 2 𝑏5
2𝑎7 𝑏−4

136 | P a g e
Mathematics
Grade Nine (9)
QUIZ

A. Simplify the following, giving your answer in positive exponent only .

1. (a 2 )(a 3 b) 11. (2a 2 b) -2


4𝑥 −3
2. (xy 3 )(2x 2 y -3 ) 12.
2𝑥 −9

−2
9𝑥𝑦 −12
3. (2m -4 )(-3m 9 ) 13. (3𝑥−2 𝑦7 )

(−𝑚𝑛)8
4. (3mn)(-2m -3 n -3 )(-4mn -3 ) 14. 𝑚𝑛7

5. (-10x 3 y)(2xy) 15. (2x 2 y -3 )(-3x -2 y 4 ) -4


(2𝑥𝑦 7 )−2
6. (x 3 ) -2 16. 4𝑥 −3

7. (x 2b ) -3 17. (-3xy -2 )(4x -4 y -4 ) - 2

8. (5az) -2 18. 2x 2 (3x 2 ) 3

9. a 2 b -4 19. (4a 2 b 3 ) 2

10. (4x 2 y 4 ) -5/2 20.(-27a -6 b 1 2 ) 1/3

B. Simplify each expression.

3 3/2

125𝑎3 𝑏 3
1. (-8a 6 b -6 ) 2/3 6. (− )
8𝑐 −6 𝑑3

2
2. -625 1/2 7. a 2 𝑏−3 𝑐 −2 (𝑎−1 𝑏2 𝑐)

3. (4a 4 ) 2 8. 9x 2 y 4 (-3x -2 y -4 )
3
−27𝑥 −3𝑦 6 2 1
4. ( −
3 ) 9. (4x 2 y 3 )( 2𝑥𝑦)
8𝑥 2

1
5. a 2 b 2 (a -4 b -6 ) 10. (−8𝑎𝑏3 )3

137 | P a g e
Mathematics
Grade Nine (9)
LESSON 27
EXPRESSIONS WITH RATIONAL EXPONENT
In our previous lessons, we learned the laws of exponents. Now, we are
going to apply this knowledge to radicals. We can say that radicals are related to
the exponents as stated in its definition below.
𝑚
𝑛
√𝑥 𝑚 or 𝑥 𝑛 , where m and n are integers and n ≠ 0 is a real number and is positive
if n is even.

Examples:
1
3
1. √8 = 83 = 2
4
3
2. The cube root of x 4 can be written as √𝑥 4 or 𝑥 3

3. The square root of 4x 3 can be written as √4𝑥 3 or (4x 3 ) 1/2

In general, we can say that:

For any nonnegative real number x and any numbers m and n, where n ≠ 1
𝑚
𝑛 𝑛
𝑥 𝑛 = √𝑥 𝑚 = ( √𝑥 𝑚 )m

4. Rewrite 3√𝑥 2 𝑦 with rational exponents


3
√𝑥 2 𝑦 = (x 2 y) 1/3

3 𝑥2 𝑦
5. Rewrite √ with rational exponents
4

3 𝑥2 𝑦 𝑥 2 𝑦 1/3
√ =( )
4 4

138 | P a g e
Mathematics
Grade Nine (9)
QUIZ

A. Write the following in radical form.


1. 8 1/3
2. 5a 2/3
3. 14 -4/5
4. (4 6 ) 1/2
5. 14 7 /5
6. 6 1/2
7. 641 1/2
8. 8 2/3
9. 3 4/5
10. 3x 3 /4
B. Write the following with rational exponents.
1. √5
3
2. √16
7
3. √𝑥
4. √6𝑥
3
5. √4𝑎4
6. √76
3
7. √22
8. √103
9. √43
3
10. √2𝑥

139 | P a g e
Mathematics
Grade Nine (9)
LESSON 28
SQUARE AND CUBE ROOT
The square root of a number is a value that, when
multiplied by itself, gives the number.

Example: 4 × 4 = 16, so a square root of 16 is 4.

Note that (−4) × (−4) = 16 too, so −4 is also a square root of 16

The symbol is √ which always means the positive square root.


Examples:

1. √36 = 6 (because 6 x 6 = 36)

2.The is 4 because 4 x 4 is 16
3. The is 5 because 5 x 5 is 25
4. -√100 = -10 since (-10) 2 = (-10)(-10) = 100 the negative square root is indicated
in the given expression.
5. √(−9)2 = √(−9)(−9) =-√81 = 9

Cube roots
To find the cube root of a number, you want to find some number that
when multiplied by itself twice gives you the original number. In other words, to
find the cube root of 8, you want to find the number that when multiplied by itself
twice gives you 8. The cube root of 8, then, is 2, because 2 × 2 × 2 = 8. Notice that
the symbol for cube root is the radical sign with a small t hree (called the index )
above and to the left . Other roots are defined similarly and identified by the
index given. (In square root, an index of two is understood and usually not written.)
Following is a list of the first eleven perfect (whole number) cube roots.

140 | P a g e
Mathematics
Grade Nine (9)
QUIZ
A. Simplify the following

1. √121 16. 3- √8
3
2. −√36 17. √8
3
3. √49 18. √−64
3
4. √(−4)2 19. √33
3
5. −√1 20. √1331
3
6. √16 21. √9262
3
7. √53 22. √15 625
3
8. √400 23. √4 913
3
9. -√(−52 ) 24. √1 728
3
10. √10 000 25. - √1
3
11. √324 26. √2197
3
12. √1024 27. √729
3
14. √576 28. √2744
3
15. √2116 29. √5832
3 3
15. √9261 30. √1

B. Determine whether each number or expression is a perfect square or


perfect cube.
1. 25 6. 64x 4 y 4
2. 27 7. 1 000x 3 y 6
3. 49 8. 169m 2 n 2
4. 216 9. 729a 6 b 9
5. 225 10. 343z 12

141 | P a g e
Mathematics
Grade Nine (9)
LESSON 29
SIMPLIFYING RADICALS
The "nth Root" used n times in a multiplication gives the original value " nth? "
1st, 2nd, 3rd, 4th, 5th, ... nth ...
Instead of talking about the "4th", "16th", etc, if we want to talk generally we say
the " nth ".
The nth Root

• The "2nd" root is the square root


• The "3rd" root is the cube root
• etc!

Just like the square root is used two times in a


2
multiplication to get the original value.
And the cube root is used three times in a
3
multiplication to get the original value.
... ... ...
The nth root is used n times in a multiplication
n
to get the original value.

So it is the general way of talking about roots


(so it could be 2nd, or 9th, or 324th, or whatever)

The nth Root Symbol

This is the special symbol that means "nth root", it is


the "radical" symbol (used for square roots) with a little n to
mean nth root.

142 | P a g e
Mathematics
Grade Nine (9)
Using it, we could use the nth root in a question like this:

Question: What is "n" in this equation?

Answer: I just happen to know that 625 = 54, so the 4th root of 625 must be 5:

Illustrative Examples:
4
1. Find √81

Solution:
4
√81 = 3 or -3 since 3 4 = 3 x 3 x 3 x 3 = 81 and (-3)(-3)(-3)(-3) = 81

However, the principal 4 t h root of 81is 3.


9
2. √(−5)9 =
𝑘
Solution: 9√(−5)9 = -5 Apply the n t h root theorem √𝑎𝑘 is a since k is odd.

We learned that to simplify radicals, we have to find the square root of any
factor of the radicands that is a perfect square. As we continue in our study, we
will develop rules for finding products and quotients of radicals.

Example 1. 33, for example, has no square factors. Its factors are 3· 11, neither
of which is a square number. Therefore, √33 is in its simplest form.

Example 2. Extracting the square root. 18 has the square factor 9.


18 = 9· 2.
Therefore, √18 is not in its simplest form. We have, √18 = √9.2
We may now extract, or take out, the square root of 9: √18 = √9.2 = 3√2
√18 is now simplified. The radicand no longer has any square factors.
The justification for taking out the square root of 9, is this theorem:

√𝑎𝑏 = √𝑎 ∙ √𝑏

143 | P a g e
Mathematics
Grade Nine (9)
The square root of a product is equal to the product of the square
roots of each factor.

(We will prove that when we come to rational exponents, Lesson 29.
Here is a simple illustration: √100 = √4.25 = √4 ∙ √25 = 2 ∙ 5 = 10

As for √9 ∙ 2 , then, it is equal to the square root of 9 times the square root
of 2, which is irrational 3√2

Example 3 Simplify √75 .

Solution. √75 = √25 ∙ 3 =5√3


75 has the square factor 25. And the square root of 25 times 3
is equal to the square root of 25 times the square root of 3.
√75 is now simplified.

Example 4. Simplify √42 .


Solution. We have to factor 42 and see if it has any square factors. We can
begin the factoring in any way. For example,
42 = 6· 7
We can continue to factor 6 as 2· 3, but we cannot continue to factor 7,
because 7 is a prime number (Lesson 32 of Arithmetic). Therefore,
42 = 2· 3· 7
We now see that 42 has no square factors -- because no factor is repeated.
Compare Example 1 and Problem 2 of the previous Lesson.
√42 therefore is in its simplest form.

Example 5. Simplify √180 .


Solution. We must look for square factors, which will be factors that are
repeated.
180 = 2· 90 = 2· 2· 45 = 2· 2· 9· 5 = 2· 2· 3· 3· 5
Therefore,
√180 = 2 ∙ 3√5 = 6√5

144 | P a g e
Mathematics
Grade Nine (9)
QUIZ

A. Simplify each expression.


4
1. √𝑝4
6
2. √𝑎12
4
3. − √16
3
4. √27𝑝3
7
5. √(−3)9
6
6. √106
4
7. √625
5
8. √32
4
9. − √81
4
10. √1 000 000 000 000

B. Simplify each radical expression.

1. √18 9. √12
3 3
2. √40 10. √16
4
3. √32 11. √6·√3
4
4. √196 12. √128

5. √68 13. √200

6. √75 14. √363


8
7. √80 15. √448

8. √44

145 | P a g e
Mathematics
Grade Nine (9)
LESSON 30
RATIONALIZING THE DENOMINATOR

Rationalizing the denominator of a radical expression is a process of


removing radicals form the denominator. The purpose of rationalization is to
simplify answers or results in a problem radical expression.

Roots Of Fractions Theorem. If all of the radicals involved represent real


𝑘
𝑘 𝑎 √𝑎
numbers then √ = 𝑘 b≠0
𝑏 √𝑏

Roots Of Product Theorem. If all of the radicals involved represent real numbers,
then
𝑘 𝑘 𝑘
√𝑎𝑏 = √𝑎 × √𝑏

Illustrative examples:

5
1. Rationalize √2

Solution:

5 √5
The radical √2 can be written as
√2

√5 √5 √2
= ∙ Multiply both the numerator and denominator by √2 to make the
√2 √2 √2

√10
= denominator a perfect square integer.
√4

√10
= 2

146 | P a g e
Mathematics
Grade Nine (9)
2
2. Rationalize the expression √𝑏.

Assume that b is a positive number.

Solution

2 2 √𝑏 √𝑏
√𝑏 =√𝑏 ∙ Multiply the fraction by
√𝑏 √𝑏

√2𝑏
=
√𝑏2

√2𝑏
= 𝑏

4+ √3
3. Rationalize the expression: 2− √3

Solution:

4+ √3 4+ √3 2 + √3
= ∙ To rationalize the denominator which is a binomial
2− √3 2− √3 2+ √3

multiply by its conjugate.


8+4√3+ 2√3+ √9
=
4+2√3− 2√3− √9

8+6√3+ 3
= 4−3

11+6√3
= 1

= 11 + 6√3

147 | P a g e
Mathematics
Grade Nine (9)
QUIZ

A. Rationalize each of the following.


2 10
1. 9.
√6 √3
4 1
2. 10. 3
√2 √9

3 1
3. 11. √8
√3

1 3 5
4. 3 12. √3
√2

6 2
5. 13. √8
√6

4 3
6. 3 14. √7
√25

√6 1
7. 15. √3
√3

√6
8.
√3

B. Assume that a, b and c are positive numbers.


√3 1+ √3
1. 6.
√3 2− √3

√2𝑎 √3
2. 7.
√3𝑏 √ − √2
3
4 2
3. 8.
√𝑐 √5− 1

√10 √3
4. 9.
√5𝑐 3 2− √3

5 √5− 4
5. 10.
√2𝑐 5 √2− √3

148 | P a g e
Mathematics
Grade Nine (9)
LESSON 31
RATIONAL EXPONENTS AND SIMPLIFYING
RADICALS
1 1 1 1 1 1 1
The product 𝑎2 • 𝑎2 = 𝑎 2 + 2 = a 1 = a. Thus, 𝑎2 = √𝑎 .Similarly, 3√𝑎 = 𝑎3 and 4√𝑎 = 𝑎4 .
1
Definition: For any positive integer k a ≥ 0 when k is even, then 𝑘√𝑎 = 𝑎𝑘 and
if
1
1
a ≠ 0, then 𝑎−𝑘 = 𝑘 .
√𝑎

𝑝
Definition: For any natural numbers p and q and any real number a, 𝑎𝑞 =
𝑞
√𝑎𝑝
−𝑝
1 𝑞
and a ≠ 0. Whereas, 𝑎 = 𝑞
𝑝 ; √𝑎𝑝 is a real number.
𝑎𝑞

Theorem If all the expressions represent real numbers, then


𝑞 𝑝
√ 𝑝√𝑎 = √ 𝑞√𝑎 = 𝑝𝑞
√𝑎

Fractional exponents are very useful in simplifying radical expressions.

Examples:
2
1. Evaluate 83

Solution:
2
3
83 = √82
3
= √64

=4

149 | P a g e
Mathematics
Grade Nine (9)
1
2. Simplify 9−2
Solution:
1
1
9−2 = 1
92
1
=
√9
1
=
3

3. Simplify the 8 t h root of m 4 . Assume m is positive.


Solution:
4
8
√𝑚4 = 𝑚8 Rewrite the expression to exponential form.
1
= 𝑚2
= √𝑚
9
4. Assume that b is positive, simplify √64𝑏3
Solution:
1
9 3
√64𝑏3 = (43 𝑏3 )9 = (4𝑏)33 = √4𝑏
3
5. Simplify √5 • √125
Solution:
1 1
3
√5 • √125 = 52 • (53 )3
1
= 52 • 51
3
= 52
= √125
√5
6. Simplify 3
√5

Solution:
1
1 1 1
√5 52
= 52− 3 = 56 = √5
6
3 = 1
√5 53

150 | P a g e
Mathematics
Grade Nine (9)
QUIZ

A. Find the numerical value of each.


2 3
1. 83 6. 92
2 2
2. 1253 7. 64−3
2 1
3. 273 8. 6254
2 1
4. 13 9. 252
2 1
5. 254 10. 643

B. Simplify each of the following.

2
6. √𝑎3
3
1. √2 • √8
1
2. (4x 4 y 6 ) 1/2 7. [(2𝑎2 )3 ]2
1 2 1
3. 𝑥 2 ∙ 𝑥 1 ∙ 𝑥 3 8. [(3𝑥 2 )−2 ]2

4. a 1/2 • a 2/3 9. 16 -1 /3

√4 4
5. 4 10. √6 • √48
√4

151 | P a g e
Mathematics
Grade Nine (9)
Victory Elijah Christian College

FIRST MONTHLY EXAM – MATHEMATICS GRADE 9

Name: Grade:

Test I. Identify the value of a, b and c of the following quadratic


equations.

1) 3x 2 + 4x – 8 = 0 6) 2 x 2 – x = 4
2) 3x 2 = - x + 8 7) (x + 5) 2 = 9
3) 5(x – 2) 2 = 0 8) 2x 2 – (3x – 6) = 0
4) (x + 4)(x – 5) =9 9)
𝑥+3
=
3𝑥
4 4 𝑥
5) 2x = 10) x(x - 6) = 3x
3𝑥 −8

Test II. Determine whether each equation is quadratic equation or not.

1) x 2 – 3x + 6 = 12 6) (x – 4) 2 + 6x = 4x – 6
2)
1
𝑥 2 = 3x – 4 7) [x (x – 2) 2 - 3] = 7
2
8) (x + 4) + 5 = 0
3) 2x 2 – 3x + 14 = x 3
9) 2(x + 3) 2 = 0
4) x 2 + 10 = 4x
10) 2x 2 – 6x + 12 = 12
5) (x – 5) 2 + 8 = 9
Test III. Apply the Zero Product Principle to solve each equation.

1) (x – 8)(2x – 6) = 0
2) (4x – 2)(x + 9) = 0
3) (3x + 12)(3x + 21)(2x – 12) = 0
4) 3x(x – 5) = 0
5) (12x)(4x + 120)(2x + 26) = 0
6) (2x – 3)(x – 5) = 0
7) (4x – 16)(2x – 4) = 0
8) (x – 5)(x – 3)(x – 9) = 0
9) 3x(2x – 46) = 0
10) (4x)(12x + 80)(13x – 65) = 0

152 | P a g e
Mathematics
Grade Nine (9)
Test IV. Solve then choose the lette r of the correct answer for each
problem.

1) x 2 – 36 = 0 6) x 2 – 121 = 0
2) x 2 – 25 = 0 7) 25x 2 = 100
3) 4x 2 – 49 = 0 8) 24x 2 = 384
4) 3x 2 = 169 9) 7x 2 – 252 = 0
5) x 2 = 1024 10) 2x 2 – 1 800 = 0

Test V. Find the equivalent quadratic equation of the following given the
roots.

1) -3, 2 7) 3,
1
3
2) 5, 5 3
3) 5, 1 8) , 2.5
4
7
4) -4, 3 9) , -6
8
5) -4, 4
2
10) -4, 12
6) - 5 , 6

Test VI. Solve the Equations by completing the squares.

1) x 2 + 5x + 6 = 0
2) 2m 2 + 3m + 4 = 0
3) 4x 2 + 8x + 3 = 0
4) y 2 + 10y + 25 = 0
5) b 2 – 3b = -2
6) a 2 + 6a = -8
7) 3n 2 + 4n + 1 = 0
8) p 2 – 8p = 20
9) -4 = a 2 – 6a
10) t 2 + 18t = 45

153 | P a g e
Mathematics
Grade Nine (9)
Victory Elijah Christian College

SECOND MONTHLY EXAM – MATHEMATICS GRADE 9


Name: Grade:

Test I. Solve each of these equations using the quadratic formula. (10pts)

1) x 2 – 8x + 16 = 0 6) 2p 2 – 3p + 12 = 0
2) y 2 – 2y – 15 = 0 7) m 2 + 6m + 12 = 0
3) 2x 2 +16x + 12 = 0 8) 3x 2 – 108 = 0
4) x2 – x + 2 = 0 9) 2s 2 + 5s = 9
5) 3x 2 – 3x – 2 = 0 10) x 2 – 7x + 12 = 0

Test II. Compute the value of discriminant for each equation and then
describe the nature of the roots (no distinct real roots, two distinct real
roots or one real root. (10pts)

1) p 2 + 5p – 4 = 0 6) x 2 – 3x – 4 = 0
2) m 2 = 6m + 9 7) 6x 2 + 6 = -10x
3) –x 2 + x + 8 = 0 8) x 2 – 4x + 12 = 0
4) 2x 2 – 5x + 6 = 0 9) 2n 2 – n = -3
5) x2 – x = 1 10) x 2 + 1 = x

Test III. Identify the sum and product of the roots of the following
equations. (20pts)

1) 2x 2 – 4x + 9 =0
2) 3x 2 + 9x – 12 = 0
3) m 2 + 8m = 8
4) 2x 2 – 3 = 6x
5) m(2m+14) = 5
6) 2x 2 – x – 12 = 0
7) 4x 2 = 6x + 5
8) 3y 2 = 5y + 8
9) p(p + 3) = 4p + 5
10) x 2 + 5x – 28 = 0

154 | P a g e
Mathematics
Grade Nine (9)
Test V. Solve the following equations. (10pts)

𝑥+10
1) = 𝑥2
6
𝑥2 𝑥−5
2) =
2 6
𝑥2 𝑥−6
3) =
10 8
𝑥+4 𝑥−2
4) =
𝑥 6
𝑥+2 1
5) 8
= 𝑥−1
𝑥+9 1
6) =
𝑥2 4
𝑥+3
7) = 𝑥 − 12
𝑥−5
𝑥+3 𝑥−1
8) 𝑥−5 2𝑥−1
=
𝑥+7 2𝑥−1
9) =
5+𝑥 𝑥+1
𝑥+1 4 𝑥
10) + =
3 𝑥−2 4

Test VI. Identify the critical values of the following inequalities . (20pts)

1) x 2 – 16 > 0
2) x 2 – 8x ≤ 0
2
3) +x≤3
𝑥
4) x 2 – 25 > 0
5) x 2 + 12x ≤ -20
6) x 2 – 6x < 16
7) 2x 2 + 5x ≤ -2
8) x 2 + 3x < -2
9) x 2 + 6x - 16 < 0
10) x 2 + 8x + 12 ≤ 0

155 | P a g e
Mathematics
Grade Nine (9)
Victory Elijah Christian College

FIRST QUARTERLY EXAM – MATHEMATICS GRADE 9


Name: Grade:

Test I. Determine whether each relation describe a quadratic function

1) y = 3x 3 + 2x – 1
2) y = 2(x -1) 2
3) 3(x – 1) = y – (x + 1)(x – 1)
4) 3x = 8y
5) y = 4x 2
6) y = 4x(x + 3)
7) y = 3 – 6(x + 2)
8) 8xy = 3
9) y = (2x – 3) 2
10) 2(x – 2) 3 = y

Test II. Identify the range of each quadratic function .

1) f(x) = 2x 2 + 3x + 5
2) f(x) = x 2 + 4x – 5
3) f(x) = -5x 2 + 4x + 8
4) f(x) = -3x 2 + 4x + 20
5) f(x) = -x 2 + 3x + 12
6) f(x) = -2x 2 + 3x + 12
7) f(x) = -3x 2 + 5x + 12
8) f(x) = -6x 2 + 12x + 8
9) f(x) = 5x 2 - 6x + 6
10) f(x) = 4x 2 + 3x + 6

Test III. Determine the vertex of the following quadratic functions .

1) f(x) = -4x 2 + 2x + 4
2) f(x) = 3x 2 + 7x + 12
3) f(x) = -x 2 +6x – 12
4) f(x) = -4x 2 + 2x + 14

156 | P a g e
Mathematics
Grade Nine (9)
5) f(x) = 5x 2 + 5x + 16
6) f(x) = 6x 2 + 6x + 8
7) f(x) = -x 2 + 16x + 28
8) f(x) = 5x 2 – 3x + 12
9) f(x) = -12x 2 +16x +12
10) f(x) = 4x 2 + 12x + 24

Test IV. Determine the direction of the parabola of each quadratic


function. Write OPENS UPWARD or OPENS DOWNWARD

1) f(x) = -4x 2 + 2x + 4 6) f(x) = 6x 2 + 6x + 8


2) f(x) = 3x 2 + 7x + 12 7) f(x) = -x 2 + 16x + 28
3) f(x) = -x 2 +6x – 12 8) f(x) = 5x 2 – 3x + 12
4) f(x) = -4x 2 + 2x + 14 9) f(x) = -12x 2 +16x +12
5) f(x) = 5x 2 + 5x + 16 10) f(x) = 4x 2 + 12x + 24

Test V. Identify the equivalent vertex form of the following.

1) y = x 2 + 3x + 2
2) y = x 2 – 2x + 1
3) y = x 2 – 5x – 45
4) y = 4x 2 – 3x + 8
5) y = x 2 + 10x + 24

Test VI. Find the missing numbers to form the equivalent y = ax 2 + bx +


c of the following vertex form.

1) y = 3(x – 6) 2 + 5
2) y = (x – 10) 2 – 6
3) y = 3(x – 8) 2 + 6
4) y = 4(x – 2) 2 + 3
5) y = 2(x + 6) 2 + 8

157 | P a g e
Mathematics
Grade Nine (9)
Test VII. Fill in the blanks to complete the given statements.

1) The sign of a in the quadratic function f(x) = a(x – h)2 + k determines


whether the parabola opens or . Recall that the
parabola opens upward if a 0 it opens downward if a 0.
2) In general, if h > 0, subtracting h from x shifts the graph to the by
h units. On the other hand, if h < 0, subtracti ng from x shifts the graph to
the by IhI units. These are called shifts of the graph.
3) In general, if k > 0 adding k to f(x) shifts the graph b k units,
while subtracting k from f(x) shifts the graph by k units. These are
called shifts of the graph of f(x).

Test VIII. Given the vertex and point identify the quadratic function to
give the value of a, b and c.

1) V(0,-1) ; (1, 0)
2) V(0,3) ; (1, 0)
3) V(2,-1) ; (1, 1)
4) V(1,-2) ; (2, 1)
5) V(-1,-4) ; (-2, -3)

Test IX. Given the three intercepts or points of the quadratic function
give the value of a, b, and c.

1) (2/3, 0), (-1, 0) and (0, -2)


2) (2, 0), (-4, 0) and (0, -8)
3) (2, 16), (3, 25) and (1, 9)
4) (-2, 5), (2, -3) and (1, 2)
5) (-1, -3), (-2, -5) and (0, -11)

Test X. Solve the following word problems.

1) The length of a rectangle is 6 cm more than its width. Find its length and
width when its area is 40 cm 2 .
2) The product of two consecutive odd numbers is 255. Find the numbers.
3) The length of a rectangle is 3 cm mor e than the width and its area is 54 cm 2 .
Find the length of the diagonal of the rectangle.

158 | P a g e
Mathematics
Grade Nine (9)
Victory Elijah Christian College

THIRD MONTHLY EXAM – MATHEMATICS GRADE 9


Name: Grade:
Test I. Directions: Find the domain of the rational expression.

4−𝑚 𝑎−𝑏
1) 6)
𝑚 𝑎2 − 2𝑎𝑏+ 𝑏2
𝑚−2 𝑥+3
2) 7)
2−𝑚 𝑥 2 − 8𝑥−20
𝑥 2𝑥
3) 𝑥+3
8)
𝑥 2 − 16
7 𝑥
4) 9)
𝑚−3 𝑥 2 − 𝑥−2
𝑚3 2𝑎
5) 10.) 𝑎−6
𝑚2 + 3𝑚+2

Test II. Directions: Perform the Test III. Directions: Determine the
indicated operations and reduce LCD of the following se of rational
answers to lowest term. expression.
1 2 2 𝑎 2 3
1. 5
+ 5
+ 5
1. ,
4 4𝑎 12𝑥
,

3 4 1 𝑥 2𝑥
2. − + 2. ,
9 9 9 𝑥+3 𝑥 2 + 6𝑥+9

4 3 6 1 4 5 𝑥+4
3. + − + 3. , ,
12 5 4 12 𝑥 2 2𝑥+4 𝑥 2 + 4𝑥+4

4 2 6 2 5 3 3 2𝑥
4. + + − 4. , , ,
5 3 8 10 𝑥+3 2𝑥−3 2𝑥+6 2𝑥 2 + 3𝑥−9

6 2 6 1 1 𝑥 2 2
5. − + + − 5. , ,
7 7 8 4 4 9𝑥 𝑥 2 − 9𝑥 𝑥+3

159 | P a g e
Mathematics
Grade Nine (9)
Test IV. Directions: Solve each rational equations and determine whether
the solution is real or extraneous.
2𝑥 3𝑥 1 𝑥 2 + 8𝑥+15
1. + = 6. =8
6 6 12 𝑥+3
1 7 9 1 3 1
2. − = 7. + =
2𝑥 4𝑥 10𝑥 𝑥+1 2 𝑥
𝑥+1 𝑥−1 2𝑥 2𝑥 𝑥
3. + = 8. + =1
𝑥−2 𝑥+2 𝑥2 − 4 𝑥2 − 4 𝑥+2
𝑥 𝑥 1 4𝑎 6𝑎 12𝑎 16𝑎
4. + − =2 9. + = −
𝑥+1 2𝑥+2 −𝑥−1 12 18 24 32
4𝑥 𝑥−4 9 1 4
5. 𝑥−4
= 𝑥 2 − 16
10. 2( 𝑥+2)
+ 2𝑥
= 𝑥+1

Test V. Directions: Solve the following problems.


1. If y varies directly as x and is equal to 12 when x = 7, find y when x = 5 .
2. If y varies directly as x and y = 12 when x = 15, find x when y = 21.
3. The circumference of a circle varies directly with its diameter. If the
circumference is 8 𝜋 cm and the diameter of a circle is 8 cm, what is the
circumference of a circle whose diameter is 12 cm.
4. Given that y and x are directly proportional, and y = 2 when x = 5, find the
value when x = 5, find the value when x = 15.
5. If y varies directly as x, with y = 8 when x = 2, find y when x = 10.
6. If D varies directly as the square of R, and D = 20 when R = 5, find D when R =
10.
7. The area of a circle varies directly as the square of its radius r, and the constant
of proportionality is 𝝅. Find the area if the radius is 4.7 cm.
8. The wing surface area of an airplane varies directly as the square of the width
of the plane. An airplane 9 m wide has 823 sq. m of wing surface area. What is
the wing surface area of a similar type of airplane 15 m wide?
9. The distance from the center d of a seesaw varies inversely with weight of the
child w. Miguel, who weighs 90 lb, sits 3.5 ft from the fulcrum. How far from the
fulcrum? must Jose sit to balance Miguel if he weighs 85 lb?
160 | P a g e
Mathematics
Grade Nine (9)
10. If time t varies inversely as rate r, with r = 15 when t = -5, find r when t = -10.
𝟏
11. If y varies inversely as x, with y = , when x = 12 find y when x = -4
𝟒

12. If x varies inversely as y, with y = 27 when x = 6, find x when y = 30.


13. The quantity x varies jointly as x and y, and has the value of 30 when x = 6 and
y = 10.
a. Find the value of z when x = 8 and y = 2
b. Find the value of y when x = 12 and z = 3/2.
14. Find the equation where r varies jointly as s and t and r = 60 when s = 3 and t
= 4.
15. If x varies jointly as y and the square of z and x = 15 when y = 5 and z = 1, find x
when y = 4 and z = 6.

161 | P a g e
Mathematics
Grade Nine (9)
Victory Elijah Christian College

SECOND QUARTERLY EXAM – MATHEMATICS GRADE 9


Name: Grade:

Test I. Directions: Express the following to scientific notation.


1) 0.000 000 34 6) 230 000 000
2) 2 000 000 000 7) 0.000 000 345
3) 345 000 000 8) 0. 000 000 34
4) 0. 000 000 000 45 9) 45 000 000 000
5) 90 000 000 000 10) 0. 000 000 000 000 000 6

Test II. Directions: Express the following in decimal notation.


1) 2 x 10 4 6) 2.34 x 10 5
2) 2 x 10 3 7) 5.678 x 10 5
3) 5.6 x 10 3 8) 3.5 x 10 4
4) 3 x 10 9 9) 2.45 x 10 7
5) 6.78 x 10 9 10) 3.56 x 10 4

Test III. Directions: Simplify the following, giving your answer in positive
exponent only.
1) (a 2 )(a 3 b) 2) (8a 2 b)-2
4𝑥 −3
3) (xy 3 )(2x 2 y-3) 4) 2𝑥 −9
−2
9𝑥𝑦 −12
5) (5m-4)(-3m 9 ) 6) (3𝑥−2 𝑦 7)
(−𝑚𝑛)8
7) (3mn)(-2m-6n-3)(-4mn-3) 8) 𝑚𝑛7
9) (-10x 6 y)(2xy) 10) (2x 2 y-3)(-3x-2y 4 )-3
3 3/2
− (−8a6b−12)2
125𝑎3 𝑏 3
11) (− ) 12)
8𝑐 −6 𝑑3 3

2
13) 9x 2 y 4 (-3x-2y-4) 14) 𝑎6 𝑏 −3 𝑐 −2 (𝑎−1 𝑏2 𝑐)
−6251
15) 2
16) (4a 6 )2

162 | P a g e
Mathematics
Grade Nine (9)
3
−27𝑥 −3 𝑦 6 2 1
17) ( 3 ) 18) (4x 2 y 3 )(2𝑥𝑦)

8𝑥 2
1
19) a 2 b 2 (a-4b-6) 20) (−8𝑎𝑏3 )3

Test IV. Directions: Write the following with rational exponents.


3 3 3
1) √5 2) √16 3) √4𝑎4 4) √22 5) √43

3
6) 7√𝑥 7) √6𝑥 8) √76 9) √103 10) √2𝑥

Test V. Directions: Simplify the following


3 3 3
1) √144 2) √196 3) √1728 4) √33 5) √12 167
3 3 3 3
6) −√36 7) - √8 8) √−64 9) √9 261 10) √15 625

11) √(−4)2 12) −√1 13) √529 14) √53 15) √400
3 3 3
16) -√(−52 ) 17) √1 728 18) √4 913 19) - √32 768 20) √10 000

Test VI. Directions: Simplify each radical expression.


4 3
1) √18 2) √32 3) √147 4) √16 5) √192

3 4
6) √40 7) √196 8) √12 9) √6·√3 10) √200

Test VII. Directions: Rationalize each of the following.


2 4 3 4 2
1) 2) 3) 4) 5)
√8 √2 √5 √𝑐 √5− 2

10 1 √7 √10
6) 7) 3
8) √10
1 9) 10)
√3 √9 2− √7 √5𝑐 3

163 | P a g e
Mathematics
Grade Nine (9)
Test VIII. Directions: Find the numerical value of each.
2 2 2 2 2
1) 83 2) 1253 3) 273 4) 13 5) 254

3 2 1 1 1
6) 6. 92 7) 64−3 8) 6254 9) 252 10) 643

164 | P a g e
Module 2
Period: 3rd and 4th Quarter
MATHEMATICS
Grade Nine (9)

Module 2
Learning Introduction and Overview
This module will discuss the importance of a rational exponents and
radicals in your everyday life. The learners will explore and familiarizes
how exponents, whether it is integral or rational exponents are used in
many ways. We are also going to look at matrices, a subject that has
wide applications in the fields of pure mathematics, statistics,
engineering and the physical and social sciences.

This module also introduces the concept of quadril aterals. Different


kinds of polygons can be found in all architectural designs. The study of
the properties of these polygons is very important in the field of
architecture and in other fields of engineering.

The learners will explore and learn the similar ities of geometric
figures. Anywhere you can see similar but not identical images. How will
you know that the two images are similar? Figures may be enlarged or
reduced from its original size yet they are similar. This module will discuss
the properties of similar figures specially triangles.

One of the most fundamental in mathematics is to understand the


concept of a triangle trigonometry. The term trigonometry is based from
the Greek word trigonon which means “triangle”, and metron which
means “measure”. This discipline dates back to over 3000 years, and has

166 | P a g e
MATHEMATICS
Grade Nine (9)
been instrumental in developing knowledge in architecture, astronomy,
navigation and surveying.

Learning Outcomes
Upon successful completion of the module the learners will be able to:

1. Simplifies radical expressions using the laws of radicals.


2. Performs operations on radical expressions.
3. Solves equations involving radical expressions.
4. Solves problems involving radicals.
5. Performs operations on a Matrix by a Real Number.
6. Identifies quadrilaterals that are parallelograms.
7. Determines the conditions that make a quadrilateral a
parallelogram.
8. Uses properties to find measures of angles, sides and other
quantities involving parallelograms.
9. Proves theorems on the different kinds of parallelogram
(rectangle, rhombus, square).
10. Proves the midline theorem.
11. Proves theorems on trapezoids and kites.
12. Solves problems involving parallelograms, trapezoids and kites.
13. Describes a proportion.
14. Applies the fundamental theorems of proportionality to solve
problems involving proportions.
15. Illustrates similarity of figures.

167 | P a g e
MATHEMATICS
Grade Nine (9)
16. Proves the conditions for similarity of triangles.
17. Applies the theorems to show that given triangles are similar.
18. Proves the Pythagorean Theorem.
19. Solves problems that involve triangle similarity an d right triangles.
20. Illustrates the six trigonometric ratios: sine, cosine, tangent,
secant, cosecant, and cotangent.
21. Finds the trigonometric ratios of special angles.
22. Illustrates angles of elevation and angles of depression.
23. Uses trigonometric ratios to solve real-life problems involving
right triangles.
24. Illustrates laws of sines and cosines.
25. Illustrates laws of sines and cosines.
26. Solves problems involving oblique triangles.

Learning Contents:
CHAPTER 5: RATIONAL EXPONENTS AND RADICALS
Lesson 31: Rational Exponents and Simplifying Radicals
Lesson 32: Addition and Subtraction of Radicals
Lesson 33: Multiplication of Radical Expression
Lesson 34: Division of Radicals
Lesson 35: Solving Radical Equations
Lesson 36: Problem Solving Involving Radical Equations

168 | P a g e
MATHEMATICS
Grade Nine (9)
CHAPTER 6: MATRIX
Lesson 37: Introduction and Some Special Matrices
Lesson 38: Addition and Subtraction of Matrices
Lesson 39: Multiplication of a Matrix by a Real Number
Lesson 40: Multiplication of Matrices

CHAPTER 7: PARALLELOGRAMS, KITES AND TRAPEZOID


Lesson 41: Properties of Parallelograms
Lesson 42: Finding the Measures of Angles, sides and other quantities
involving Parallelogram
Lesson 43: Properties of Rhombuses, Rectangles, and Squares
Lesson 44: Midline Theorem
Lesson 45: Trapezoid and Kites

CHAPTER 8: SIMILARITIES OF GEOMETRIC FIGURES


Lesson 46: Ratio and Proportion
Lesson 47: Proportional Segments and Similarity
Lesson 48: Similar Triangles
Lesson 49: Proportionality Theorems
Lesson 50: Similarity and Isosceles Triangle Theorems

CHAPTER 9: PROPERTIES OF RIGHT TRIANGLES


Lesson 51: Pythagorean Theorem
Lesson 52: The Pythagorean Condition
Lesson 53: Special Triangles

169 | P a g e
MATHEMATICS
Grade Nine (9)
CHAPTER 10: SOLVING PROBLEM RELATED TO TRIANGLE TRIGONOMETRY
Lesson 54: Angles and Their Measures
Lesson 55: Trigonometric Ratios
Lesson 56: D°M’S’’ and Decimal Degrees
Lesson 57: Cofunctions and Trigonometric Function Values of Any
Angle
Lesson 58: Solving Right Triangles
Lesson 59: Angles of Elevation and Depression
Lesson 60: Law of Sines
Lesson 61: Law of Cosines
Lesson 62: Fundamental Identities

170 | P a g e
MATHEMATICS
Grade Nine (9)

Lesson 31
Rational Exponents and Simplifying
Radicals
1 1 1 1 1 1 1
+
The product 𝑎 • 𝑎 = 𝑎 = a1 = a. Thus, 𝑎 = √𝑎 .Similarly, √𝑎 = 𝑎 and √𝑎 = 𝑎 .
3 4
2 2 2 2 2 3 4

Definition: For any positive integer k a ≥ 0 when k is even, then 𝑘√𝑎 =


1
𝑎𝑘 and if
1
1
a ≠ 0, then 𝑎 −𝑘 = 𝑘 .
√𝑎

Definition: For any natural numbers p and q and any real number a,
𝑝
𝑞
𝑎 = √𝑎𝑝 and a ≠ 0.
𝑞

−𝑝
1 𝑞
Whereas, 𝑎 = 𝑞
𝑝 ; √𝑎𝑝 is a real number.
𝑎𝑞

Theorem If all the expressions represent real numbers, then

𝑞 𝑝
√ 𝑝√𝑎 = √ 𝑞√𝑎 = 𝑝𝑞
√𝑎

171 | P a g e
MATHEMATICS
Grade Nine (9)
Fractional exponents are very useful in simplifying radical expressions.
Examples:
2 1
1. Evaluate 83 2. Simplify 9−2
Solution: Solution:
2 1
1
9− 2 =
3
83 = √82 1
92
= √64
3
1
=
√9
=4
1
=
3

3. Simplify the 8th root of m4. 4. Assume that b is positive,


Assume m is positive. simplify √64𝑏 3
9

Solution: Solution:
4 1
8
√𝑚4 = 𝑚8 Rewrite the 9
√64𝑏 3 = (43 𝑏 3)9 = (4𝑏 )33 =
expression to exponential form. 3
√4𝑏
1
=𝑚 2

= √𝑚
5. Simplify √5 • √125
3
√5
6. Simplify 3
√5
1 1
Solution: √5 • √125 = 5 • (5
3 3 )3
2
Solution:
1
1
= 5 • 51
2 √5 52 1 1 1
= = 52− 3 = 56 = √5
6
3 1
3 √5 53
= 52
= √125

172 | P a g e
MATHEMATICS
Grade Nine (9)

Quiz
A. Find the numerical value of each.
2 3
1. 83 6. 92
2 2
2. 1253 7. 64−3
2 1
3. 27 3 8. 6254
2 1
4. 13 9. 252
2 1
5. 254 10. 643

B. Simplify each of the following.


2
1. √2 • √8 6. √𝑎3
3

1
2. (4x4y6)1/2 7. [(2𝑎2 )3 ]2
1 2 1
3. 𝑥 2 ∙ 𝑥 1 ∙ 𝑥 3 8. [(3𝑥 2 )−2 ]2
4. a1/2 • a2/3 9. 16-1/3
√4
5. 10. √6 • √48
4
4
√4

173 | P a g e
MATHEMATICS
Grade Nine (9)

Lesson 32
Addition and Subtraction of Radicals
Two radical expressions are called like-radical expressions if both the indices
and the radicands are alike. They can be added or subtracted and the result
expressed is the same. We can also use the distributive property of addition and
subtraction.

Adding and subtracting radicals is the same as in adding and subtracting


polynomials terms. We simply add or subtract those with like terms, while cannot
add or subtract those with unlike terms. In the expression 2 √3 + 3√5 , we cannot
add the two radicals since they have different radicands.

Simplify each of the following.


1.) 4 + 3√7 + 6 + 5√7
Solution:
4 + 3√7 + 6 + 5√7
= (4 + 6) + (3√7 + 5√7)

= 10 + 8√7

174 | P a g e
MATHEMATICS
Grade Nine (9)
2. 5√27 + 6√3 − 4√48
Solution:

3.) √24𝑎 + √81𝑎


3 3

Solution:

Quiz
A. Perform the indicate operation(s)
1. 2√3 − 4√3 6. 8√72 + 2√20 − 3√5
2. 5√𝑥 − 9√𝑥 7. 8√50 + 3√72 − 2√98
3. 2√3 + √5 8. 3√5𝑚 − 5√5𝑚
4. 2√3 + √12 9. 𝑎√3 + 2𝑏 √3 + 2√3
5. √18 + √108 + 2√50 10. 4 √7 + 2𝑎 √7 − 2 √28
3 3 3

B. Simplify the following.


1. 3√𝑥 + √𝑥 6. a√3 + 2𝑏√3 + 2√3 − 5√3
2. √20 + √40 + √80 7. √6𝑦 2 - y√96
3. 4√3 + 7√12 − 5√75 8. √6𝑥 2 𝑦 - √16𝑥 2 𝑦 + √9𝑥 2 𝑦
4. 4√6 + √7 − √6 − 4√7 9. 4m√27𝑚2 𝑛 + 2m√3𝑛
5. 3√5𝑚 − 5√5𝑚 10. 7 √5𝑥 + 4 √5𝑥
3 3

175 | P a g e
MATHEMATICS
Grade Nine (9)

Lesson 33
Multiplication of Radical Expression

It is possible to simplify complicated radical expressions using different


properties, such as commutative, associative, and distributive and the product
property of square roots.
Problem

Simplify.

Look for perfect squares in each radicand,


and rewrite as the product of two factors.

Identify perfect squares

Rewrite as the product of radicals.

Simplify, using

Multiply.

Answer

176 | P a g e
MATHEMATICS
Grade Nine (9)
Problem 1
Simplify.
Use the rule to multiply the radicands.

Recall that .

Look for perfect squares in the radicand.


Rewrite as the product of radicals.

Answer:

Problem 2
Simplify.
Notice that both radicals are cube roots, so you can use the
rule to multiply the radicands.

Look for perfect cubes in the radicand. Since is not a


perfect cube, it has to be rewritten as .
Rewrite as the product of radicals.
Answer:

177 | P a g e
MATHEMATICS
Grade Nine (9)

Quiz
A. Find the product.

1. (√6)(√11) 2. (√3)(√5)
3. (√15)(√2) 4. (2√2)(√5)

5. (5 √3𝑥 2 𝑦)(√12𝑥 2 𝑦 3 ) 6. (√6 · √11)3


4 4

7. 2√6· 4√8 8. (4√3𝑎3 )2

9. √3𝑥 ∙ √2𝑥 ∙ 4√3𝑥 2 10. 3 √5 ∙ √25


3 4

B. Perform the indicated operation.

1. √50𝑎 • √2𝑎𝑏 2. √30𝑚3 𝑛 • √10𝑚𝑛2

3. √13𝑥 2 • √13𝑥 4. √16𝑥 3 𝑦 2 • √5𝑥 3

5. √54𝑎 2 𝑏 4 • √2𝑎𝑏 6. 2√49𝑝 • √2𝑝

7. √45𝑎2 𝑏𝑐 • √4𝑎2 𝑐 8. (3√6 + 4√3)(√6 − 3√3)


9. (√7 √2)2 10. (√5 − 3√7)(√5 − √7)

178 | P a g e
MATHEMATICS
Grade Nine (9)

Lesson 34
Division of Radicals

The quotient property of square roots can be used in simplifying expressions


involving division of radicals. We use the fractional form and then rationalize the
result.
Just as you can swap between the multiplication of radicals and a radical
containing a multiplication, so also you can swap between the division of roots
and one root containing a division.
𝟖
• Simplify: √𝟐

You can simplify this by working inside, and then taking the square root:

...or else by splitting the division into two radicals, simplifying, and
cancelling:

𝟑
• Simplify: √𝟐𝟓

179 | P a g e
MATHEMATICS
Grade Nine (9)
𝟔√𝟐
• Simplify:
√𝟑

Don't stop once you've rationalized the denominator. As the above


demonstrates, you should always check to see if something remains to be
simplified.
𝟑
• Simplify: 𝟐+√𝟐

This expression is in the "wrong" form, due to the radical in the denominator.
But if you try to multiply through by root-two, you won't get anything useful:

Multiplying through by another copy of the whole denominator won't help,


either:

180 | P a g e
MATHEMATICS
Grade Nine (9)

But look what happens when I multiply by the same numbers, but with the
opposite sign in the middle.

This multiplication made the radical terms cancel out, which is exactly what
you want. This "same numbers but the opposite sign in the middle" thing is
the "conjugate" of the original expression. By using the conjugate, you can
do the necessary rationalization.

Do not try to reach inside the numerator and rip out the 6 for "cancellation". The
only thing that factors out of the numerator is a 3, but that won't cancel with
the 2 in the denominator. Nothing cancels!

181 | P a g e
MATHEMATICS
Grade Nine (9)
𝟏+√𝟕
• Simplify: 𝟐−
√𝟕

You'll multiply by the conjugate in order to "simplify" this expression. The


denominator's multiplication results in a whole number (okay, a negative,
but the point is that there aren't any radicals):

The numerator's multiplication looks like this:

Then the simplified (rationalized) form is:

182 | P a g e
MATHEMATICS
Grade Nine (9)

Quiz
A. Simplify the following.
√42 √6
1. 2.
√6 √18

√20 √2
3. 4.
√6 √7

2 √2 √27
5. 6.
2√27 √𝑏3

√5𝑥 5 1
7. 8.
√4𝑦 5 7− √6

√5 2√3+ √7
9. 10.
√5− √15 √6

B. Perform the indicated operation and simplify answers whenever possible.


12√5+ 3√10 √6
1. 6.
√5 √2+ √6

−2√6+ 3√6 2√𝑎3 𝑏3


2. 7.
√3 √𝑎2 𝑏2

8 3√4+ 2 3√16 10√4𝑎2 𝑏2


3. 3 8.
2 √2 √2
4
√𝑥 4 3 √64𝑎8
4. 9. 4
√𝑥 6 √8𝑎2

2√𝑥 2 2 3√8
5. 10. 3
√𝑥 √−81

183 | P a g e
MATHEMATICS
Grade Nine (9)

Lesson 35
Solving Radical Equations
Equations containing radicals with variables in the radicand are called
radical equations. To solve radical equations, square both sides of the equation
to eliminate the radicals.
Examples:
1. Solve the equation: Checking:

x = 5:

x –1=(x –7)(x –7)


x – 1 = x2 – 14x + 49
x –1= x2 –14x +49
So, the answer is x = 10.
0= x2 –15x +50
0=(x –5)(x –10)
x = 5, x = 10
2. Solve the equation: Checking:

Since this equation is in the form "(square


root) = (number)", You can proceed
directly to squaring both sides: So the solution is x = 27.

x –2=25
x = 27

184 | P a g e
MATHEMATICS
Grade Nine (9)
3. Solve the equation: Checking,

Now squaring both sides will work better:


So, the solution is x = 49.

4. Find the solution: ...and here's the check:

Since the solution works in


the original equation, then
the solution is valid, and the
answer is: x = 4

185 | P a g e
MATHEMATICS
Grade Nine (9)

Quiz
A. Solve.
1. √𝑥 + 2 = 1
2. 4 + √𝑥 = 9
3. √𝑥 − 2 = 23
4. √𝑥 − 49 = 0
5. √3𝑚 = −9
6. √3𝑥 = 3
7. 4 − √3𝑎 = 1
8. 9 =√15𝑥 + 42 − 3
9. √6𝑎 + 1 − 8 = −3
2𝑥+3
10. √ =1
5

B. Solve each of the following radical equations and check your answers.
1. √𝑥 + 2 = 5
2. √𝑥 − 1 = 3
3. √𝑥 + 4 = 8 − 𝑥
4. √4𝑥 − 3 = 𝑥
5. 5 = √4𝑥 + 9
6. √𝑦 + 2 + 4 = 12

7. √5𝑥 + 9 − 3 = 2𝑥
8. √𝑦 + 2 + 1 = √3𝑦 + 3

9. √𝑥 2 - 1 = 7
10. 4√𝑥 = 2

186 | P a g e
MATHEMATICS
Grade Nine (9)

Lesson 36
Problem Solving Involving Radical
Equations
Since we already know how to simplify and apply the four fundamental
operations in radicals, we are now ready to apply radicals in problem solving.
Examples:
1. One half the square root of a number is 6. Find the number.
Solution: Let x = the number
1
Writing the equation, we have 2 √𝑥 = 6
1
2 ( √𝑥) = 2(6)
2
√𝑥 = 12
2
(√𝑥) = 22
x = 144
2. The square root of 5 more than twice a number is 7. Find the number.
Solution: Let x = the number
2
Write the equation (√5 + 2𝑥) = 72
5 + 2x = 49
2x = 49 – 5
2x = 44
x = 22

187 | P a g e
MATHEMATICS
Grade Nine (9)

Quiz
Solve the following problems.

1. Three times the square root of a number is 27. Find the numbers.
2. Twice the square of a number is 14. Find the number.
3. If 7 is subtracted from the square root of a number, the result is 45. Find the
number.
4. One-fourth of the square of a number is 2. Find the number.
5. The square root of a number when divided by 5 is equal to 2. Find the number.
6. Find the perimeter of a triangle if its sides measure 4 + √3 , 3 + √12, 2 + 3√3.
7. One third of the square root of a number is 3. Find the number.
8. The square root of the difference between a numbers and 5, plus the square
root of the number is equal to 5. Find the number.
9. The square root of the sum of a number and 5 is 3. Find the number.
10. Show that 3 + √5 and 3 - √5 satisfy the quadratic equation x2 – 6x + 4 = 0 by
substituting each number for x

188 | P a g e
MATHEMATICS
Grade Nine (9)

Lesson 37
Introduction and Some Special
Matrices
In this lesson, we are going to look at matrices, a subject that has wide
applications in the fields of pure mathematics, statistics, engineering and the
physical and social sciences.
Let us consider the case of a class library. The librarian will certainly keep a
record of all the types of books available in the library. After a lending session,
the record may show data as in table.
Language/Type Fiction Textbooks General Reference
Malay 25 47 22 30
Chinese 40 72 38 40
English 80 85 67 54
The table shows that of all the fiction books left in the library, 25 are in Malay,
40 in Chinese and 80 in English. Of all the English books available, 80 are fiction,
85 are textbooks, 67 are general and 54 are reference books.
The numbers in the table can be arranged in a rectangular array
4 columns
25 47 22 30
3 rows ( 40 72 38 40 )
80 85 67 54

189 | P a g e
MATHEMATICS
Grade Nine (9)
Here, the first column represents the number of books that are fiction while
the first row represents the number of books in Malay, and so on. A set of numbers
arranged in this manner is called a matrix (plural- matrices). The numbers within
a matrix are known as its elements. In the matrix above, there are 3 rows across
and 4 columns down. Thus, we say that the matrix has an order of 3 by 4 (also
written as 3 x 4), or it is a 3 x 4.
In general, an m x n matrix refers to a matrix having m rows across and n
columns down. Its order is hence “m by n” or “m x n”.

Row Matrix
A matrix with exactly one row is called a row matrix. Examples of row
matrices are:
a. (2 5)
b. (5 7 -2)
c. (-3 4 5 2)
The order of a row matrix is always 1 x n, where n is the number of columns
in the matrix. A row matrix is also referred to as a row vector.
Column Matrix
A matrix with exactly one column is called a column matrix. Examples of
column matrices are:
2
a. ( )
3
2
b. (−3)
4
1
0.8
c. ( )
8.7
−6

190 | P a g e
MATHEMATICS
Grade Nine (9)
The order of a column matrix is always m x 1, where m is the number of rows
in the matrix. A column matrix is also referred to as a column vector
Square Matrix
If a matrix has exactly the same number of rows and columns, m = n, it is
called a square matrix. A matrix with n rows across and n columns downs is
written as a n x n matrix. The following are examples of square matrices.
1 0
a. ( )
0 1
b. (8)
3 0 0 0
0 4 0 0
c. ( )
0 0 8 0
0 0 0 1
1 6 7
d. (8 − 6 9)
9 0 4
Notice that in the above matrix (c), all elements except those in the leading
diagonal are zeros. Such a matrix is called a diagonal matrix. The matrix a
and b are also diagonal matrices

Zero Matrix or Null Matrix


If every element of a matrix A is equal to zero, A is called a zero matrix or
null matrix. A zero matrix is usually denoted by 0, it may be any order. The
following are some example of zero matrices.
0 0
a. ( )
0 0
b. (0 0)

Equal Matrices

If A and B are two matrices of the same order and their corresponding
elements are equal, then A is equal to B.
Conversely, if A = B, then
a. A and B are of same order
b. their corresponding elements are equal
191 | P a g e
MATHEMATICS
Grade Nine (9)
20 𝑏 4𝑎 𝑎+3
If A = ( ), B = ( ) and A = B, find the values of a, b, c, and d.
𝑐 16 𝑑−4 𝑑

Solution
20 𝑏 4𝑎 𝑎+3
( )=( )
𝑐 16 𝑑−4 𝑑
20 = 4a b=a+3 c=d–4 d = 16
20
a= 4 b = 5+ 3 c = 16 - 4
a=5 b=8 c = 12

Quiz
A. State the order of each of B. Which of the following pairs of matrices
the following matrices. are equal? If they are not equal state the
3
1. ( ) reason.
1 2 3
2 5 1. A = ( ) B=( )
2. ( ) 3 2
3 −5
2 5 6
3. ( ) 2. L = (2 5 − 4) B = (2 5 − 4)
8 −4 9
4. (1 3)
2 3 5 7 0 0 0
3. P = ( ) Q = (0)
5. (4 − 6 3 5) 0 0 0
1 3 2 −6
6. (2 4 5 7 8) 4. P = (
3 4
) Q=(
3 4
)
7. (1 −5 0 9) 7 −5 7 −5
4 6 −8
1 3 −9 3
8. 5. A = ( −5) B = (3 − 5 6)
5 −5 6
1 −6 −7 6
(6 7 − 9)
2.4
3.4
9. ( )
−5.4
−6.7
−3 9 9 9
10. ( )
9 −8 8 8

192 | P a g e
MATHEMATICS
Grade Nine (9)
C. State which of the following matrices are equal.
5
A = (5 7) B=( ) C = (7 5)
7
7 3 7 −4 5
D=( ) E= ( ) F=( )
5 5 −4 7 3
3 5 3 7 7
G=( ) H=( ) I=( )
7 −4 5 −4 5
3 2 3
J = (3 2 7) K = (2) L = (4 5)
7 7 8
7
3 5
M = (2 3 4) N=( ) O = ( 2)
7 −4
3
2 3 4
P = (3 2 7) Q = (7 5) R=( )
5 7 8
−4 5 5 1
S=( ) T=( ) U=( )
7 3 7 5

D. Find the value of the unknowns in each of the following.


2 3 2𝑎 𝑏
1. ( )= ( )
5 𝑘 𝑐 7
2𝑥 18 14 2𝑘
2. ( )= ( )
3𝑦 36 15 6ℎ
1
𝑥 𝑥+4 3 ℎ
3. (2 )= ( )
5 3𝑦 𝑘−9 27
7 5 𝑏 𝑎 5 13
4. ( )= ( )
7 −3 𝑐 𝑑 −𝑎 6
5. (2𝑥 − 3 𝑦 + 4 ) = (7 6)
2𝑥 − 5 𝑦−4
6. ( ) = (0)
𝑧+3 5𝑘

193 | P a g e
MATHEMATICS
Grade Nine (9)

Lesson 38
Addition and Subtraction of Matrices
If P and Q are two matrices of the same order, the sum of P and Q denoted
P + Q, is obtained by adding the corresponding elements of P and Q. The
difference of P and Q, denoted by P – Q, is obtained by subtracting the
corresponding elements of Q from P.
Thus, it follows that the matrices P + Q and P – Q have the same orders.
For example
𝑏𝑎 𝑝 𝑞
If P = ( ) and Q = ( )
𝑑 𝑐 𝑟 𝑠
𝑎+𝑝 𝑏+𝑞 𝑎−𝑝 𝑏−𝑞
P+Q=( ) and P – Q = ( )
𝑐+𝑟 𝑑+𝑠 𝑐−𝑟 𝑑−𝑠
Example 1
3 7
2 3 −3 5 2 5 7
If A = ( ), B = ( ), C=( ) and D = ( 9 − 8)
5 7 −6 1 −3 4 6
4 2
Evaluate each of the following where possible.
a. A + B b. B – A
c. B + C d. C + 0
e. D + C f. 0 + D
Solution
2 3 −3 5 2 + (−3) 3+5 −1 8
a. A + B = ( )+( )=( )=( )
5 7 −6 1 5 + (−6) 7+1 −1 8
−3 5 2 3 −3 − 2 5−3 −5 2
b. B – A = ( )− ( )= ( )=( )
−6 1 5 7 −6 − 5 1−7 −11 −6

194 | P a g e
MATHEMATICS
Grade Nine (9)
c. B is of order 2 x 2 while C is of order 2 x 3. Since the order of Band C are
different, B + C does not exist. B + C is not defined.
2 5 7 0 0 0 2 5 7
d. C + 0 = ( )+( )=( )
−3 4 6 0 0 0 −3 4 6
e. D is of order 3 x 2 and C is of order 2 x 3. Since the order of D and C are not
the same, D + C is not defined.
0 0 3 7 3 7
f. 0 + D = ( 0 −0 ) + ( 9 8 ) = ( 9 8)
0 0 4 2 4 2
Example 2
Solve the following matrix
3 5 13 8
a. A + ( )=( )
−4 6 6 3
3 −2 4 2 −4 9
b. ( )-B = ( )
7 6 −5 6 −3 4
Solution
a. In this question, the order A has to be 2 x 2
3 5 13 8 13 8 3 5
A+( )=( ) A=( )− ( )
−4 6 6 3 6 3 −4 6
13 − 3 8−5 10 3
A=( ) A=( )
6 − (−4) 3−6 10 −3
b. The order of B has to be 2 x 3
3 −2 4 2 −4 9
( )-B = ( )
7 6 −5 6 −3 4
3 −2 4 2 −4 9
B=( )− ( )
7 6 −5 6 −3 4
3−2 − 2 − (−4) 4 − 9 1 2 −5
B=( ) B=( )
7−6 6 − (−3) −5−4 1 9 −9

195 | P a g e
MATHEMATICS
Grade Nine (9)

Quiz
A. Evaluate each of the following.
2 4 3 1
1. ( )+ ( )
7 −4 2 0
2. (2 5) + (6 − 7)
3 5 7 3 −4 7 9 7
3. ( )+ ( )
−2 1 6 5 5 −3 −4 3
−4 7 4 −3
4. ( ) - ( )
−5 3 2 4
1 3
5. 0 − 4)
( ) (
2 5
3 −2
6. ( ) + ( )
4 5
2 3 7 −3 5 4
7. ( )+( )
9 −4 6 −4 9 −2
3 4
8. ( ) − ( )
9 2
−5 3 7 3 −5 4
9. ( )-( )
−4 2 −4 −6 3 9
3 2 2 7
10. ( 1 5 ) – ( −5 4)
−4 6 −3 − 1

B. Evaluate each of the following.


3 2 5 4 6 3
1. ( )+ ( )− ( )
4 −1 3 2 1 −2

2 3 −4 3 1 5 1 0 0
2. ( )+ ( )+( )
6 −1 3 −3 2 7 0 1 1

2 3 4 5 −3 4
3. (4 − 7) – ( −2 7 ) + ( −1 7 )
5 −3 6 −1 −6 2

3 4 5
4. ( ) − ( ) + ( )
5 7 −3

5. (1 3) − (3 2 ) + (6 5)

196 | P a g e
MATHEMATICS
Grade Nine (9)
C. Solve the following matrix equations.

3 2 7 5
1. A + ( )= ( )
1 −4 −3 4

1 −4 6 7
2. A – ( )=( )
5 3 −4 3

5 4 5 −2
3. ( )- A =( )
7 −1 0 7
2 −4 3 3 1 4
4. ( )+ 𝐴 = ( )
6 1 0 −3 5 −2
1 4 5 3 5 −6
5. A - ( )=( )
−3 0 6 0 −1 4

6. (1 3 −4 6) + A = (3 0 −4 9)
2 −5
7. A + ( ) = ( )
−1 6
𝑥 6𝑥
8. (3𝑦) - A = ( )
−𝑦

4 4 1 6
9. A + ( )= ( )
2 −4 −3 −4

2 −5 3 6
10. A – ( )=( )
4 3 −5 4

𝟓 −𝟓 𝟏 𝟑 𝟎 𝟐
D. If A = ( ), B = ( ) and C = ( )
−𝟒 𝟗 −𝟐 𝟒 −𝟏 𝟒
Find the value of each of the following.
1. A + B
2. B + A
3. B + C
4. C +
5. A + (B + C)

197 | P a g e
MATHEMATICS
Grade Nine (9)

Lesson 39
Multiplication of a Matrix by a Real
Number
7 4 6 5
Let A = ( 18 7 2 7 ) represent the number of different types of books
15 25 10 8
borrowed by the students at the end of the week from the class library
mentioned earlier. If the students now borrow twice as many of each type as
they did previously, then the number of books borrowed will be represented by
B, where:
2𝑥7 2𝑥4 2𝑥6 2𝑥5
B = ( 2 𝑥 18 2𝑥7 2𝑥2 2𝑥7)
2 𝑥 15 2 𝑥 25 2 𝑥 10 2𝑥8
14 8 12 10
= ( 36 14 4 14 )
30 50 20 16
It is seen that every element of B is twice the corresponding element of A.
Therefore, we write B = 2A.
3𝑥7 3𝑥4 3𝑥6 3𝑥5
Similarly, if B = 3A, then ( 3 𝑥 18 3𝑥7 3𝑥2 3𝑥7)
3 𝑥 15 3 𝑥 25 3 𝑥 10 3𝑥8
In general, if k is any real number, we have
7𝑘 4𝑘 6𝑘 5𝑘
kA = ( 18𝑘 7𝑘 2𝑘 7𝑘 )
15𝑘 25𝑘 10𝑘 8𝑘

198 | P a g e
MATHEMATICS
Grade Nine (9)
Example 1
3 −2 −1 3
If A = ( ) and B = ( ), evaluate
1 4 6 −5
a. 3A + 2B
b. 4B – 2A
Solution
3 −2 −1 3
a. 3A + 2B = 3( ) + 2( )
1 4 6 −5
9 −6 −2 6
=( ) +( )
3 12 12 − 10
7 0
=( )
15 2
−1 3 3 −2
b. 4B – 2A = 4( ) - 2( )
6 −5 1 4
−4 12 6 −4
=( ) - ( )
24 − 20 2 8
−10 16
=( )
22 − 28
Example 2
Solve the following matrix equations.
a. 2A = (6 8 − 4)
3 𝑥 0 4 2 5
b. 2 ( )+ ( ) = 3( )
0 𝑦 6 −2 2 14
2 5 4 3
c. 4
( ) (
− 3 + 2𝑎 = 8 7)
6 −1 4 −5

199 | P a g e
MATHEMATICS
Grade Nine (9)
Solution
a. 2A = (6 8 − 4)
1
Multiply both sides by 2
1 1
(2A) = 2 (6 8 − 4)
2

A = (3 4 − 2)
3 𝑥 0 4 2 5
b. 2 ( )+ ( ) = 3( )
0 𝑦 6 −2 2 14
6 2𝑥 0 4 6 15
( )+ ( )= ( )
0 2𝑦 6 −2 6 42
6 2𝑥 + 4 6 15
( )= ( )
6 2𝑦 − 2 6 42
2x + 4 = 15 and 2y – 2 = 42
2x = 11 2y = 44
11
x= y = 22
2

2 5 4 3
c. ( 4 − 3 ) + 2𝑎 = ( 8 7)
6 −1 4 −5
4 3 2 5
2𝐴 = ( 8 7) − ( 4 −3 )
4 −5 6 −1
2 −2
2𝐴 = ( 4 10 )
−2 −4
2 −2
1
𝐴 = 2( 4 10 )
−2 −4
1 −1
𝐴=( 2 5 )
−1 −2

200 | P a g e
MATHEMATICS
Grade Nine (9)

Quiz
A. Simplify each of the following.

1. 3(1 5) 2. 2(1 −2 3)
6
−2 1
3. 4 ( ) 4. 2 ( 4 )
1
−8
1 6 15 −1 0.5 3
5. ( ) 6. −2 ( )
3 21 − 24 −0.8 2 1.2
1 5
0 4 1 −1 3 0
7. 4 ( −4 3) 8. 3 ( )− 4( )
5 0 −1 −2 1 −1
−1 2
3 −1 −4 −1
9. 4 ( 4 5 ) − 3( 0 2 )
−3 −2 −1 4
1 2 4 1 0 4 −2 4 −3
10. 2 ( ) + 3( )− 5( )
−3 4 −2 −3 5 −1 0 6 −7

B. In each of the following cases, find the matrix A which satisfies the given
relationship.
3 −4 1 2 3
1. 𝐴 + ( ) = ( ) 2. 3𝐴 = ( )
4 3 4 5 6
1 2 1 7
1 2 1
3. (3 4) − 𝐴 = ( 4 − 5) 4. 𝐴 − ( )= 𝐴
3 4 2
5 6 3 −4
C. Solve the following equations.
4 4 12 2 2 0
1. b(( ) + 𝑐 ( ) = ( ) 2. a( ) + 𝑏 ( ) = ( )
4 −4 4 2 −2 8
2𝑥 𝑥 18 1 2 3 5 𝑎 𝑏
3. 3( ) + 3 (3𝑦) = ( ) 4. 2( )− ( )= ( )
𝑦 36 3 4 𝑐 6 7 𝑑
5 3 2 𝑎 𝑏 𝑐 9 12 6
5. 2( )+ ( )= ( )
1 6 3 −2 −4 5 𝑑 𝑒 𝑓

201 | P a g e
MATHEMATICS
Grade Nine (9)
𝟒 𝟒 𝟏 𝟐 𝟏 𝟒
D. If A = ( ) ,𝑩 = ( ) and C = ( ), find the following.
𝟐 𝟕 −𝟏 𝟑 𝟑 −𝟓
1. A + B 2. A – B – C 3. A + 2B
4. 2A – 2C + 3B 5. 3A – 2B + 4C

E. Solve.
A music store selling VCD and CD has two branches. The number of VCDs
and CDs sold by this music store at its branches per week is given by matrix A.
The new arrival of VCDs and CDs in the two branches per week is given by the
matrix B. The number of VCDs and CDs in stock at the beginning of January is
represented by the matrix C.

a. Evaluate C + 3B – 4A b. Explain what the answer in (a) represents.

202 | P a g e
MATHEMATICS
Grade Nine (9)

Lesson 40
Multiplication of Matrices
Ben bought 3 pencils, 2 ball-point pens and 4 exercise books. The purchase
made can be represented by the matrix (3 2 4). The cost of each item is P15,
15
P25 and P40, respectively, which can be represented by the matrix (25)
40
The total cost of the purchase is 3 x 15 + 2 x 25 + 4 x 40 = P255
In matrix form. we write
15
(3 2 4) (25) = (3 x 15 + 2 x 25 + 4 x 40)
40
= 225
showing that the total cost is obtained by adding the product of the first element
of the row and the first element of the column, the second element of the row
and the second element of the column, and the third element of the row and
the third element of the column.

John bought 4 pencils, 5 ball-point pens and 2 exercise books while Alvin bought
2 pencils, 5 ball-point pens and 5 exercise books. The cost of their purchase may
be represented as follows:
John 4 x 15 + 5 x 25 + 2 x 40 = 265
Alvin 2 x 15 + 4 x 25 + 5 x 40 = 330

203 | P a g e
MATHEMATICS
Grade Nine (9)
In matrix form, the purchase made by 3 boys can be written as:
3 2 4 15 3 𝑥 15 + 2 𝑥 25 + 4 𝑥 40 255
(4 5 2 ) (25) = (4 𝑥 15 + 5 𝑥 25 + 2 𝑥 40) = (265)
2 4 5 40 2 𝑥 15 + 4 𝑥 25 + 5 𝑥 40 330
One month later, Ben, John and Alvin made the same purchases but at slightly
20
higher prices given by this matrix (35). How much must they pay pen? The cost
45
is determined by the same method:
3 2 4 20 3 𝑥 20 + 2 𝑥 35 + 4 𝑥 45 310
(4 5 2 ) (35) = (4 𝑥 20 + 5 𝑥 35 + 2 𝑥 45) = (345)
2 4 5 45 2 𝑥 20 + 4 𝑥 35 + 5 𝑥 45 405

The above results may be represented as a product of two matrices:

The example above shows how two matrices are multiplied to get the cost
matrix. This process is called matrix multiplication. Note that matrix multiplication
is possible only when the number of columns in the left matrix is equal to the
number of rows in the right matrix. The order of the resulting matrix is given by the
number of rows in the left matrix and the number of columns in the right matrix.

204 | P a g e
MATHEMATICS
Grade Nine (9)
For example
A 3 x 3 matrix multiply by a 3 x 2 matrix will result in a 3 x 2 matrix.
A 3 x 2 matrix multiply by a 2 x 3 matrix will result in a 3 x 3 matrix.
A 3 x 2 matrix multiply by a 3 x 3 matrix is not possible.

In general,
by multiplying a m x n matrix by a n x p matrix, the result is a m x p matrix.

A matrix product is defined only if the number of columns of the first matrix is
equal to the number of rows in the second matrix.
Example 1
3 2 2 1 0
Find the matrix product of ( ) 𝑎𝑛𝑑 ( )
1 7 4 6 3
Solution
The orders of the matrices are 2 x 2 and 2 x 3, respectively. Since the number
of columns in the first matrix is equal to the number of rows in the second matrix,
matrix multiplication is possible.
The order of the matrix product is therefore 2 x 3.
Using the multiplication procedure, we have

(3 𝑥 2) + (2 𝑥 4) (3 𝑥 1) + (2 𝑥 6) (3 𝑥 0) + (2 𝑥 3)
=( )
(1 𝑥 2) + (7 𝑥 4) (1 𝑥 1) + (7 𝑥 6) (1 𝑥 0) + (7 𝑥 3)
14 15 6
=( )
30 43 21

205 | P a g e
MATHEMATICS
Grade Nine (9)
Example 2
1 3 1 0 1 2 3
If A = ( ), B = ( ) and C = ( ), find the products AB, BC and
4 7 0 1 4 5 6
CB where possible.
Solution
1 3 1 0 (1 𝑥 1) + (3 𝑥 0) (1 𝑥 0) + (3 𝑥 1) 1 3
AB =( )( )= ( ) =( )
4 7 0 1 (4 𝑥 1) + (7 𝑥 0) (4 𝑥 0) + (7 𝑥 1) 4 7
For the product BA
1 0 1 2 3 (1 𝑥 1) + (0 𝑥 4) (1 𝑥 2) + (0 𝑥 5) (1 𝑥 3) + (0 𝑥 6)
BC=( )( )= ( )
0 1 4 5 6 (0 𝑥 1) + (1 𝑥 4) (0 𝑥 2) + (1 𝑥 5) (0 𝑥 3) + (1 𝑥 6)
1 2 3
= ( )
4 5 6
For the product of CB: The order of C is 2 x 3 and that of B is 2 x 2. Since the
number of columns of C is not equal to the number of rows of B, the product CB
is not defined.

Quiz
A. Find the matrix products of the following where possible.
2 4 1
1. ( )( )
1 3 7
1 1 4
2. ( ) ( )
7 3 7
2 3 4 1 2
3. ( )( )
1 7 6 3 4
1 0 3 0 4
4. ( )( )
2 0 3 1 2
4 3
2 4
5. ( 1 5) ( )
1 7
3 7

206 | P a g e
MATHEMATICS
Grade Nine (9)
1
3 2 5 1 2
6. ( 0 2 3 0) 3
1 4 7 0 4
( )
2 1 3 2
7. ( )( )
1 2 2 3
3 2 2 1
8. ( )( )
2 3 1 2
7 5 1 0
9. ( )( )
6 4 0 1
1 0 0 0
10. ( )( )
2 0 3 4

207 | P a g e
MATHEMATICS
Grade Nine (9)

Lesson 41
Properties of Parallelograms
Quadrilateral is a polygon with four sides.
A parallelogram is a quadrilateral with both pairs of opposite sides parallel.

When given a Parallelogram, the definition and theorem are stated…


A parallelogram is a quadrilateral
with both pairs of opposite sides
parallel

If a quadrilateral is parallelogram,
the 2 pairs of opposite sides are
congruent.

If a quadrilateral is parallelogram,
the 2 pairs of opposite angles are
congruent.

If a quadrilateral is parallelogram,
the consecutive angles are
supplementary.

208 | P a g e
MATHEMATICS
Grade Nine (9)
If a quadrilateral is parallelogram,
the diagonals bisect each other.

If a quadrilateral is parallelogram,
the diagonal form two congruent
triangles.

Proof of Theorem: If a quadrilateral is a parallelogram, the 2 pairs of opposite


sides are congruent.

STATEMENTS REASONS
1 Given
2 Draw segment Two points determine exactly one
from A to C line.
3 A parallelogram is a quadrilateral
with both pairs of opposite sides
parallel.
4 If two parallel lines are cut by a
transversal, the alternate interior
angles are congruent.

209 | P a g e
MATHEMATICS
Grade Nine (9)
5 Reflexive property: A quantity is
congruent to itself.
6 ASA: If two angles and the
included side of one triangle are
congruent to the corresponding
parts of another triangle, the
triangles are congruent.
7 CPCTC: Corresponding parts of
congruent triangles are congruent.
Proof of Theorem: If one pair of opposite sides of a quadrilateral are both and
parallel and congruent, the quadrilateral is a parallelogram

STATEMENTS REASONS
1 Given

2 Draw segment Two points determine exactly one


from A to C line.
3 If two parallel lines are cut by a
transversal, the alternate interior
angles are congruent.
4 Reflexive property: A quantity is
congruent to itself.

210 | P a g e
MATHEMATICS
Grade Nine (9)
5 SAS: If two sides and the included
angle of one triangle are
congruent to the corresponding
parts of another triangle, the
triangles are congruent.
6 CPCTC: Corresponding parts of
congruent triangles are congruent.
7 If two lines are cut by a transversal
and the alternate interior angles
are congruent, the lines are
parallel.
8 A parallelogram is a quadrilateral
with both pairs of opposite sides
parallel.

Quiz
A. State whether each statement is true or false.
1. A parallelogram is a quadrilateral with both pairs of opposite sides are
parallel.
2. The sides of a quadrilateral are segments.
3. Each diagonal of a parallelogram divides the parallelogram into three
congruent triangles.
4. Opposite angles of a parallelogram are congruent.
5. Any two consecutive angles of a parallelogram are complementary.
6. The diagonals of a parallelogram bisect each other.
7. Opposite angles of a parallelogram are congruent.

211 | P a g e
MATHEMATICS
Grade Nine (9)
B. Refer to the parallelogram RSPV. Complete each statement.

1. RP II 6. ROS ≅
2. PV ≅ 7. m RPV + m PVS =

3. RPV ≅ 8. SRV ≅
4. SO ≅ 9. PRS ≅

5. 2VO = 10. 2SO =

212 | P a g e
MATHEMATICS
Grade Nine (9)

Lesson 42
Finding the Measures of Angles, sides
and other quantities involving
Parallelogram
Given a parallelogram ABCD. Complete each statement along with the
definition or property used.

213 | P a g e
MATHEMATICS
Grade Nine (9)
2. Consider the following parallelograms. Find the values of the unknowns x, y, z.

(i)

(ii)
Answer: x, y and z will be complementary to 50°.
So, required angle = 180° - 50° = 130°

(iii)
Answer: z being opposite angle= 80°
x and y are complementary, x and y
= 180° - 80° = 100°

214 | P a g e
MATHEMATICS
Grade Nine (9)

(iv)
Answer: As angles on one side of a line are always complementary
So, x = 90°
So, y = 180° - (90° + 30°) = 60°
The top vertex angle of the above figure = 60° x 2=120°
Hence, bottom vertex Angle = 120° and z = 60°

(v)
Answer: y= 112°, as opposite angles are equal in a parallelogram
x= 180°-(112°-40°)=28°
As adjacent angles are complementary so angle of the bottom left vertex =180°-
112°=68°
So, z=68°-40°=28°
Another way of solving this is as follows:
As angles x and z are alternate angles of a transversal so they are equal in
measurement.
3. Can a quadrilateral ABCD be a parallelogram if

215 | P a g e
MATHEMATICS
Grade Nine (9)
Answer:
(i) It can be, but not always as you need to look for other criteria as well.
(ii) In a parallelogram opposite sides are always equal, here AD BC, so it’s not a
parallelogram.
(iii) Here opposite angles are not equal, so it is not a parallelogram.
5. The measures of two adjacent angles of a parallelogram are in the ratio 3:2.
Find the measure of each of the angles of the parallelogram.
Answer: Opposite angles of a parallelogram are always add up to 180°.

6. Two adjacent angles of a parallelogram have equal measure. Find the


measure of each of the angles of the parallelogram.
Answer: 90°, as they add up to 180°
7. The adjacent figure HOPE is a parallelogram. Find the angle measures x, y and
z. State the properties you use to find them.

Answer: Angle opposite to y = 180° - 70°=110°


Hence, y = 110°
x = 180° - (110° + 40°) = 30°, (triangle’s angle sum)
z = 30° (Alternate angle of a transversal)

216 | P a g e
MATHEMATICS
Grade Nine (9)
8. The following figures GUNS and RUNS are parallelograms. Find x and y. (Lengths
are in cm)

Answer: As opposite sides are equal in a parallelogram

Answer: As you know diagonals bisect each other in a parallelogram.

9. In the given figure both RISK and CLUE are parallelograms. Find the value of x.

217 | P a g e
MATHEMATICS
Grade Nine (9)

Answer: In parallelogram RISK

NEWS is a parallelogram.
1. If NE = 12, and find SW.
2. If m NSW = 105º, find m NEW
3. If m NSE = 65º and m 60º, find m SWE
4. NS = 5 – 3x and EW = 2x + 5. Find the value of x
5. m SNE = 7x – 32 and m EWS = 3x + 40. Find x
Solution
1. Opposite sides of a parallelogram are congruent. Hence, SW = NE = 12.
2. Opposite angles of a parallelogram are congruent. Hence,
m NEW = m NSW = 105º

218 | P a g e
MATHEMATICS
Grade Nine (9)
3. Consecutive angles are supplementary
m NSW + m SWE = 180º
But m NSW = m NSE + m ESW = 65º + 60º = 125º
Therefore, m SWE = 180º - m NSW
= 180º - 125º
= 55º
4. Opposite sides of a parallelogram are congruent. Therefore,
NS = EW
5 – 3x = 2x + 15
-3x – 2x = 15 – 5
-5x = 10
x = -2
5. Opposite angles of a parallelogram are congruent. Therefore,
m SNE = m EWS
7x – 32 = 3x + 40
7x – 3x = 40 + 32
4x = 72
x = 18

Quiz
A. Find the measures of the numbered angles in each parallelogram.
1. 2.

219 | P a g e
MATHEMATICS
Grade Nine (9)
B. Given: LOAD is a parallelogram with m<L = 2x and m<D = (3x + 15).Find
the measures of the following angles.

1. m<L 3. m<A
2. m<D 4. m<O

C. Find the value of x in the parallelogram ABCD. Find the length of each side.

1. x 4. AD
2. AB 5. BC
3. DC

D. Find the value of each variable.

220 | P a g e
MATHEMATICS
Grade Nine (9)
E. Use the figure below to find the value of x with the given information.

ZEAL is a parallelogram
1. ZE = 9x – 2 and LA = 3x + 6
2. YL = 6x and LE = 4x + 8
3. ZY = 7x – 5 and YA = 4x + 16
4. ZA = 18 – x and ZY = 3x – 5
5. m EZA = 2x + 10, m AZL = x + 5 and M EAL = 75º
6. m LZE = 6x + 32 and m ZEA = 68 + 4x
7. m ZEL = 10x + 20 and m ELA = 8x + 26.

F. Refer to parallelogram TAPE. Find each measure.

1. EP 5. TN
2. TE 6. m TEP
3. m 1 7. m ATE
4. m 2

221 | P a g e
MATHEMATICS
Grade Nine (9)

Lesson 43
Properties of Rhombuses,
Rectangles, and Squares
The three special parallelograms — rhombus, rectangle, and square — are so-
called because they’re special cases of the parallelogram. (In addition, the
square is a special case or type of both the rectangle and the rhombus.)

The three-level hierarchy you see with

in the above quadrilateral family tree works just like

222 | P a g e
MATHEMATICS
Grade Nine (9)
A dog is a special type of a mammal, and a Dalmatian is a special type of a
dog.

Here are the properties of the rhombus, rectangle, and square. Note that
because these three quadrilaterals are all parallelograms, their properties
include the parallelogram properties.
• The rhombus has the following properties:
• All the properties of a parallelogram apply (the ones that matter here are
parallel sides, opposite angles are congruent, and consecutive angles are
supplementary).
• All sides are congruent by definition.
• The diagonals bisect the angles.
• The diagonals are perpendicular bisectors of each other.
The rectangle has the following properties:
• All the properties of a parallelogram apply (the ones that matter here are
parallel sides, opposite sides are congruent, and diagonals bisect each
other).
• All angles are right angles by definition.
• The diagonals are congruent.

223 | P a g e
MATHEMATICS
Grade Nine (9)
The square has the following properties:
• All the properties of a rhombus apply (the ones that matter here are
parallel sides, diagonals are perpendicular bisectors of each other, and
diagonals bisect the angles).
• All the properties of a rectangle apply (the only one that matters here is
diagonals are congruent).
• All sides are congruent by definition.
• All angles are right angles by definition.
Now try working through a problem. Given the rectangle as shown, find the
measures of angle 1 and angle 2:

Here’s the solution: MNPQ is a rectangle, so angle Q = 90°. Thus, because there
are 180° in a triangle, you can say

Now plug in 14 for all the x’s.

224 | P a g e
MATHEMATICS
Grade Nine (9)
Now find the perimeter of rhombus RHOM.

Here’s the solution: All the sides of a rhombus are congruent, so HO equals x+ 2.
And because the diagonals of a rhombus are perpendicular, triangle HBO is a
right triangle. You finish with the Pythagorean Theorem:

Combine like terms and set equal to zero:

Factor:
(x – 3)(x + 1) = 0
Use Zero Product Property:
x – 3 = 0 or x + 1 = 0
x = 3 or x = –1
You can reject x = –1 because that would result in triangle HBO having legs with
lengths of –1 and 0.

225 | P a g e
MATHEMATICS
Grade Nine (9)

Quiz
A. State whether each statement is true or false.
1. The diagonals of a rectangle are congruent.
2. In a rhombus, the diagonals are parallel.
3. The diagonals of a square bisect each other.
4. In a square, all sides are congruent.
5. Consecutive angles of square are both supplementary and congruent.
6. All angles of square are congruent.

B. Given: BEST is a rectangle. ST = 24, BT = 7 and BS = 25.


Find:
1. ES 2. BE
3. ET 4. m<BES
C. ABCD is a rhombus. Find the measures of the numbered angles in the
figure.

226 | P a g e
MATHEMATICS
Grade Nine (9)
D. Use rectangle JOYS

1. If TY = 5, find SO.
2. If SY = 14, find JO.
3. If m 3 = 58º, find m 1.
4. If m 2 = 132º, find m 5.
5. If JT = 3x + 1 and JY = 4x + 12, find OT.
6. If OT = 5x – 9 and JY = 4x + 12, find OT.
7. If JY = 2x2 and SO = 7x – 3, find x.
8. if m = 31º, find m 2.
E. Use rhombus to answer each question.

1. If m TCH = 68º, find m TCA.


2. If m TNA = 115º, find m HTA.
3. If m TAC = 32º, find m TAH.
4. If m 120º, find m HTA.
5. If m HAC = 3x – 15 and m HAT = x + 5, find x.

227 | P a g e
MATHEMATICS
Grade Nine (9)

Lesson 44
Midline Theorem
Proof of Midline Theorem
Statement Given Reasons
1. D and E are the
midpoint of and

2. EF = DE Point-Plotting Theorem
3. EB = EC Definition of Midpoint
Vertical Angle Theorem
4.
SAS Postulate
5.
CPCTC
6.
The AIP Theorem
7.
8. AD = DB Definition of Midpoint
9. DB = FC CPCT
10. AD = FC Transitive Property
If two sides of a quadrilateral are parallel and
11. congruent, then the quadrilateral is a
parallelogram
12. Definition of parallelogram
13. DF = DE + EF = 2DE Definition of Between, step 2 and APE
14. AC = 2DE In a parallelogram, any two opposite sides are
congruent
15. DE = ½ AC Multiplication Property of Equality

228 | P a g e
MATHEMATICS
Grade Nine (9)
Example:
Find the value of x in each figure.

Solution:
a. x = ½(12) b. 6 = ½(x – 1)
x=6 12 = x – 1
x = 13

Quiz
A. Using the marked figure below to find each indicated length.
1. If HI = 9, then PO = ?
2. If HI = 15, then PO = ?
3. If HI = 29, then PO = ?
4. If PO = 14, then HI = ?
5. If PO = 28, then HI = ?
6. If PO = 29, then HI = ?
7. If HI = 2x + 3 and PO = 5x + 4, then HI = ? and PO = ?
8. If HI = 2x + 1 and PO = 6x – 4, then HI = ? and PO = ?
9. If PO = 3x – 8 and HI = x + 4, then HI = ?
10. IF PO = 28 and HI = 2x + 6, then HI = ?

229 | P a g e
MATHEMATICS
Grade Nine (9)

B. In PQRS. points A, B, and C are the midpoints of PQ, PR and RQ, respectively.
Complete the following statements.

1. BC = 12, then QP =
2. PR = 40, then AC =
3. PQ = 40, then BC =
4. If BC = 7, then PQ =
5. If QR = 12, then AB =
6. If AC = 3x + 5 and PR = 4x + 16, then PR =
7. If AB = 7x – 4 and QR = 3x + 14, then QC =
8. If AB = 15 and AC = 19, then QR + PR =
9. If QA = x + 4 and PA = 2x – 5, then BC =
10. If QP = 20 and QR = 18, then BC + AB =

230 | P a g e
MATHEMATICS
Grade Nine (9)

Lesson 45
Trapezoid and Kites
Recall that a trapezoid is a quadrilateral with only one pair of opposite sides
parallel and that the parallel sides are called bases and the nonparallel sides are
called legs. If the legs of a trapezoid are equal, it is called an isosceles
trapezoid. Figure is an isosceles trapezoid.
Figure 1 An isosceles trapezoid.
A pair of angles that share the same
base are called base angles of the
trapezoid. In Figure 1, ∠ A and ∠ B or
∠ C and ∠ D are base angles of
trapezoid ABCD. Two special properties of
an isosceles trapezoid can be proven.
Theorem : Base angles of an isosceles trapezoid are equal.
Theorem : Diagonals of an isosceles trapezoid are equal.
In isosceles trapezoid ABCD (Figure 2) with bases AB and CD :
• By Theorem 53, m ∠ DAB = m ∠ CBA, and m ∠ ADC = m ∠ BCD.
• By Theorem 54, AC = BD.

231 | P a g e
MATHEMATICS
Grade Nine (9)
Figure 2 An isosceles trapezoid with its
diagonals.
Recall that the median of a trapezoid is a
segment that joins the midpoints of the
nonparallel sides.
Theorem : The median of any trapezoid has two properties: (1) It is parallel to both
bases. (2) Its length equals half the sum of the base lengths.
In trapezoid ABCD (Figure 3) with bases AB and CD , E the midpoint of AD ,
and F the midpoint of BC , by Theorem 55:
Figure 3 A trapezoid with its median.

Example 1: In Figure 4, find m ∠ ABC and


find BD.
Figure 4 An isosceles trapezoid with a
specified angle and a specified diagonal.
m ∠ ABC = 120°, because the base angles
of an isosceles trapezoid are equal.
BD = 8, because diagonals of an isosceles
trapezoid are equal.

232 | P a g e
MATHEMATICS
Grade Nine (9)
Example 2: In Figure 5, find TU.
Figure 5 A trapezoid with its two bases
given and the median to be computed.
Because the median of a trapezoid is half
the sum of the lengths of the bases:

Kite is a quadrilateral with distinct pairs of


congruent and adjacent sides.
Shown above is KITE with KI ≅ KE and IT
≅ TE. The vertex angles of kite are <K and <T
while the nonvertex angles are <I and <E.
The nonvertex angles of a kite are congruent.
The diagonals of a kite are perpendicular
The diagonal connecting the vertices of the vertex angles of a kite is the
perpendicular bisector of the other diagonal.
The vertex angles of a kite are bisected by its diagonal.
The area of a kite is equal to half the product of its diagonals. That is if d1 and
1
d2 are diagonals of a kite, then area of kite = 2d1d2

233 | P a g e
MATHEMATICS
Grade Nine (9)

Quiz
A. Find the measures of each angle of the given figure.

B. Find the measures of the median of the given figure.


Given : AB = 24 cm

Find:
1. CD 5. KL
2. EF 6. MN
3. GH 7. OP
4. IJ
C. Find the value of x in each isosceles trapezoid.

234 | P a g e
MATHEMATICS
Grade Nine (9)

Lesson 46
Ratio and Proportion
A ratio is a comparison of two similar quantities obtained by dividing one
quantity by the other. Ratios are written with the: symbol.
Example: The ratio of 6 to 3 is
6÷3 = 6/3 = 6:3 = 2
Example: The ratio of 3 to 6 is
3÷6 = 3/6 = 3:6 = 1/2
Notes about ratios:
o Since a ratio is only a comparison or relation between quantities, it is
an abstract number. For instance, the ratio of 6 miles to 3 miles is only
2, not 2 miles.
o As you can see above, ratios can be written as fractions. They also
have all the properties of fractions that you have learned in the
previous part of this station.
o The ratio of 6 to 3 should be stated as 2 to 1, but common usage has
shortened the expression of ratios to be called simply 2.
o If two quantities cannot be expressed in terms of the same unit, there
cannot be a ratio between them.
Can you find these ratios?
1. 2 quarts to 7 gallons
2. 5 dollars to 25 cents
3. 4 meters to 3 kilograms

235 | P a g e
MATHEMATICS
Grade Nine (9)
Answers:
1. 7 gallons = 7 × 4 = 28 quarts so, the ratio of 2 quarts to 28 quarts = 2 / 28 = 1:14
2. 5 dollars = 500 cents so, the ratio of 500 cents to 25 cents = 500 / 25 = 20
3. Meters cannot be expressed in terms of kilograms, so this ratio cannot be
expressed.
Problem:
If two full time employees accomplish 20 tasks in a week, how many such
tasks will 5 employees accomplish in a week?
2:5 = 20:x
2 × x = 5 × 20
x = 50 tasks
This answer is obtained by knowing about proportions and how they are
used. You can set up proportions by using ratios. Remember, ratios are
comparing similar things. In the problem above, the first ratio is comparing
employees and the second is comparing tasks.
Definition:
A proportion is a statement of the equality of two ratios.
A proportion is of a statement that two ratios are equal. It can be written in
two ways:
1) Two equal fractions,

2) Using a colon,

236 | P a g e
MATHEMATICS
Grade Nine (9)
When two ratios are equal, then the cross products of the ratios are equal.

That is, for the proportion

Example:
6:3=2:1 or 6/3=2/1 or 6x1 = 3x2
are ways to write the proportion expressed as: 6 is to 3 as 2 is to 1
Example:
2:8=1:4 or 2/8=1/4 or 2x4=8x1
are ways to write the proportion expressed as: 2 is to 8 as 1 is to 4

Notes about proportions:


If any three terms in a proportion are given, the fourth may be found. Given
the proportion: a:b = c:d or a/b=c/d

and using the principals of manipulating equations, we observe that


a = (b × c) / d and c = (a × d) / b
b = (a × d) / c and d = (b × c) / a

An easy way to remember this is to say that in a proportion:


The product of the means is equal to the product of the extremes.

237 | P a g e
MATHEMATICS
Grade Nine (9)
It is important to remember that to use the proportion, the ratios must be
equal to each other and must remain constant.
Try a few proportion problems:
1. Find the value of x in 16:8 = x:5
2. Find the value of x in 25:15 = 10:x
3. A pipe transfers 236 gallons of fuel to the tank of a ship in 2 hours. How
long will it take to fill the tank of the ship that holds 4543 gallons?
4. An I-beam 12 feet long weighs 52 pounds. How much does an I-beam
of the same width weigh if it is 18 feet long?
1 16:8 = x: 5
16=x
8=5
x × 8 = 16 × 5
x = 10

2 25:15 = 10: x
25=10
15 = x
x × 25 = 15 × 10
x=6

3 236:2 = 4543: x
x × 236 = 2 × 4543
x = 38.5 hours

4 12:52 = 18:x
x × 12 = 52 × 18
x = 78 pounds

238 | P a g e
MATHEMATICS
Grade Nine (9)

Quiz
A. Write each of the following as ratio:
1. 18 boys to 16 girls in the classes
2. 100 cm on 1 m
3. 200 dL to L
4. 34a km in 5 hours
5. 5 pencils for P30.00
6. 8 heartbeats in 6 seconds
7. 100 km in 12 liters of gasoline
8. eight 25-centave coins and four 50-centavo coins
9. 3 batteries for P33. 00
10. 500 meters for 3.5 minutes
B. Find the missing values in the following proportions:
3 𝑐 30 𝑑
1. = 27 2. =4
8 36
14 𝑐 𝑎 10
3. = 33 4. = 15
22 21
28 7
5. =6
𝑏
𝒑 𝟕
C. Given 𝒒 = 𝟒. Fill in the blank with the correct answer

1. The means are and


2. The extremes are and
3. 4p =
𝑞
4. =
𝑝
4
5. =
𝑞

239 | P a g e
MATHEMATICS
Grade Nine (9)
D. Solve for x in each proportion.
𝑥 8 6 12
1. = 2. =
4 5 𝑥 14
3 𝑥 3 24
3. = 4. =
12 36 𝑥 40
7 4 12 𝑥
5. = 6. =
8 𝑥 18 2
𝑥−3 2 2+𝑥 𝑥+2
7. = 8. =
4 3 8 5
3+𝑥 4 7 2
9. = 10. =
𝑥 3 2𝑥−1 𝑥

240 | P a g e
MATHEMATICS
Grade Nine (9)

Lesson 47
Proportional Segments and Similarity
According to the theorem on proportional segments. “two segments are
divided proportionally if the measures of the segment of one has the same ratio
as the measures of the other”.
Examples

I LM I:I MN I = I PQ I:I QR I
3:12 = 5:20
3(20) = 12(5)
60 = 60
This shows that M and Q divide the segments proportionally.

241 | P a g e
MATHEMATICS
Grade Nine (9)

Comparing the two figures, we can say that,


The figures are said to be congruent ≅ if they have the same shape and
are equal in size. On the other hand, two figures are similar ~ if corresponding
angles are equal and corresponding sides are proportional.

242 | P a g e
MATHEMATICS
Grade Nine (9)

Quiz
A. Segments AB and CD are divided proportionally by X and Y respectively. Find
the missing part

B. Identify whether the following figures are similar or congruent.


1. 6.

2.
7.

3. 8.

9.
4.

5. 10.
A B C

243 | P a g e
MATHEMATICS
Grade Nine (9)

Lesson 48
Similar Triangles
Definition: In mathematics, polygons are similar if their corresponding
(matching) angles are congruent (equal in measure) and the ratio of their
corresponding sides is in proportion.
(This definition allows for congruent figures to also be "similar",
where the ratio of the corresponding sides is 1:1.)

Facts about similar triangles:

1
The ratio of the corresponding sides is called the ratio of
2

similitude or scale factor.

244 | P a g e
MATHEMATICS
Grade Nine (9)
T X

R S
W Y
TR = 18 WY = a TS = 12 XY = 15 RS=16 WX = b
𝑇𝑅 𝑇𝑆 𝑅𝑆
= =
𝑊𝑌 𝑋𝑌 𝑊𝑋
18 12 16
= =
𝑏 15 𝑎
18 12 12 16
= 15 ; =
𝑏 15 𝑎

18(15) = 12b; 15(16) = 12a


270 = 12b; 240 = 12a
b= 22.5 a = 20

Quiz
Find the numbers represented by each letter.
1. ∆TON
t = 12
o = 15
n = 20
∆TON ~∆AME. If e = 30 find a and m.
2. ∆ABC
a=5
b=7
c=9
∆ABC~∆XWZ. If x=16 and w–21, find z.
3. ∆ABC
a=4
b=6
c=8
∆ABC ~∆DEF. If d = 10, find e and f.

245 | P a g e
MATHEMATICS
Grade Nine (9)
4. ∆ART
a = 20
r = 15
t = 18
∆ART ~∆LUV. If u = 20, find l and v.
5. ∆BSW
b = 25
s = 20
w = 25
∆BSW~∆ARM. If r=18, find a and m.
B. In the figure, if AB II DC, x = 12, a = 10, b = 24, and EB = 30, find the
value of y, z, and k.

246 | P a g e
MATHEMATICS
Grade Nine (9)
C. In the figure, MN II ST.

a. If NU = 4, UM = 6, and MS = 3, find NT.


b. If SU = 24, NT = 6, and NU = 12, find MS.

247 | P a g e
MATHEMATICS
Grade Nine (9)

Lesson 49
Proportionality Theorems
Basic Proportionality Theorem
Postulate: if a line is parallel to one side of a triangles and
intersect the other side of interior points, then the measures of one
those sides and the two segments onto which it is divided are
proportional to the measures of the three corresponding segments on
the other side.
XY II AB, where the distance (AC) ~ (BC)
(XC) ~ (YC), and (AX) ~ (BY)
Therefore, we can say that
𝐴𝐶 𝑋𝐶 𝐴𝑋
= =
𝐵𝐶 𝑌𝐶 𝐵𝑌
In triangle ABC, if DE II BC, then
𝐴𝐷 𝐴𝐸 𝐴𝐷 𝐴𝐸
1. = 2. =
𝐵𝐷 𝐶𝐸 𝐴𝐵 𝐴𝐶
𝐵𝐷 𝐶𝐸 𝐴𝐷 𝐴𝐵
3. = 4. =
𝐴𝐵 𝐴𝐶 𝐷𝐸 𝐵𝐶
𝐴𝐸 𝐴𝐶
5. =
𝐷𝐸 𝐵𝐶

248 | P a g e
MATHEMATICS
Grade Nine (9)
Basic Proportionality Theorem
A line is parallel to the other side, is a line cuts the two sides of a triangle in
proportional segments.
If the tree coplanar parallel lines have two traversals, the measures of
corresponding segments of transversals are proportional.
Given 𝑙1 II 𝑙2 II 𝑙3 intersected by transversal line LM and line NO at A, B, C, D, E,
and F.
𝐴𝐵 𝐷𝐸
Prove: 𝐵𝐶 = 𝐸𝐹

Proof: When A, B, C, D, E and F are distinct points


Statement Reason
1. AB exist 1. If A and B are points, then line AB exits.
2. AF intersects BE 2. These two points are contained in one line with
the point of intersection X.
𝐴𝐵 𝐴𝑋 𝐴𝑋 𝐷𝐸
3. = ; = 3. If a line intersects two sides of a triangle and is
𝐵𝐶 𝐹𝑋 𝐹𝑋 𝐸𝐹

parallel to the third side, than the two sides are parallel.
𝐴𝐵 𝐷𝐸
4. = 4. Transitivity
𝐵𝐶 𝐸𝐹

The angle bisector of a triangle separates the segments of the opposite sides
whose lengths are proportional to the lengths of the other two sides.

249 | P a g e
MATHEMATICS
Grade Nine (9)
Example:
Given: ∆𝐴𝑆𝐵; 𝑆𝑅 𝑏𝑖𝑠𝑒𝑐𝑡𝑠 𝑎𝑛𝑔𝑙𝑒 𝑆 C
AB intersects at point R S
𝐴𝑅 𝐴𝑆
Prove: 𝑅𝐵 = 𝑆𝐵

A R B
Proof:
Statement Reason
1. ASR ≅ BSR 1. Def. of angle bisector
2. ASR ≅ 𝑆𝐶𝐵 2. The corresponding angles of
paralle lines cut by a transversal are
congruent
3. SBR ≅ SCB 3. Alternate interior angles are
congruent
4. SCB ≅ SBC 4. Transitivity
5. SC ≅ RB 5. The two opposite sides of ≅ angles
are≅

6.
𝐴𝑅
=
𝐴𝑆
6. A line intersects two sides of a
𝑅𝐵 𝑆𝐵
triangle and is parallel to the third
side. The first two sides are
proportional.

250 | P a g e
MATHEMATICS
Grade Nine (9)

Quiz
In the following figures, determine the sides that are divided proportionally.
1. Given: ED II BC 2. Given DE II AC

𝐴𝐷
a. =
𝐴𝐶
𝐶𝐵
b.
𝐴𝐶
= a. 𝐶𝐸
=
𝐴𝐷
𝐸𝐵
c.
𝐴𝐵
= b. =
𝐴𝐸 𝐵𝐷

d.
𝐴𝐸
= c. BD : AD =
𝐴𝐵
d. AD : DE =

B. Use the figure to determine whether each statement is true or false.

𝑁𝑆 𝑁𝑂 𝑁𝑆 𝑆𝑂
1. = 4. =
𝑆𝐸 𝑂𝑇 𝑁𝐸 𝐸𝑇
𝑁𝑆 𝑁𝑂 𝑁𝑂 𝑆𝑂
2. = 5. =
𝑁𝐸 𝑂𝑇 𝑁𝑇 𝐸𝑇
𝑁𝑆 𝑆𝑂
3. =
𝑆𝐸 𝐸𝑇

251 | P a g e
MATHEMATICS
Grade Nine (9)
C. Determine whether the given information implies that SE II OR.

𝑆𝐻 𝐸𝐻 𝐻𝑂 𝑂𝑅
1. = 4. =
𝑂𝐻 𝑅𝐻 𝑆𝑂 𝐸𝑅
𝑆𝐻 𝐻𝐸 𝐻𝑆 𝐻𝐸
2. = 5. 𝑆𝑂 =
𝑆𝑂 𝐸𝑅 𝑆𝐸
𝑆𝑂 𝐸𝑅
3. =
𝑂𝐻 𝑅𝐻

D. Use the figure below to find each indicated length.


1. EO = 6, VO = 4, AO = 9, BO =
2. AB = 3, BO = 2, EV = 4, VO =
3. BO = 7, AB = 4, VO = 5, VE =
4. OA = 9, EO = 12, EV = 5, AB =
5. BV = 12, AE = 20, VO = 10, EO =

C. Find each indicated length.

1. AB = 5, BD = 2, BC = 7, DE =
2. AB = 4, BD = 1, DE = 6.25, BC =
3. AB = 3.4, AC = 6.3, CE = 2.1, BD =
4. AB = 34, AC = 26, BD = x + 4, CE = x, AE =
5. AB = 18, AC = 21, CE = x, BD = x – 1, AD =

252 | P a g e
MATHEMATICS
Grade Nine (9)

Lesson 50
Similarity and Isosceles Triangle
Theorems
AAA Similarity Theorem
If in two triangles the corresponding angles are congruent, then the two triangles
are similar.
If ∆BOS ↔ ∆VIC O I
∠B≅∠V, ∠O ≅∠I, ∠S ≅∠C C V
then ∆BOS ~ ∆VIC B S

SAS Similarity Theorem


If in two triangles two pairs of corresponding sides are proportional and the
included angles are congruent, then the triangles are similar.
If ∆ICE ↔ ∆BOX OB C O
𝐶𝐼 𝐶𝐸
=
𝑂𝐵 𝑂𝑋
and ∠C ≅∠O B X
then ∆ICE ~∆BOX I E

253 | P a g e
MATHEMATICS
Grade Nine (9)
SSS Similarity
If the two triangles three corresponding sides are proportional, then the
triangles are similar
If ∆ SUN ↔ ∆BLK U L
SU 𝑈𝑁 𝑆𝑁
= =
𝐵𝐿 𝐿𝐾 𝐵𝐾
Then ∆SUN ~∆BLK S N B K
Isosceles triangles are triangles whose two sides are congruent. These
congruent sides are called legs and the third side is called the base.

The angles at the base are called the base angles and the angle opposite the
base is called the vertex angle of the isosceles triangle.

Vertex angle
Leg base angles

If two sides of a triangle are congruent, then the angles opposite to these sides
are congruent.

254 | P a g e
MATHEMATICS
Grade Nine (9)
Proof:

Let S be the midpoint of .


Join R and S.

Since S is the midpoint of PQ, .


By Reflexive Property,

It is given that
Therefore, by SSS,

Since corresponding parts of congruent triangles are congruent,

The converse of the Isosceles Triangle Theorem is also true.


If two angles of a triangle are congruent, then the sides opposite those angles
are congruent.

If , then .

255 | P a g e
MATHEMATICS
Grade Nine (9)

Quiz
A. Consider the figure illustrated below to compare the given statements.

1. If XP ≅ XR, then < ≅< .


2. If XO ≅ XW, then < ≅< and < ≅<
3. If ∆PXR is an isosceles right triangle, then the measure of <P is
B. Given ∆JHO≅ ∆MAI with legs 7 cm long, name all pairs of congruent angles.

C. Given the ratios of the lengths of the sides of ∆FOR and ∆BUT.

1. ∆FOR 3:3:6 a. ∆BUT 1:1:2 b. ∆BUT 4:4:9


2. ∆FOR 2:3:4 a. ∆BUT 4:6:10 b. ∆BUT 6:9:12
3. ∆FOR 4:8:10 a. ∆BUT 6:10:12 b. ∆BUT 6:12:15
4. ∆FOR 2:4:5 a. ∆BUT 6:8:15 b. ∆BUT 8:16:20
5. ∆FOR 1:2:3 a. ∆BUT 3:4:5 b. ∆BUT 4:16:20
6. ∆FOR 2:3:7 a. ∆BUT 4:6:14 b. ∆BUT 1:14:20
7. ∆FOR 1:3:3 a. ∆BUT 3:9:9 b. ∆BUT 2:4:12
8. ∆FOR 2:1:7 a. ∆BUT 6:3:21 b. ∆BUT 12:14:26
9. ∆FOR 3:4:5 a. ∆BUT 9:12:20 b. ∆BUT 2:7:19
10. ∆FOR 2:3:7 a. ∆BUT 2:1:6 b. ∆BUT 8:12:28

256 | P a g e
MATHEMATICS
Grade Nine (9)

Lesson 51
Pythagorean Theorem
The theorem states that:
"The square on the hypotenuse of a right triangle is
equal to the sum of the squares on the two legs" (Eves
80-81).

This theorem is talking about the area of the squares


that are built on each side of the right triangle.

Accordingly, we obtain the following areas for the squares, where the green and
blue squares are on the legs of the right triangle and the red square is on the
hypotenuse.

area of the green square is

area of the blue square is

area of the red square is


From our theorem, we have the following relationship:
Area of green square + area of blue square = area of red square or
𝑎2 + 𝑏 2 = 𝑐 2

257 | P a g e
MATHEMATICS
Grade Nine (9)
This equation allows you to find the length of a side of a right triangle when
they've given you the lengths for the other two sides, and, going in the other
direction, allows you to determine if a triangle is a right triangle when they've
given you the lengths for all three sides.
• Given the right triangles displayed below, find the lengths of the remaining
sides.

a) The side opposite the right triangle is c; how you name the other two
sides really doesn't matter. You'll plug the values into the Theorem, and
solve:
802 = 482 + b2
6400 = 2304 + b2
4096 = b2
64 = b
Normally, you had needed both answers, from the "plus / minus", to solve
the quadratic equation by taking square roots. In this case, though, you
knew going in that you would need to find a positive value for the length
of the third side, so you can ignore the negative solution.
b = 64

258 | P a g e
MATHEMATICS
Grade Nine (9)
b) This triangle one works the same way as did (a):
912 = 842 + b2
8281 = 7056 + b2
1225 = b2
35 = b
b = 35
• Given the triangles below, determine if the triangles are right.

a) You need to see if the squares of the legs equal the square of the
hypotenuse:
452 + 552 = 2025 + 3025 = 5050
752 = 5625
The triangle for (a) is NOT a right triangle.
b) This one works the same as for (a):
282 + 452 = 784 + 2025 = 2809
532 = 2809
The triangle for (b) is a right triangle.

259 | P a g e
MATHEMATICS
Grade Nine (9)
Right-angled triangle
In any right-angled triangle, ABC, the side opposite
the right-angle is called the hypotenuse. Here we use
the convention that the side opposite angle A is
labelled a. The side opposite B is labelled b and the
side opposite C is labelled c.
Pythagoras’ theorem states that the square
of the hypotenuse, (c2), is equal to the sum
of the squares of the other two sides, (a 2 +
b2).
Pythagoras’ theorem: c2 = a2 + b2
Example:
Suppose AC = 9cm and BC = 5cm as shown.
Find the length of the hypotenuse, AB.
Solution
Here, a = BC = 5, and b = AC = 9. Using the
theorem
c2 = a2 + b2
= 52 + 92
= 25 + 81
= 106
c =√106 = 10.30 (2dp.) The hypotenuse has length 10.30cm.

260 | P a g e
MATHEMATICS
Grade Nine (9)
Example
In triangle ABC shown, suppose that the length of the hypotenuse is 14cm and that a
= BC =
3cm. Find the length of AC.
Solution
Here a = BC = 3, and c = AB = 14. Using the theorem
c2 = a2 + b2
142 = 32 + b2
196 = 9 + b2
b2 = 196 − 9
= 187
b =√187 = 13.67 (2dp.) The length of AC is 13.67cm.
Exercises
1. In triangle ABC in which C = 90◦, AB = 25 cm and AC = 17 cm. Find the length BC.
Answer: 8.33 cm. (2dp.)
2. In triangle ABC, the angle at B is the right-angle. If AB = BC = 5 cm find AC.
Answer: AC = √ 50 = 7.07 cm. (2dp.)
3. In triangle CDE the right-angle is E. If CD = 55cm and DE = 37cm find EC.
Answer: EC = √ 1656 = 40.69 cm. (2dp.)

261 | P a g e
MATHEMATICS
Grade Nine (9)

Quiz
A. Determine the measures of the missing sides.
A right triangle ABC with right angle at C.

1. a = 3, b = 4, c = ?
2. a = 14, b = 16, c = ?
3. b = 9, c = 18, a = ?
4. b = 21, c = 35, a = ?
5. c = 100, a = 57, b = ?
6. c = 30, b = 18, a = ?
7. c = 20 b = 8, a = ?
8. a = 4 b = 6 , c = ?
9. a = 12, b = 13, c = ?
10. c = 14, b = 4, a = ?
11. a = 12, c = 24, b = ?
12. a = 4y, b = 3x – 1, c = ?
13. b = 2x + 1, a = 2x + 4, c = ?
14. a = 3x – 1, c = x + 2, b = ?
15. a = 4x – 1, c = 9x + 2, a = ?
B. The lengths of the three sides of a triangle are given. Determine whether the
triangle is a right triangle.
1) 3, 4, 5 5) 6, 7, 10
2) 5, 12, 13 6) 5, 8, 9
3) 15, 20, 25 7) √6, √10, 4
5 3
4) 4 , 4, 1 8) 40, 41

262 | P a g e
MATHEMATICS
Grade Nine (9)
C. In the figure, ABC is a right triangle with B is a right angle.

a. If a = 6 and c = 8, find b.
b. If a = 15 and c = 20, find b.
c. If a = 21 and b = 35, find c.
d. If b = 30 and c = 18, find a
e. If a = 14 and b = 20, find x.

263 | P a g e
MATHEMATICS
Grade Nine (9)

Lesson 52
The Pythagorean Condition
The converse of the Pythagorean Theorem states that if a, b and c are the
three sides of a triangle and c2 = a2 + b2, then the triangle is a right triangle with
c as the length of the hypotenuse. What if the triangle is not a right triangle? How
is c2 related to a2 + b2 if the triangle is acute or obtuse?

Draw an acute triangle and one obtuse triangle. Measure the longest side
(and label as c), and measure the other two sides (and label as a and b).
Compute c2 and a2 + b2 for each triangle. How do you compare c2 to a2 + b2 in
the acute triangle? in the obtuse triangle?
The following theorem will tell us to how to determine whether a triangle is acute
or obtuse
Theorem
If a, b and c are the lengths of the three sides of a triangle with c as the
longest side, then the triangle is:
acute if c2 < a2 + b2
obtuse if c2 > a2 + b2

264 | P a g e
MATHEMATICS
Grade Nine (9)
The following figures show the three different kinds of triangles and the
comparisons of their sides c being the longest side.

right triangle acute triangle obtuse triangle


c2 = a2 + b2 c2 < a2 + b2 c2 > a2 + b2
Examples: Determine whether the triangle with the given sides is right, acute or
obtuse.
1.) 6, 8, 9
Solution
The longest side is c = 9. We use theorem to determine the kind of triangle.
c2 a2 + b 2
(9)2 (6)2 + (8)2
81 36 + 64
81 < 100
Since c2 < a2 + b2, the triangle is an acute triangle.
2.) 5, 8, 12
Solution: The longest side is c = 12. So, we check
c2 a2 + b 2
(12)2 (5)2 + (8)2
144 25 + 64
144 > 89
Since c2 > a2 + b2, the triangle is an obtuse triangle.

265 | P a g e
MATHEMATICS
Grade Nine (9)

Quiz
A. Given the three sides of a triangle, determine if it is acute, right, or obtuse.
1) 1, 3, 6 9) 3, 3, 3√2
2) 4, 5, 9 10) 3, 7, 10
3) 4, 6, 8 11) 4, 7, 12
4) 4, 6, 9 12) 12, 5√12, 16
5) 6, 12, 20 13) 12, 14, 20
6) 8, 12, 20 14) 14, 16, 20
7) 12, 15, 21 15) 21, 20, 28
8) 21, 24, 29

B. In ACO, find the least integer value of AO if C is an obtuse angle.

C. In PJO, find the greatest integer value of PJ if O is an acute angle.

266 | P a g e
MATHEMATICS
Grade Nine (9)

Lesson 53
Special Triangles
The 45º- 45º- 90º triangle is one of two special right triangles we will be
investigating. The "special" nature of these triangles is their ability to yield exact
answers instead of decimal approximations when dealing with trigonometric
functions.

If we represent the legs of an


isosceles right triangle by 1, we can use
the Pythagorean Theorem to establish
pattern relationships between the lengths
of the legs and the hypotenuse. These
relationships will be stated as
"short cut formulas" that will allow us to
quickly arrive at answers regarding side
lengths without applying trigonometric
functions, or other means.
There are two pattern formulas that
apply ONLY to the 45º-45º-90º triangle.

Note: The legs need not be a length of 1 for these patterns to apply.
The patterns will apply with any length legs.

267 | P a g e
MATHEMATICS
Grade Nine (9)
45º-45º-90º (Isosceles Right Triangle)
Pattern Formulas
(you do not need to memorize these formulas as such, but you do need to
memorize the patterns)

H=hypotenuse
L = leg
The 30º- 60º- 90º triangle is one of two special right triangles we will be
investigating. The "special" nature of these triangles is their ability to yield exact
answers instead of decimal approximations when dealing with trigonometric
functions.

If you draw an altitude in an equilateral


triangle, you will form two congruent 30º-
60º- 90º triangles. Starting with the sides of
the equilateral triangle to be 2, the
Pythagorean Theorem will allow us to
establish pattern relationships between the
sides of a 30º- 60º- 90º triangle. These
relationships will be stated here as "short
cut formulas" that will allow us to quickly
arrive at answers regarding side lengths

268 | P a g e
MATHEMATICS
Grade Nine (9)
without applying trigonometric functions,
or other means.
There are three pattern relationships that
we can establish that apply ONLY to a 30º-
60º-90º triangle.

Note: the hypotenuse need not be a length of 2 for these patterns to apply.
The patterns will apply with any length hypotenuse.

30º-60º-90º Triangle
Pattern Formulas
(you do not need to memorize these formulas as such, but you do need to
memorize the relationships)

Labeling:
H= hypotenuse
LL = long leg (across
from 60º)
SL = short leg (across
from 30º)

269 | P a g e
MATHEMATICS
Grade Nine (9)
Short Cut Pattern Formulas:
(These formulas give answers directly.
We don't need to work out the trig,
as we already know the pattern relationships)
short leg: You must remember that these
formula patterns can be used
ONLY in a 30º-60º-90º triangle.

long leg:

combining the first two:

Using the patterns to find the lengths of sides:


EASY: x is the short leg y is the long leg

Find x and y.

Answer Answer

270 | P a g e
MATHEMATICS
Grade Nine (9)
HARDER: 6 is the short leg and
x is the hypotenuse y is the long leg
(start with what you
have given)
Find x and y.

Answer

Answer

Quiz
A. Find the value of each variable.

B. Solve each indicated length.


1. ∆HER is 30-60-90 triangle. Find the indicated lengths.
a) HE = 40, HR = ?, RE = ?
b) HE = 31, HR = ?, RE = ?
c) HR = 12, HE = ?, RE = ?
d) HE = 45, HR = ?, RE = ?
e) HE = 60, HR = ?, RE = ?

271 | P a g e
MATHEMATICS
Grade Nine (9)

2. ∆ MEN is a 45-45-90 triangle. Find the indicated lengths.


a) MN = 42, NE = ?, ME = ?
b) NE = 20, MN = ?, ME = ?
c) ME = 14, MN = ?, NE = ?
d) NE = 40, MN = ?, ME = ?
e) ME = 50, MN = ?, NE = ?

C. The length of a side of a square is given. Find the length of its diagonal and its
perimeter.
1) √8 cm 4) 3√2 cm
2) 7.5 cm 5) 4
3) 12 cm

D. Use the accompanying figure to find the length of the missing sides.
1. a=2
2. a=6
3. b = √3
4. b = 2 √3
5. c = 20

272 | P a g e
MATHEMATICS
Grade Nine (9)

Lesson 54
Angles and Their Measures
Coterminal angles are angles in
standard position (angles with the initial
side on the positive x-axis) that have a
common terminal side. For example, 30°, –
330° and 390° are all coterminal.
To find a positive and a negative
angle coterminal with a given angle, you
can add and subtract 360° if the angle is
measured in degrees or 2π if the angle is
measured in radians.
Example 1:
Find a positive and a negative angle coterminal with a 55° angle.
55° – 360° = –305°
55° + 360° = 415°
A –305° angle and a 415° angle are coterminal with a 55° angle.
Example 2:
Find the three positive angles that are coterminal with 48ᵒ.
a. Add multiples of 360ᵒ. There are many answers.
48 + 360ᵒ = 408ᵒ
48 + 2(360ᵒ) = 768ᵒ
48 + 3(360ᵒ) = 1128ᵒ
Thus, the angles of measures 408. 768, and 1 128 are co terminal with 48
273 | P a g e
MATHEMATICS
Grade Nine (9)
Example 3:
Find two negative angles that are coterminal with 23ᵒ
Solution
Subtract multiples of 360ᵒ. There are many answers
23 – 360ᵒ = -337ᵒ 23 – 2(360ᵒ) = -697ᵒ
Thus, angles of measures -337ᵒ and -697ᵒ are coterminal with 23ᵒ
There is another commonly used unit of angle measure called the radian.
𝜋𝑟𝑎𝑑
To convert degrees to radians, multiply the number of degrees by .
180°

Example 4
Convert each degree measure to radian measure.
a. 150ᵒ b. 480ᵒ
Solution
𝜋𝑟𝑎𝑑 5𝜋
a. 150ᵒ = (150ᵒ)( )= 𝑟𝑎𝑑
180° 6
𝜋𝑟𝑎𝑑 8𝜋
b. 480ᵒ = (480ᵒ)( 180° ) = rad
3
180°
To convert from radians to degrees, multiply the number of radians by 𝜋𝑟𝑎𝑑

Example
Convert each radian measure to degree measure.
3𝜋 7𝜋
a. rad b. rad
4 2

Solution
3𝜋 3𝜋 180°
a. rad =( rad)(𝜋𝑟𝑎𝑑) = 135°
4 4
7𝜋 7𝜋 180°
b. rad = ( 2 rad)(𝜋𝑟𝑎𝑑) = 630ᵒ
2

274 | P a g e
MATHEMATICS
Grade Nine (9)

Quiz
A. Convert each radian measure to degree measure.
𝜋 7𝜋
1) 4 rad 6) 8 rad
3𝜋 𝜋
2) 7 rad 7) 2 rad
7𝜋 5𝜋
3) 9 rad 8) 18 rad
13𝜋 17𝜋
4) rad 9) rad
36 72
31𝜋 43𝜋
5) 36 rad 10) 72 rad

B. Find the three positive angles that are coterminal with


1)48⁰ 5)190⁰
2)25° 6)-45⁰
3)278⁰ 7)-12°
4)-67⁰ 8)-35°

C. Find the three negative angles that are coterminal with


1)-23⁰ 5)45°
2)47⁰ 6)36°
3)89⁰ 7)-78⁰
4)-12⁰

E. Express each in radians.


1)20 ⁰ 9)12°
2)-23⁰ 10)26°
3)89⁰ 11)-56°
4)126⁰ 12)10°
5)-678⁰ 13)-13°
6)-45⁰ 14)-25°
7)-745⁰ 15)452⁰
8)968⁰

275 | P a g e
MATHEMATICS
Grade Nine (9)

Lesson 55
Trigonometric Ratios
In a right triangle, there are actually six possible trigonometric ratios, or
functions.

A Greek letter (such as theta or phi ) will now be used to represent the angle.

Notice that the three new ratios at the right are reciprocals of the ratios on the
left.

276 | P a g e
MATHEMATICS
Grade Nine (9)

Applying a little algebra shows the connection between these functions.

Reciprocal Functions Also Important

Examples:

1. Given the triangle at the right, express the exact


value of the six trigonometric functions in relation to
theta.

Solution: Find the missing side of the right triangle using the Pythagorean
Theorem. Then, using the diagram, express each function as a ratio of
the lengths of the sides. Do not "estimate" the answers.

277 | P a g e
MATHEMATICS
Grade Nine (9)
Be careful not to jump to
the conclusion that this is
a 3-4-5 right triangle. The
4 in on the hypotenuse
and must be the largest
side.

The following examples pertain to a right triangle in Quadrant I:


𝟏𝟐
2. Given: 𝒄𝒐𝒔𝜽 = , 𝐟𝐢𝐧𝐝 𝒔𝒆𝒄𝜽
𝟏𝟑

Solution: This is an easy problem, since cosine and secant are


reciprocal functions.

278 | P a g e
MATHEMATICS
Grade Nine (9)
𝟐 √𝟓
3. Find 𝑠𝑒𝑐𝜃 and 𝑐𝑜𝑡𝜃 , given 𝒔𝒊𝒏𝜽 = 𝟑 𝒂𝒏𝒅 𝒄𝒐𝒔𝜽 = 𝟑

Solution: Draw a diagram to get a better understanding of the given


information.

Since sine is opposite over hypotenuse,


position the 2 and the 3 accordingly in
relation to the angle theta. Now, since
cosine is adjacent over hypotenuse, position
these values (the 3 should already be
properly placed). Be sure that the largest
value is on the hypotenuse and that the
Pythagorean Theorem is true for these
values. (If you are not given the third side,
use the Pythagorean Theorem to find it.)

Now, using your diagram, read off the values for the secant and the
cotangent.

Secant: Cotangent:

279 | P a g e
MATHEMATICS
Grade Nine (9)

40
4. Given that cos 𝛼 = 41 and 𝛼 is an acute angle, find the other five trigonometric

function values of 𝛼.
Solution
Use the definition of the cosine function.
𝑎𝑑𝑗. 40
cos 𝛼 = ℎ𝑦𝑝 = 41

To find the length of the side opposite angle 𝛼, we use the Pythagorean
theorem:
a2 + b2 = c2
a2 + 402 = 412
a2 = 1681 – 1600
a2 = 81
a=9
You may now use the lengths of the three sides to find the other five ratios.
9 41 41 9 40
sin 𝛼 = 41
csc 𝛼 = 9
sec 𝛼 = 40
tan 𝛼 = 40
cot 𝛼 = 9

The trigonometric function values of an acute angle of a right triangle can be


found when the lengths of its three sides are known. However, there are instances
when we are asked to find the trigonometric function values when the measure
of the acute angle is given. For special angles such as 30°, 45°, 60° we can use
concepts in geometry to determine their trigonometric function values.
An isosceles triangle whose legs have length 1 has a hypotenuse of √2 .

280 | P a g e
MATHEMATICS
Grade Nine (9)
Trigonometric Function Values of 45°
1 √2 √2
sin 45° = = csc 45° =
√ 2 2 1

1 √2 √2
cos 45° = = sec 45° =
√ 2 2 1

tan 45° = 1 cot 45° = 1


For 30° 60° 90° triangle, we know:
a. the side opposite the 30° - angle is half as long as the hypotenuse, and
b. the side opposite the 60°- angle √3 times the length of the side opposite the
30° angle.

Trigonometric Function Values of 30°


1 2
sin 30° = 2 csc 30° = 1
√3 2
cos 30° = sec 30° =
2 √3

1 √3
tan 30° = cot 30° =
√3 1

Trigonometric Function Values of 60°


√3 2 2√ 3
sin 60° = csc 60° = =
2 √ 3 3
1 2
cos 60° = 2 sec 60° = 1 = 2
√3 1 √3
tan 60° = cot 60° = =
1 √3 3

281 | P a g e
MATHEMATICS
Grade Nine (9)

Quiz
A. If 𝜶 is an acute angle in a right triangle, find the other trigonometric function
values of 𝛼
5
1. sin 𝛼 = 13
7
2. tan 𝛼 = 24
15
3. sin 𝛼 =
113
12
4. cos 𝛼 = 37
15
5. cot 𝛼 = 8

B. Suppose that a and b are the lengths of the legs of a right triangle, and c is
the hypotenuse. If a is the side opposite the angle 𝜽, find the values of the six
trigonometric functions of 𝜽.
1. a = 14 b = 21 5. b = 3√2 c = 9√2
2. a = 8 c = 10 6. a = 22.5 b = 22.5
3. b = 25 c = 75 7. b = 2√5 c = 4√5

4. a = 4
5
b=4
9
8. a = 16 c = 12

C. Find the six trigonometric function values of 𝜽 in each right triangle.

282 | P a g e
MATHEMATICS
Grade Nine (9)

Lesson 56
D°M’S’’ and Decimal Degrees
The angle whose measure is 40 degree, 32 minutes, 24seconds can be
written as 40ᵒ32’24’’. This DᵒM’S’’ form is called the standard form. However, while
minutes and seconds are generally used in applications, it is also important to
know its equivalent decimal degrees which are much easier to use with scientific
calculators.
Example 1
Convert 80ᵒ40’30’’ into decimal degrees.
Solution
Convert the units with the lowest value to the next higher unit. That is, convert
seconds to minutes then minutes to degrees.
30’’(1/60) = 0.5
Combine this with 40’. Adding them will result to 40.5 minutes. Convert 40. 5
into degrees.
40. 5 (1/60) = 0.675ᵒ
Combining this to 80ᵒ would give 80.675ᵒ. Thus, 80ᵒ40’30’’ = 80.675ᵒ.

283 | P a g e
MATHEMATICS
Grade Nine (9)
Example 2
Convert 45.4545ᵒ into DᵒM’S’’ or standard form.
Solution
Retain the whole number part. Concentrate on the decimal part. Convert
0.4545ᵒ to minutes.
0.4545 x 60 = 27.27’
Get the decimal part, 0.27 and convert it to seconds
0.27 x 60 = 16. 2
The final answer is 45ᵒ27’16’’.

Convert 120.96ᵒ into DᵒM’S’’


Retain the whole number part. Concentrate on the decimal part.
Convert 0.96 to minutes.
0.96(60) = 57.6’
Get the decimal part, 0.6 and convert it to seconds.
0.6 (60) = 36’’
The final answer is 120ᵒ57’36’’.
Example 4
Convert 135.84ᵒ into DᵒM’S’’
Retain the whole number part. Concentrate on the decimal part.
Convert 0.84 to minutes.
0.84(60) = 50.4’
Get the decimal part, 0.6 and convert it to seconds.
0.4 (60) = 24’’
The final answer is 135ᵒ50’24’’.

284 | P a g e
MATHEMATICS
Grade Nine (9)

Quiz
A. Change the following to decimal degrees.
1) 25⁰ 30’ 25’’ 6) 40⁰ 40’ 15’’
2) 12⁰ 14’ 45’’ 7) 45⁰ 45’ 45’’
3) 16⁰ 24’ 30’’ 8) 14⁰ 15’ 45’’
4) 45⁰ 56’ 30’’ 9) 23⁰ 45’
5) 89⁰ 45’’ 10) 24⁰ 45’ 56’’

B. Change the following decimal degrees to D⁰M’S’’


1) 24. 52⁰ 1) 7. 15⁰
1) 45.15⁰ 1) 6. 002⁰
1) 56.01⁰ 1) 8. 24⁰
1) 89.24⁰ 1) 9.125⁰
1) 16. 45⁰ 1) 3.165⁰

285 | P a g e
MATHEMATICS
Grade Nine (9)
Lesson 57
Cofunctions and Trigonometric Function
Values of Any Angle

In the diagram at the left, the


measures of the angles designated

by and add to 90º. These


angles are complementary angles.

The sine of an acute angle is


equal to the cosine of its
complement.
In this triangle, and
The cosine of an acute
angle is equal to the sine of its
complement.

Such examinations, lead us to three sets of "cofunction" identities:


Sine and cosine are Tangent and cotangent are Secant and cosecant are
cofunctions. cofunctions. cofunctions.

286 | P a g e
MATHEMATICS
Grade Nine (9)
Also written:

Sine and cosine are Tangent and cotangent are Secant and cosecant are
cofunctions. cofunctions. cofunctions.

Notice the connection of the letters C & O:


* sine and cosine cofunctions
* tangent and cotangent cofunctions
* secant and cosecant cofunctions
* complementary
Examples:

1. Solution: Since sine and cosine are


cofunctions, we know that the value will be the
complement of 15º.
Answer: 75º

2. Find the value of 𝜃 for which tan 𝜃 = cot 30ᵒ


Solution
Cot 30ᵒ = tan (90 – 30)
= tan 60ᵒ
3. Express each function in terms of its function.
a. sin 47⁰ b. cot 78⁰

287 | P a g e
MATHEMATICS
Grade Nine (9)
Solution
a. sin 47⁰ = cos (90 – 47)
= cos 43⁰
b. cot 78⁰ = tan (90 -78)
= tan 12⁰
Exact trigonometric function values can be found without the help of a
scientific calculator when the terminal side of an angle coincides with one of the
axes or makes a 30ᵒ-. 45ᵒ, or 60ᵒ- angle with the axes. We can also find
approximations of these values with a scientific calculator. With the use of a
angle. In general, we can approximate the trigonometric function values of any
angle. In general, we can approximate the function values of all angles without
using a reference angle.
Examples:
Find each of the following function values.
a. sin 115ᵒ b. cos 452ᵒ c. tan (-72.5)ᵒ
Solution
Use scientific calculator to find the trigonometric function values.
a. sin 115ᵒ = 0.906307787 rounded to 0.9063
b. cos 452ᵒ = -0.034899496 rounded to -0.0349
c. tan (-72.5)ᵒ = -3.171594802 rounded to -3.1716

288 | P a g e
MATHEMATICS
Grade Nine (9)

Quiz
A. Find each of the following function values.
1) sin 23⁰ 6) csc 34⁰
2) cot (-34⁰) 7) tan 45⁰
3) cos 133⁰ 8) cos 45⁰
4) sec 344⁰ 9) tan 234⁰
5) sin 133⁰ 10) cot 56⁰

B. Express each function in terms of its cofunction.


1) cos 76⁰ 6) sin 34⁰
2) cos 46⁰ 7) cot 23⁰
3) csc 67⁰ 8) sec 45.23⁰
4) tan 45. 89⁰ 9) sin 34. 34⁰
5) tan 76.4⁰ 10) cot 45.34⁰

289 | P a g e
MATHEMATICS
Grade Nine (9)

Lesson 58
Solving Right Triangles
Formulas:

where A represents the angle of reference.

The formulas can be remembered The formulas can be


by: remembered by:
oh heck, another hour of algebra! Oscar had a heap of apples

There are many such memory tricks.

Basic Trigonometry Rules:


• These formulas ONLY work in a right triangle.
• The hypotenuse is always across from the right
angle.
• Questions usually ask for an answer to the
nearest units.
• You will need a scientific or graphing calculator.

290 | P a g e
MATHEMATICS
Grade Nine (9)
How to set up and solve a trigonometry problem when solving for a
side of the triangle:

Example 1: In right triangle ABC, hypotenuse AB=15 and angle


A=35º. Find leg length, BC, to the nearest tenth.
Set Up the Drawing:
1. Draw a picture depicting the situation.
2. Be sure to place the degrees INSIDE the
triangle.
3. Place a stick figure at the angle as a point
of reference.
4. Thinking of yourself as the stick figure, label
the opposite side (the side across from you),
the hypotenuse (across from the right
angle), and the adjacent side (the leftover
side).
5. Notice how the values on the sides of the
triangle "pair up". The h pairs with the 15,
the o pairs with the x, but the a stands
alone. The a has no companion term. This
means that the a is NOT involved in the
solution of this problem. Cross it out!
6. This problem deals with o and h which
means it is using sin A.

291 | P a g e
MATHEMATICS
Grade Nine (9)
Set Up the Formula:
1. Place the degrees in the formula
for angle A.
2. Replace o and h with their
companion terms.
3. Using your scientific/graphing
calculator, determine the value of
the left side of the equation. (On
most scientific calculators, press 35
ANSWER: 8.6 first and then press the sin key. On
most graphing calculators, press
the sin key first and the 35
second.)
4. Solve the equation
algebraically. In this case, cross
multiply and solve for x. Or just
remember that if the x is on the
top, you will multiply to arrive at
your answer. If x is on the bottom,
divide to arrive at your answer
(see next example).
5. Round answer to the desired
value.

292 | P a g e
MATHEMATICS
Grade Nine (9)
Example 2
Solve for the right triangle that
a = 17 and c = 29
Solution
We are given the leg a and the hypotenuse c. the
figure and the given data are shown.
Apply the sine ratio to find A
𝑠𝑖𝑑𝑒
Sin A =
ℎ𝑦𝑝𝑜𝑡𝑒𝑛𝑢𝑠𝑒
17
Sin A =29

Sin A = 0.58621
A = 35.89ᵒ or 35ᵒ 53’
A is the inverse sin 0.58621 or sin-1 0.58621
Angle A and Angle B are complementary angles. Thus,
B = 90 - 35ᵒ53’
= 54ᵒ7’
Use the Pythagorean Theorem to solve for b.
b = √𝑐 2 − 𝑎2
= √292 − 172
= √841 − 289
= 23.49
Thus, A = 35ᵒ 53’ , B = 54ᵒ7’, and b = 23.49

293 | P a g e
MATHEMATICS
Grade Nine (9)

Quiz
A. Solve each right triangle given some of its parts. Give the lengths of the sides
to two decimal places and the angles in degrees and minutes.
1) A =35 ; c = 14.7 6) A = 55⁰ ; c = 25.31
2) A = 17⁰ 12’ ; c = 27.41 7) B = 76⁰ 32’ ; c = 65.45
3) A = 64⁰ 23’; c = 65. 03 8) B = 12⁰ 40, ; a = 20
4) A = 47⁰ ; c = 52 9) a = 12 ; c = 18
5) b = 1.8 ; c = 4 10) a = 16 ; b = 20

B. The following sets of numbers are called Pythagorean Triples.


These numbers satisfy the Pythagorean Theorem, c2 = a2 + b2. If the given
numbers are the measurements of the sides of the right triangle ABC, find the
measures of the acute angles in each set.
1) 3, 4, 5 6) 13, 84, 85
2) 5, 12, 13 7) 16, 63, 65
3) 8, 15, 17 8) 17, 144, 145
4) 9, 40, 41 9) 24, 143, 145
5) 12, 35, 37 10) 25, 312, 313

294 | P a g e
MATHEMATICS
Grade Nine (9)

Lesson 59
Angles of Elevation and Depression
In order to solve problems involving angles of elevation and depression, it is
necessary to
▪ use basic right triangle trigonometry
▪ solve equations which involve one fractional term is also important to
know.
▪ find an angle given a right triangle ratio of sides.
▪ the fact that corresponding angles formed by parallel lines have the
same measure.
A typical problem of angles of elevation and depression involves
organizing information regarding distances and angles within a right triangle. In
some cases, you will be asked to determine the measurement of an angle; in
others, the problem might be to find an unknown distance.
Suppose a tree 50 feet in height casts a shadow of length 60 feet. What is
the angle of elevation from the end of the shadow to the top of the tree with
respect to the ground?
First, we should make a diagram to
organize our information. Look for these
diagrams to involve a right triangle. In this
case, the tree makes an angle 90º with
the ground. A diagram of this right
triangle is shown below.

295 | P a g e
MATHEMATICS
Grade Nine (9)
In the diagram, known distances are labeled. These are the 50 and 60 foot
legs of the right triangle corresponding to the height of the tree and
the length of the shadow.
The variable q is chosen to represent the unknown measurement, the
object of the question.
To relate the known distances and the variable, an equation is written. In
this case the equation involves the lengths of the sides which are opposite and
adjacent to the angle q. Using the ratio of opposite to adjacent sides, we have
50
= 𝑡𝑎𝑛𝜃
60
50
We use inverse tangent of ≈ 0.8333 or 𝜃 = 𝑡𝑎𝑛−1 (0.8333) ≈ 39.8° which is
60

the angle of elevation.


Example: A person stands at the window of a building so that his eyes are 12.6 m
above the level ground in the vicinity of the building. An object is 58.5 m away
from the building on a line directly beneath the person. Compute the angle of
depression of the person’s line of sight to the object on the ground.
Solution:
The angle of depression of the line of sight is the angle, θ, that the line of sight
makes with the horizontal, as shown in the figure to the right. Since the ground is
level, it is parallel to any horizontal line, and so the angle that the line of sight
makes with the ground is equal to θ as well. As a result, we have
12.6
tan 𝜃 = 58.5

so that
tan 𝜃 = 0.21538462
𝜃 = 12.15ᵒ

296 | P a g e
MATHEMATICS
Grade Nine (9)

Quiz
Solve each problem.
1. The angle of elevation to the top of a flagpole is 36⁰ from a point 54 meters
from the base of the flagpole. Find the height of the flagpole.
2. From the top of a tower which is 185ft. tall, the angle of depression to a house
is 12⁰. How far is the house from the base of the tower?
3. From a point 49.5 m to the base of a flagpole, the angle of elevation to the
top is 43⁰12’. Find the height of the flagpole.
4. From a window of one building, the angle of depression to the base of the
second building is 29⁰30’ and the angle of elevation to the top is 47⁰50’. If the
buildings are 200 m apart, how high is the second building?
5. From the top of a cliff 220 m high, the angle of depression of a boat is 30⁰.
How far out is the boat from the foot of the cliff?
6. The angle of elevation of the top of a tower 40 m high is 37º when seen from
a point on level ground. Find the distance of the point from the foot of the tower.
7. From the window of a building 45 m above ground level, the angle of
depression of a car on level ground is 42º. How far is the car from the building?
8. At a point 27 m from the front of a pole, the angle of elevation of the top of
the pole is 53º. Find the height of the pole.
9. A lighthouse is 72 m high. Find the angle of elevation of its top from a point
125 m away in level ground.
10. What is the angle of elevation of the top of a tree 12.7 m high from a point
23.7 m away on level ground?

297 | P a g e
MATHEMATICS
Grade Nine (9)

Lesson 60
Law of Sines
The Law of Sines (or Sine Rule) is very useful for solving triangles:

It works for any triangle:

a, b and c are sides.


A, B and C are angles.
(Side a faces angle A,
side b faces angle B and
side c faces angle C).

So when we divide side a by the sine of angle A it is equal to side b divided by


the sine of angle B, and also equal to side c divided by the sine of angle C
Well, let's do the calculations for a triangle I prepared earlier:

a/sin A = 8 / sin (62.2°) = 8 / 0.885... = 9.04...


b/sin B = 5 / sin (33.5°) = 5 / 0.552... = 9.06...
c/sin C = 9 / sin (84.3°) = 9 / 0.995... = 9.05...

298 | P a g e
MATHEMATICS
Grade Nine (9)
The answers are almost the same!
(They would be exactly the same if I used perfect accuracy).
So now you can see that:
a/sin A = b/sin B = c/sin C
Let us see an example:
Example: Calculate side "c"

Law of Sines: a/sin A = b/sin B = c/sin C

Put in the values we know: a/sin A = 7/sin(35°) = c/sin(105°)

Ignore a/sin A (not useful to us): 7/sin(35°) = c/sin(105°)

Now we use our algebra skills to rearrange and solve:

Swap sides: c/sin(105°) = 7/sin(35°)

Multiply both sides by sin(105°): c = ( 7 / sin(35°) ) × sin(105°)

Calculate: c = ( 7 / 0.574... ) × 0.966...

Calculate: c = 11.8 (to 1 decimal place)

299 | P a g e
MATHEMATICS
Grade Nine (9)
Finding an Unknown Angle
In the previous example we found an unknown side ...
... but we can also use the Law of Sines to find an unknown angle.
In this case it is best to turn the fractions upside down (sin A/a instead of a/sin A,
etc):

Example: Calculate angle B

Start with: sin A / a = sin B / b = sin C / c

Put in the values we know: sin A / a = sin B / 4.7 = sin(63º) / 5.5

Ignore "sin A / a": sin B / 4.7 = sin(63º) / 5.5

Multiply both sides by 4.7: sin B = (sin63º/5.5) × 4.7

Calculate: sin B = 0.7614...

Inverse Sine: B = sin-1(0.7614...)

B = 49.6º

300 | P a g e
MATHEMATICS
Grade Nine (9)

Quiz
Solve each triangle below.
1. A = 48⁰25’, B = 54⁰48’, b = 86.4 6. a = 857, b = 781, B = 72.7⁰
2. A = 29.5⁰ , C = 113.2⁰, c = 75 7. A = 89⁰, b = 62.9, C = 34.8⁰
3. A = 85.7⁰, b = 70.1, C = 76.6⁰ 8. a = 45 , B = 90.46⁰, C = 34.8⁰
4. a = 40. 2 , B = 85.9⁰, C = 26.4⁰ 9. b = 78.9, c = 90.6, C = 56⁰23’
5. b = 25.9, c = 38.4, C = 41⁰20’ 10. a = 78, b = 89, B = 56.8⁰

B. Solve the following triangles.

301 | P a g e
MATHEMATICS
Grade Nine (9)

Lesson 61
Law of Cosines
The Law of Cosines (also called the Cosine Rule) is very useful for solving triangles:

It works for any triangle:


a, b and c are sides.
C is the angle opposite side c

Let's see how to use it in an example:


Example: How long is side "c"

We know angle C = 37º, a = 8 and b = 11

302 | P a g e
MATHEMATICS
Grade Nine (9)
The Law of Cosines says: c2 = a2 + b2 − 2ab cos(C)

Put in the values we know: c2 = 82 + 112 − 2 × 8 × 11 × cos(37º)

Do some calculations: c2 = 64 + 121 − 176 × 0.798…

More calculations: c2 = 44.44...

Take the square root: c = √44.44 = 6.67 to 2 decimal places

Answer: c = 6.67
How to Remember
How can you remember the formula?
Well, it helps to know it's the Pythagoras Theorem with something extra so it works
for all triangles:

Pythagoras (only for Right-Angled


a2 + b2 = c2
Theorem: Triangles)

a2 + b2 − 2ab
Law of Cosines: (for all triangles)
cos(C) = c2

So, to remember it:


• think "abc": a2 + b2 = c2,
• then a 2nd "abc": 2ab cos(C),
• and put them together: a2 + b2 − 2ab cos(C) = c2
When to Use
The Law of Cosines is useful for finding:
• the third side of a triangle when we know two sides and the angle between them
(like the example above)
• the angles of a triangle when we know all three sides (as in the following
example)

303 | P a g e
MATHEMATICS
Grade Nine (9)
Example: What is Angle "C"?

The side of length "8" is opposite angle C, so it is side c. The other two sides
are a and b.
Now let us put what we know into The Law of Cosines:

Start with: c2 = a2 + b2 − 2ab cos(C)

Put in a, b and c 82 = 92 + 52 − 2 × 9 × 5 × cos(C)

Calculate: 64 = 81 + 25 − 90 × cos(C)

Calculate: 64 = 81 + 25 − 90 × cos(C)

Now we use our algebra skills to rearrange and solve:

Subtract 25 from
39 = 81 − 90 × cos(C)
both sides:

Subtract 81 from
−42 = −90 × cos(C)
both sides:

Swap sides: −90 × cos(C) = −42

Divide both sides


cos(C) = 42/90
by −90:

Inverse cosine: C = cos-1(42/90)

Calculator: C = 62.2° (to 1 decimal place)

304 | P a g e
MATHEMATICS
Grade Nine (9)
In Other Forms
We just saw how to find an angle when we know three sides. It took quite a few
steps, so it is easier to use the "direct" formula (which is just a rearrangement of
the c2 = a2 + b2 − 2ab cos(C) formula):

Example: Find Angle "C" Using the Law of


Cosines (angle version)
In this triangle we know the three sides:
a = 8, b = 6 and c=7
Use the Law of Cosines (angle version) to find
angle C:

cos C = (a2 + b2 − c2)/2ab

= (82 + 62 − 72)/2×8×6

= (64 + 36 − 49)/96

= 51/96

= 0.53125

C = cos-1(0.53125)

= 57.9° to one decimal place

305 | P a g e
MATHEMATICS
Grade Nine (9)

Quiz
A. Solve for the missing parts of triangle ABC given the following information.
1. a = 5, B = 75⁰, c = 9 2. a = 8.1, b = 6.2, c = 10.3
3. B = 65⁰, a = 11.5, c = 18.9 4. a =5.73, b = 8.24, c = 9.31
5. C = 24⁰, b = 12, a = 13 6. A = 34⁰, b = 24, c = 34
7. a = 12, B = 89⁰, c = 13 8. a = 8, b = 6, c = 10
9. B = 70⁰, a = 12, c = 19 10. a =6, b = 8, c = 9

B. Solve the following triangles.

306 | P a g e
MATHEMATICS
Grade Nine (9)

Lesson 62
Fundamental Identities
The fundamental (basic) trigonometric identities can be divided into
several groups. First are the reciprocal identities. These include

Next are the quotient identities. These include

Next there are the identities for negatives. These include

Finally, there are the Pythagorean identities. These include

Example 1
Verify the identity cos 𝛼 tan 𝛼 = sin 𝛼
sin 𝛼
cos 𝛼 tan 𝛼 = cos 𝛼 ∙ cos 𝛼
= sin 𝛼

307 | P a g e
MATHEMATICS
Grade Nine (9)
Example 2: Verify the identity

Example 3: Verify the identity

308 | P a g e
MATHEMATICS
Grade Nine (9)

Quiz
A. Verify each identity.
1 1− 𝑠𝑖𝑛2 𝑥
1) – 1 = cot2𝜃 6) = 𝑠𝑖𝑛2 𝑥
𝑠𝑖𝑛2 𝜃 𝑐𝑠𝑐 2 𝑥−1

2) sec(-x) = sec x 7) cos 𝜃 tan 𝜃 = sin 𝜃


cos 𝜃 1+sin 𝜃
3) = 8) sin 𝜃 csc 𝜃 = 1
1−sin 𝜃 cos 𝜃

4) tan 𝜃 cot 𝜃 = 1 9) sec𝜃 cos 𝜃 = 1


𝑐𝑠𝑐 2 𝑥
5) (1 + sin x) (1 –sin x) = cos2 x 10) csc x sec x = 𝑐𝑜𝑡

309 | P a g e
MATHEMATICS
Grade Nine (9)
FOURTH MONTHLY EXAM – MATHEMATICS GRADE 9

Test I. Directions: Find the numerical value of each.


2 2
1) 1253 6) 64−3

2 1
2) 273 7) 6254

2 3
3) 83 8) 92

2 1
4) 13 9) 252

2 1
5) 25 4 10) 64 3

Test II. Directions: Simplify the following.

1) 3√𝑥 + 2√𝑥 6) 2√400 + √40 + √80

2) 4√3 + 8√12 − 12√75 7) 6√6 + 4√7 − 5√6 − 4√7

3) 3√5𝑥 − 13√5𝑥 8) a√3 + 4𝑥 √3 + 2√3 − 5𝑥√3

4) 6√50 + 5√72 − 14√98 9) 3√5𝑎 − 52√5𝑎

5) 2𝑎√3 + 2𝑏 √3 + 4𝑏√3 10) 4 √7 + 2𝑥 √7 − 2 √28


3 3 3

310 | P a g e
MATHEMATICS
Grade Nine (9)
Test III. Directions: Find the product.

1. (√8)(√40) 6. (√23)(√20)

2. (√15)(√20) 7. (2√6)(√10)

3. (5 √9𝑥 2 𝑦)(√36𝑥 2 𝑦 ) 8. 2√49𝑎 • √16𝑎


4 4

4. √45𝑎2 𝑏 • √50𝑎2 𝑏 9. (3√6 + 4√6)(√6 − 3√6)

5. (√7 √4)2 10. (√5 − 3√7)(√5 − √7)

Test IV. Directions: Simplify the following.


√42 √6
1. 6.
√7 √12

√20 √2
2. 7.
√10 √8

2 √2 √27
3. 8.
2√10 √37

√5𝑥 5 1
4. 9.
√4𝑥 7 7− √3

√6 2√3+ √7
5. 10.
√5− √6 √6

311 | P a g e
MATHEMATICS
Grade Nine (9)
Test V. Directions: Solve each of the following radical equations.

1. 4 + √𝑥 = 9 7. √2𝑥 − 2 = 23
2. √𝑥 − 81 = 0 8. √3𝑥 = −9
3. √3𝑥 = 3 9. 4 − √3𝑥 = 1
4. 9 =√15𝑥 + 12 − 3 10. √𝑥 + 4 = 8 − 𝑥
5. √2𝑥 − 3 = 𝑥

6. √𝑥 + 2 + 4 = 12

Test VI. Directions: Solve the following problems.

1. Three times the square root of a number is 27. Find the numbers.

2. Twice the square of a number is 14. Find the number.

3. If 7 is subtracted from the square root of a number, the result is 45. Find the
number.

4. One-fourth of the square of a number is 2. Find the number.

5. The square root of a number when divided by 5 is equal to 2. find the number.

312 | P a g e
MATHEMATICS
Grade Nine (9)
THIRD QUARTERLY EXAM – MATHEMATICS GRADE 9

Test I. Directions: State the order of each of the following matrices.


3 2 5
1. ( ) 6. ( )
1 3 −5
2 5 6
2. ( ) 7. (1 3)
8 −4 9
3. (4 5) 8. (2 4 5 7 8)

1 3 −9
4. (1 −5 9) 9. ( 5 − 5 6)
1 −6 −7
6 7 −9
2.4
3.4 9 9 9
5. ( ) 10. ( )
−5.4 −8 8 8
−6.7

Test II. Directions: State which of the following matrices are equal.
5
A = (5 7) B=( ) C = (7 5)
7
7 3 7 −4 5
D=( ) E= ( ) F=( )
5 5 −4 7 3
3 5 3 7 7
G=( ) H=( ) I=( )
7 −4 5 −4 5
3 2 3
J = (3 2 7) K = ( 2) L = (4 5)
7 7 8
7
3 5
M = (2 3 4) N=( ) O = ( 2)
7 −4
3
2 3 4
P = (3 2 7) Q = (7 5) R=( )
5 7 8
−4 5 5 1
S=( ) T=( ) U=( )
7 3 7 5

313 | P a g e
MATHEMATICS
Grade Nine (9)
Test III. Directions: Evaluate each of the following.
2 4 3 1
1. ( )+ ( )
7 −4 8 0
2. (2 5) + (12 − 7)
3 5 7 3 −4 7 9 7
3. ( )+ ( )
−2 1 6 5 21 −3 −4 3
−4 7 4 −3
4. ( ) - ( )
−23 3 2 4
1 3
5. (0) − (23)
2 5
3 −2
6. ( ) + ( )
4 5
2 3 7 −3 5 4
7. ( )+( )
9 −4 6 −4 9 −2
3 4
8. ( ) − ( )
9 2
−5 3 7 3 −5 4
9. ( )-( )
−4 2 −4 −6 3 9
3 2 2 7
10. ( 1 5 ) – ( −5 4)
−4 6 −3 −1

314 | P a g e
MATHEMATICS
Grade Nine (9)
Test IV. Directions: Solve the following matrix equations.
4 2 7 5
1. A + ( )= ( )
1 −8 −3 4
1 −4 6 7
2. A – ( )=( )
4 8 −4 3
5 4 21 −2
3. ( )- A =( )
7 −7 0 7
2 −4 3 15 1 4
4. ( )+ 𝐴 = ( )
6 21 0 −3 45 −2
21 4 5 21 5 −6
5. A - ( )=( )
−3 0 34 0 − 34 4
6. (1 3 −4 6) + A = (21 0 −4 9)
2 −5
7. A + ( ) = ( )
−1 6
12 64
8. ( ) - A = ( )
3 −4
4 4 1 6
9. A + ( )= ( )
2 −4 −3 −4
2 −5 3 6
10. A – ( )=( )
4 3 −5 4

Test V. Directions: Simplify each of the following.

1. 3(1 5)

2. 2(1 −2 3)
0 4 1 −1 3 0
3. 3( ) − 4( )
5 0 −1 −2 1 −1
3 −1 −4 −1
4. 4( 4 5 )− 3( 0 2 )
−3 −2 −1 4
1 2 4 1 0 4 −2 4 −3
5. 2( )+ 3( ) − 5( )
−3 4 −2 −3 5 −1 0 6 −7

315 | P a g e
MATHEMATICS
Grade Nine (9)
Test VI. Directions: Solve the following equations.
4 4 16
1. b(( ) + 𝑏 ( ) = ( )
4 −4 24
2 2 12
2. a( ) + 𝑏 ( ) = ( )
2 −2 20
2𝑎 𝑎 15
3. 3( ) + 3 ( ) = ( )
𝑏 3𝑏 30
1 2 3 12 𝑏 𝑐
4. 2( )− ( )= ( )
3 4 𝑎 16 7 𝑑
5 3 2 𝑎 𝑏 𝑐 9 12 6
5. 2( )+ ( )= ( )
1 6 3 −2 −4 5 𝑑 𝑒 𝑓
𝟔 𝟒 𝟓 𝟐 𝟕 𝟒
D. If A = ( ) ,𝑩 = ( ) and C = ( ),
−𝟏𝟐 𝟕 −𝟏 𝟔 𝟏𝟐 −𝟓
find the following.

1. A + B

2. A – B – C

3. A + 2B

4. 2A – 2C + 3B

5. 3A – 2B + 4C

316 | P a g e
MATHEMATICS
Grade Nine (9)
Test VII. Directions: Find the matrix products of the following where possible.
2 4 2
1. ( )( )
1 3 7
3 1 4
2. ( ) ( )
5 3 7
5 3 4 1 2
3. ( )( )
1 12 6 3 4
1 0 3 3 4
4. ( )( )
2 0 3 1 2
4 3
2 4
5. ( 2 5) ( )
1 7
3 7
Test VIII. Directions: State whether each statement is true or false.

1. A parallelogram is a quadrilateral with both pairs of opposite sides are


parallel.

2. The sides of a quadrilateral are segments.

3. Each diagonal of a parallelogram divides the parallelogram into three


congruent triangles.

4. Opposite angles of a parallelogram are congruent.

5. Any two consecutive angles of a parallelogram are complementary.

317 | P a g e
MATHEMATICS
Grade Nine (9)
Test IX. Directions: Find the value of x in the parallelogram ABCD. Find the length
of each side.

1. x 4. AD

2. AB 5. BC

3. DC

Test X. Directions: Use the figure below to find the value of x with the given
information.

ZEAL is a parallelogram

1. ZE = 18x – 4 and LA = 6x + 12

2. YL = 12x and LE = 8x + 16

3. ZY = 14x – 10 and YA = 8x + 32

4. ZA = 36 – 2x and ZY = 6x – 10

5. m EZA = 2x + 10, m AZL = x + 5 and M EAL = 75º

6. m LZE = 6x + 32 and m ZEA = 68 + 4x

7. m ZEL = 10x + 20 and m ELA = 8x + 26.

318 | P a g e
MATHEMATICS
Grade Nine (9)
Test XI. Directions: Use rectangle JOYS to answer each question.

1. If TY = 10, find SO.

2. If SY = 24, find JO.

3. If m 3 = 29º, find m 1.

4. If m 2 = 66º, find m 5.

5. If JT = 6x + 2 and JY = 8x + 24, find OT.

6. If OT = 5x – 9 and JY = 4x + 12, find OT.

7. If JY = 2x2 and SO = 7x – 3, find x.

8. if m = 31º, find m 2.

Test XII. Directions: Use rhombus to answer each question.

1. If m TCH = 34º, find m TCA.

2. If m TNA = 115º, find m HTA.

3. If m TAC = 16º, find m TAH.

4. If m 120º, find m HTA.

5. If m HAC = 3x – 15 and m HAT = x + 5, find x.

319 | P a g e
MATHEMATICS
Grade Nine (9)
FIFTH MONTHLY EXAM – MATHEMATICS GRADE 9

Test I. Directions: Using the marked figure below to find each indicated length.
1. If HI = 12, then PO = ?
2. If HI = 21, then PO = ?
3. If HI = 30, then PO = ?
4. If PO = 24, then HI = ?
5. If PO = 28, then HI = ?
6. If PO = 19, then HI = ?
7. If HI = 4x + 6 and PO = 10x + 8, then HI = ? and PO = ?
8. If HI = 2x + 1 and PO = 6x – 4, then HI = ? and PO = ?
9. If PO = 3x – 8 and HI = x + 4, then HI = ?
10. IF PO = 28 and HI = 2x + 6, then HI = ?

Test II. Directions: In PQRS. points A, B, and C are the midpoints of PQ, PR and
RQ, respectively. Complete the following statements.
1. BC = 12, then QP =
2. PR = 20, then AC =
3. PQ = 10, then BC =
4. If BC = 7, then PQ =
5. If QR = 12, then AB =
6. If AC = 6x + 10 and PR = 8x + 32, then PR =
7. If AB = 14x – 8 and QR = 6x + 28, then QC =
8. If AB = 15 and AC = 19, then QR + PR =
9. If QA = x + 4 and PA = 2x – 5, then BC =
10. If QP = 20 and QR = 18, then BC + AB =

320 | P a g e
MATHEMATICS
Grade Nine (9)
Test III. Directions: Find the measures of the median of the given figure.

Given : AB = 108 cm

QR = 216 m

Find:

1. CD 6. MN

2. EF 7. OP

3. GH 8. KL

4. IJ

Test IV. Directions: Solve for x in each proportion.


𝑥 8 3 12
1. = 6. =
4 16 𝑥 28

3 𝑥 3 24
2. = 7. =
12 36 𝑥 40

7 4 12 𝑥
3. = 8. =
8 𝑥 18 2

𝑥−6 2 2+𝑥 𝑥+2


4. 4
= 12
9. 8
= 5

𝑥 12 6 12
5. = 10. =
4 5 𝑥 14

321 | P a g e
MATHEMATICS
Grade Nine (9)
Test V. Directions: Identify whether the following figures are similar or congruent.

1. 6.

2.

7.

3.

8.

4. 9.

5. 10.

322 | P a g e
MATHEMATICS
Grade Nine (9)
Test VI. Directions: Find the numbers represented by each letter.
1. ∆TON
t = 12
o = 15
n = 20
∆TON ~∆AME. If e = 60 find a and m.

2. ∆ABC
a=5
b=7
c=9
∆ABC ~∆XWZ. If x = 32, findw and z.

3. ∆ABC
a=4
b=6
c=8
∆ABC ~∆DEF. If d = 12, find e and f.

4. ∆ART
a = 20
r = 15
t = 18
∆ART ~∆LUV. If u = 10, find l and v.

5. ∆BSW
b = 25
s = 20
w = 25
∆BSW ~∆ARM. If r = 9, find a and m.

323 | P a g e
MATHEMATICS
Grade Nine (9)
SIXTH MONTHLY EXAM – MATHEMATICS GRADE 9

Test I. Directions: Determine whether the given information implies that SE II OR.
𝑆𝐻 𝐸𝐻 𝐻𝑂 𝑂𝑅
1. = 4. =
𝑂𝐻 𝑅𝐻 𝑆𝑂 𝐸𝑅

𝑆𝐻 𝐻𝐸 𝐻𝑆 𝐻𝐸
2. = 5. 𝑆𝑂 =
𝑆𝑂 𝐸𝑅 𝑆𝐸

𝑆𝑂 𝐸𝑅
3. =
𝑂𝐻 𝑅𝐻

Test II. Directions: Use the figure below to find each indicated length.

1. EO = 12, VO = 8, AO = 18, BO =
2. AB = 6, BO = 4, EV = 8, VO =
3. BO = 14, AB = 8, VO = 10, VE =
4. OA = 18, EO = 24, EV = 10, AB =
5. BV = 24, AE = 40, VO = 20, EO =

324 | P a g e
MATHEMATICS
Grade Nine (9)
Test III. Directions: Give the ratios of the lengths of the sides of ∆FOR and ∆BUT.

1. ∆FOR 2: 4: 5 a. ∆BUT 6: 8: 15 b. ∆BUT 8: 16: 20

2. ∆FOR 1: 2: 3 a. ∆BUT 3: 4: 5 b. ∆BUT 4: 16: 20

3. ∆FOR 2: 3: 7 a. ∆BUT 4: 6: 14 b. ∆BUT 1: 14: 20

4. ∆FOR 1: 3: 3 a. ∆BUT 3: 9: 9 b. ∆BUT 2: 4: 12

5. ∆FOR 2: 1: 7 a. ∆BUT 6: 3: 21 b. ∆BUT 12: 14: 26

6. ∆FOR 3: 4: 5 a. ∆BUT 9: 12: 20 b. ∆BUT 2: 7: 19

7. ∆FOR 2: 3: 7 a. ∆BUT 2: 1: 6 b. ∆BUT 8: 12: 28

8. ∆FOR 3: 3: 6 a. ∆BUT 1: 1: 2 b. ∆BUT 4: 4: 9

9. ∆FOR 2: 3: 4 a. ∆BUT 4: 6: 10 b. ∆BUT 6: 9: 12

10. ∆FOR 4: 8: 10 a. ∆BUT 6: 10: 12 b. ∆BUT 6: 12: 15

Test IV. Directions: Determine the measures of the missing sides.

A right triangle ABC with right angle at C.


A
1. a = 3, b = 4, c = ?

2. a = 14, b = 16, c = ?
b c
3. b = 9, c = 18, a = ?

4. b = 31, c = 35, a = ?
C a B
5. c = 100, a = 47, b = ?

325 | P a g e
MATHEMATICS
Grade Nine (9)
Test V. Directions: Given the three sides of a triangle, determine if it is acute, right,
or obtuse.

1) 2, 3, 6 6) 3, 3, 3√5

2) 6, 7, 12 7) 3, 7, 3√7

3) 8, 12, 16 8) 12, 7, 12

4) 14, 4, 3 9) 12, 5√4, 16

5) 2, 3, 5 10) 12, 14, 20

Test VI. Directions: ∆HER is 30-60-90 triangle. Find the indicated lengths.

a. HE = 12, HR = ?, RE = ?

b. HE = 8, HR = ?, RE = ?

c. HR = 15, HE = ?, RE = ?

d. HE = 5, HR = ?, RE = ?

e. HE = 10, HR = ?, RE = ?

Test VII. Directions: ∆ MEN is a 45-45-90 triangle. Find the indicated lengths.

a. MN = 22, NE = ?, ME = ?

b. NE = 24, MN = ?, ME = ?

c. ME = 8, MN = ?, NE = ?

d. NE = 16, MN = ?, ME = ?

e. ME = 10, MN = ?, NE = ?

326 | P a g e
MATHEMATICS
Grade Nine (9)
Test VIII. Directions: Find the three positive angles that are coterminal with the
following.

1) 23⁰ 4) 35°

2) -15⁰ 5) -76°

3) 378⁰

Test IX. Directions: Express each in radians.

1. 60 ⁰ 6. 44°

2. -24⁰ 7. 76°

3. 90⁰ 8. -86°

4. 106⁰ 9. 60°

5. -178⁰ 10. -23°

327 | P a g e
MATHEMATICS
Grade Nine (9)
FOURTH QUARTERLY EXAM – MATHEMATICS GRADE 9

Test I. Directions: Suppose that a and b are the lengths of the legs of a right
triangle, and c is the hypotenuse. If a is the side opposite the angle 𝜽, find the
values of the six trigonometric functions of 𝜽.

1. a = 7 b = 14

2. a = 4 c=5

3. b = 25 c = 75

5 9
4. a = 4 b=4

5. b = 4√2 c = 10√2

Test II. Directions: Change the following to decimal degrees.

1) 25⁰ 20’ 30’’ 4) 24⁰ 30’ 15’’

2) 12⁰ 15’ 15’’ 5) 45⁰ 30’ 45’’

3) 16⁰ 45’ 30’’

Test III. Directions: Change the following decimal degrees to D⁰M’S’’

1) 24. 2⁰ 4) 89.45⁰

2) 45.15⁰ 5) 16. 35⁰

3) 56.221⁰

328 | P a g e
MATHEMATICS
Grade Nine (9)
Test IV. Directions: Find each of the following function values.

1) sin 23⁰ 4) tan 45⁰

2) csc 34⁰ 5) cos 133⁰

3) cot (-34⁰)

Test V. Directions: Express each function in terms of its cofunction.

1) cos 76⁰ 4) cot 23⁰

2) sin 34⁰ 5) csc 67⁰

3) cos 46⁰

Test VI. Directions: The following sets of numbers are called Pythagorean Triples.
These numbers satisfy the Pythagorean Theorem, c2 = a2 + b2. If the given
numbers are the measurements of the sides of the right triangle ABC, find the
measures of the acute angles in each set.

1. 3, 4, 5 3. 8, 15, 17 5. 16, 63, 65

2. 5, 12, 13 4 . 13, 84, 85

Test VII. Directions: Solve the following triangles.

329 | P a g e
MATHEMATICS
Grade Nine (9)
Test VIII. Directions: Verify each identity

1. cos 𝑥 tan 𝑥 = sin 𝑥


cos 𝜃 1+sin 𝜃
2. =
1−sin 𝜃 cos 𝜃

3. sin 𝜃 csc 𝜃 = 1

4. tan 𝜃cot 𝜃 = 1

5. sec𝜃cos 𝜃 = 1

Test IX. Directions: Solve each problem.

1) The angle of elevation to the top of a flagpole is 36⁰ from a point 54 meters
from the base of the flagpole. Find the height of the flagpole.

2) From the top of a tower, which is 185ft. tall, the angle of depression to a house
is 12⁰. How far is the house from the base of the tower?

3) From a point 49.5 m to the base of a flagpole, the angle of elevation to the
top is 43⁰12’. Find the height of the flagpole.

330 | P a g e

You might also like